CEMI a la maison 9e et 10e ann ee - Lundi 23 mars 2020 ... · 19.Un nombre de Fibonacci. 20.Un...

171
CEMI ` a la maison 9e et 10e ann´ ee - Lundi 23 mars 2020 Magicien de l’addition Tu auras besoin de : Deux joueurs Une feuille de papier et un crayon Comment jouer : 1. Commence par un total de 0 (sur le papier). 2. Les deux joueurs alterneront les tours, ce qui changera le total. D´ ecidez quel joueur commence. 3. ` A ton tour, tu peux ajouter 1, 2, 3, 4, 5, 6, 7, 8, 9, ou 10 au total. Ces nombres peuvent ˆ etre utilis´ es plus d’une fois au cours du jeu. 4. Le joueur qui porte le total ` a 52 gagne la partie! Jouez ` a ce jeu plusieurs fois. Pouvez-vous trouver une strat´ egie qui vous permettra de gagner la plupart du temps? Et ` a chaque fois? Est-il pr´ ef´ erable de commencer ou de passer en second ou cela n’a-t-il pas d’importance? Variantes : Comment votre strat´ egie changerait-elle si le jeu ´ etait jou´ e jusqu’` a 55 au lieu de 52? Quelle serait votre strat´ egie si les joueurs ´ etaient autoris´ es `a utiliser les nombres de 1 ` a 15 mais devaient jouer jusqu’` a 300 (au lieu de 52)? Quelle serait votre strat´ egie si les joueurs ´ etaient autoris´ es ` a utiliser les nombres de 1 ` a n, avec n> 1, mais devaient jouer jusqu’`a un total de T o` u T est un entier positif sup´ erieur ` a3n? Plus d’infos : Consultez la page internet du CEMI ` a la maison, lundi 30 mars, pour une discussion sur une strat´ egie pour ce jeu. Nous vous encourageons ` a discuter de vos id´ ees en ligne en utilisant n’importe quel forum qui vous convient. Nous utilisons parfois des jeux dans nos concours de math´ ematiques! Allez voir Question 2 du concours Hypatie 2003 pour un autre jeu o` u nous cherchons une strat´ egie. 1

Transcript of CEMI a la maison 9e et 10e ann ee - Lundi 23 mars 2020 ... · 19.Un nombre de Fibonacci. 20.Un...

Page 1: CEMI a la maison 9e et 10e ann ee - Lundi 23 mars 2020 ... · 19.Un nombre de Fibonacci. 20.Un multiple de 11. 22.Le plus petit nombre de la grille. 23.Le troisi eme c^ot e d’un

CEMC.UWATERLOO.CA | le CENTRE d’ÉDUCATION en MATHÉMATIQUES et en INFORMATIQUE

CEMI a la maison

9e et 10e annee - Lundi 23 mars 2020Magicien de l’addition

Tu auras besoin de :

• Deux joueurs

• Une feuille de papier et un crayon

Comment jouer :

1. Commence par un total de 0 (sur le papier).

2. Les deux joueurs alterneront les tours, ce qui changera le total. Decidez quel joueur commence.

3. A ton tour, tu peux ajouter 1, 2, 3, 4, 5, 6, 7, 8, 9, ou 10 au total.

Ces nombres peuvent etre utilises plus d’une fois au cours du jeu.

4. Le joueur qui porte le total a 52 gagne la partie!

Jouez a ce jeu plusieurs fois. Pouvez-vous trouver une strategie qui vous permettra de gagner laplupart du temps? Et a chaque fois? Est-il preferable de commencer ou de passer en second ou celan’a-t-il pas d’importance?

Variantes :

• Comment votre strategie changerait-elle si le jeu etait joue jusqu’a 55 au lieu de 52?

• Quelle serait votre strategie si les joueurs etaient autorises a utiliser les nombres de 1 a 15 maisdevaient jouer jusqu’a 300 (au lieu de 52)?

• Quelle serait votre strategie si les joueurs etaient autorises a utiliser les nombres de 1 a n, avecn > 1, mais devaient jouer jusqu’a un total de T ou T est un entier positif superieur a 3n?

Plus d’infos :

Consultez la page internet du CEMI a la maison, lundi 30 mars, pour une discussion sur une strategiepour ce jeu. Nous vous encourageons a discuter de vos idees en ligne en utilisant n’importe quel forumqui vous convient.

Nous utilisons parfois des jeux dans nos concours de mathematiques! Allez voir Question 2 duconcours Hypatie 2003 pour un autre jeu ou nous cherchons une strategie.

1

Page 2: CEMI a la maison 9e et 10e ann ee - Lundi 23 mars 2020 ... · 19.Un nombre de Fibonacci. 20.Un multiple de 11. 22.Le plus petit nombre de la grille. 23.Le troisi eme c^ot e d’un

CEMC at HomeGrade 9/10 - Monday, March 30, 2020

Addition Magician - SolutionThe StrategyYou likely noticed that the player that brings the total to 42, 43, 44, 45, 46, 47, 48, 49, 50, or 51generally loses the game on the next turn. The next player can reach 52 by adding 10, 9, 8, 7, 6,5, 4, 3, 2, or 1, respectively, and so will win the game as long as they choose the correct number.Therefore, the player that brings the total to 41 is guaranteed to be able to bring the total to 52 ontheir next turn.

Using similar reasoning, the player that brings the total to 30 is guaranteed to be able to bring thetotal to 41 on their next turn. Also the player that brings the total to 19 is guaranteed to be ableto bring the total to 30 on their next turn, and the player that brings the total to 8 is guaranteed tobe able to bring the total to 19 on their next turn.

Putting all of this together, we see that there is a strategy that guarantees a win for the first player,regardless of what the second player does. (This is what is called a winning strategy for the game.)The first player starts by adding 8 to the total of 0. In the turns that follow, the first player will addwhatever is needed to bring the totals to 19, 30, 41, and then 52. Our analysis above explains whythis is always possible within the rules of the game.

Notice that the target numbers 8, 19, 30, 41, and 52 all differ by 11. We can describe the strategymore concisely as follows: Go first and start by adding 8. For all turns that follow, if the other playeradds n, then you add 11 − n.

The Variations• In the first variation, the winning total is 55 which is a multiple of 11. A winning strategy in

this variation is to go second and, on each turn, if the other player adds n, then add 11− n, sothat the total changes by 11 in total over the two turns. For example, if they add 4 then youadd 7. This way the second player will bring the total to 11, 22, 33, 44, and then 55 to win.

• In the second variation, the player that brings the total to a number between 285 and 299inclusive will generally lose the game since the next player can reach 300. Since the allowablenumbers in this variation are 1 to 15, we focus on multiples of 16. Since 300 is 12 more than amultiple of 16, the winning strategy is to go first and start with 12. Then, if the other playeradds n, you add 16 − n, so that the total changes by 16 over the two turns. For example, ifthey add 7, you add 9. The first player can always bring the total to the next number that is12 more than a multiple of 16, eventually reaching 300 to win.

• In the third variation, we need to consider different cases for T :

If T is a multiple of n + 1, then go second. Whatever number the other player chooses, youchoose the number that totals n+ 1 when summed with their chosen number. This means youwill bring the total to each multiple of n+1, in turn, eventually reaching T . (The first variationabove is an instance of this case.)

If T is not a multiple of n + 1, then go first. Find the remainder when T is divided by n + 1and start with this number. Whatever number the other player chooses, choose the numberthat totals n + 1 when summed with their chosen number. Eventually you will bring the totalto T . (The second variation above is an instance of this case.)

1

Page 3: CEMI a la maison 9e et 10e ann ee - Lundi 23 mars 2020 ... · 19.Un nombre de Fibonacci. 20.Un multiple de 11. 22.Le plus petit nombre de la grille. 23.Le troisi eme c^ot e d’un

CEMC.UWATERLOO.CA | le CENTRE d’ÉDUCATION en MATHÉMATIQUES et en INFORMATIQUE

CEMI a la maison

9e et 10e annee - Mardi 24 mars 2020Casse-tete de nombres croises

Utilisez les indices a la page suivante pour completer la grille de nombres croisesci-dessous. Chaque carre de la grille contient exactement un chiffre. Notez que cer-taines reponses peuvent etre trouvees en utilisant seulement l’indice donne tandisque d’autres necessitent les reponses a d’autres indices.Vous aurez peut-etre besoin de faire quelques recherches avant de trouver certainesreponses !

1 2 3 4 5 6 7

8 9 10

11 12 13

14 15 16 17 18

19 20 21 22

23 24 25

26 27 28 29 30

31 32 33 34

35 36 37 38 39

40 41 42

Plus d’infos :Visitez la page internet du CEMI a la maison, mardi 31 mars, pour consulter lasolution de ce casse-tete de nombres croises. Nous vous encourageons a prendre letemps de discuter et d’etudier les references de ce casse-tete qui sont nouvelles pourvous.

1

Page 4: CEMI a la maison 9e et 10e ann ee - Lundi 23 mars 2020 ... · 19.Un nombre de Fibonacci. 20.Un multiple de 11. 22.Le plus petit nombre de la grille. 23.Le troisi eme c^ot e d’un

Horizontalement

1. La somme des carres des trois premiersnombres premiers.

3. Le nombre d’annees pendant lesquelles leGrinch a supporte la fete de Noel des Choux(Whos).

5. Un cube parfait.

8. Avec 31 horizontalement , une pairede facteurs de 832.

9. 6 !

10. Un nombre premier de Mersenne.

11. Le nombre de Hardy-Ramanujan.

13. 25% de 300.

14. Combien as-tu depense si tu recois 4,17$de monnaie sur 10$ ?

16. La factorisation en nombres premiers de140.

19. Un nombre de Fibonacci.

20. Un multiple de 11.

22. Le plus petit nombre de la grille.

23. Le troisieme cote d’un triangle droit dontl’hypotenuse est 18 verticalement et

l’autre cote est 39 horizontalement .

24. Un nombre triangulaire.

25. Le nombre de bits dans 5 octets.

26. Le chiffre 9 en binaire.

29. Un palindrome.

31. Le point de congelation de l’eau en degresFahrenheit.

32. MMDXIII.

35. 32 horizontalement -

16 horizontalement .

37. Le solde apres avoir investi $100 a 3%d’interet simple pendant 8 ans.

39. Le meme chiffre repete.

40. Les 11e, 12e, et 13e chiffres de pi.

41. La valeur ASCII du caractere b minuscule.

42. Le numero atomique de l’argent.

Verticalement

1. Chiffres consecutifs en ordre decroissant.

2. 1000 de moins que l’annee de laConfederation du Canada.

3. Le nombre d’indices de ce casse-tete.

4. Le nombre d’aretes d’un icosaedre.

5. Les chiffres de 32 horizontalementdans l’ordre inverse.

6. Le plus petit multiple commun de 6 et 7.

7. Le nombre 55 en hexadecimal.

9. Le dernier chiffre est la moyenne des deuxpremiers chiffres.

12. Le chiffre du milieu est la somme des deuxautres chiffres.

13. Le plus petit nombre d’Achille.

15. Le nombre de pattes dans une ferme qui a24 poulets, 18 cochons et 33 araignees.

17. Le nombre d’annees dans 5 siecles.

18. Le nombre prefere de Sheldon Cooper.

19. La somme des angles interieurs d’un tri-angle.

21. 9 horizontalement

+ 29 horizontalement .

23. Le numero de la derniere symphonie deMozart.

25. La valeur totale (en cents) de 9 pieces de25 cents, 12 pieces de 10 cents, et 14 piecesde 5 cents.

27. Multiples de 3 consecutifs.

28. Le nombre de cotes d’un dodecagone.

30. Un carre parfait.

33. L’annee de naissance de Blaise Pascalmoins l’annee de naissance de Carl Gauss.

34. Chiffres impairs consecutifs.

35. Le plus grand diviseur commun de13 horizontalement et

17 verticalement .

36. Le nombre de minutes dans 3480 secondes.

37. La somme des chiffres de11 horizontalement .

38. Le nombre de jours dans deux quinzaines.

2

Page 5: CEMI a la maison 9e et 10e ann ee - Lundi 23 mars 2020 ... · 19.Un nombre de Fibonacci. 20.Un multiple de 11. 22.Le plus petit nombre de la grille. 23.Le troisi eme c^ot e d’un

CEMC at Home

Grade 9/10 - Tuesday, March 24, 2020

Crossnumber Puzzle - Solution

3 8 5 3 3 4 3

2 6 7 2 0 1 2 7

1 7 2 9 7 5

5 8 3 2 2 5 7

1 3 8 8 0 3

4 8 4 5 4 0

1 0 0 1 1 3 1

3 2 2 5 1 3

2 5 6 1 2 4 5 5

5 8 9 9 8 4 7

1

Page 6: CEMI a la maison 9e et 10e ann ee - Lundi 23 mars 2020 ... · 19.Un nombre de Fibonacci. 20.Un multiple de 11. 22.Le plus petit nombre de la grille. 23.Le troisi eme c^ot e d’un

CEMC.UWATERLOO.CA | le CENTRE d’ÉDUCATION en MATHÉMATIQUES et en INFORMATIQUE

CEMI a la maison

9e et 10e annee - Mercredi 25 marsConstruire une banderole

Un programme informatique peut etre utilise pour dessiner des banderoles composees de carres et detriangles. Le programme utilise les cinq instructions suivantes :

Instruction Signification

C Dessine un grand carre

c Dessine un petit carre

T Dessine un grand triangle

t Dessine un petit triangle

N[I] Repete les instructions , I, exactement N fois

Par exemple, le programme c 2[T t] C dessine la banderole suivante :

Questions :

1. Etant donne le programme t 4[c] T 3[t C], dessine la banderole correspondante.

2. Cree deux programmes differents qui dessineront la banderole suivante :

3. Etant donne le programme 2[2[c C] t T], dessine les formes manquantes dans la bande-role suivante :

4. Etant donne le programme incomplet ?[2[?] t ?[c T ?]], complete les instructionsmanquantes pour dessiner la banderole suivante :

1

Page 7: CEMI a la maison 9e et 10e ann ee - Lundi 23 mars 2020 ... · 19.Un nombre de Fibonacci. 20.Un multiple de 11. 22.Le plus petit nombre de la grille. 23.Le troisi eme c^ot e d’un

CEMC.UWATERLOO.CA | le CENTRE d’ÉDUCATION en MATHÉMATIQUES et en INFORMATIQUE

5. Suppose que tu veuilles dessiner la banderole suivante :

Tu crees le programme 2[C T t] 2[T C c] qui dessine incorrectement cette banderole :

Quelles sont les erreurs dans ton programme ?

6. Une nouvelle instruction si est maintenant disponible. L’instruction (a:d/e) signifie que sila forme precedemment dessinee etait a, alors la forme suivante dessinee est d. Si la formeprecedemment dessinee n’etait pas a, alors la forme suivante dessinee est e.

Par exemple, le programme c (c:C/t) (t:T/c) dessine la banderole suivante :

Pour chacun des programmes dans les parties de (a) a (f), decide si le programme dessinera labanderole suivante ou pas :

(a) 2[T (t:T/t)]

(b) T (T:t/c) (t:T/C)

(c) T 2[(t:T/t)]

(d) t (t:T/c) (c:C/t)

(e) T (T:t/C) (C:c/T)

(f) 3[(T:t/T)]

7. Essaye de creer tes propres nouvelles instructions. Ajoute peut-etre de nouvelles formes, oude nouvelles fonctionnalites telles que l’enchaınement vertical des formes. Echangez vos pro-grammes avec un ami ou un membre de votre famille et essayez de dessiner les banderoles del’autre.

Plus d’infos :

Visite le site du CEMI a la maison, mercredi 1er avril, pour trouver les solutions a ces questions.

Cette tache fait travailler tes muscles de la pensee informatique ! Pour plus d’informations sur le lienentre cette tache et l’informatique, consulte Chain, extrait du Beaver Computing Challenge de 2016.

2

Page 8: CEMI a la maison 9e et 10e ann ee - Lundi 23 mars 2020 ... · 19.Un nombre de Fibonacci. 20.Un multiple de 11. 22.Le plus petit nombre de la grille. 23.Le troisi eme c^ot e d’un

CEMC at Home

Grade 9/10 - Wednesday, March 25, 2020

Build a Banner - Solution

1. t 4[s] T 3[t S]

2. One possible program is s T s T t S t S.Another possible program is 2[s T] 2[t S].

3. 2[2[s S] t T]

4. 2[2[S] t 3[s T s]]

5. The small triangle instruction, t, and the small square instruction, s, should be moved outsideof their repeating blocks. The correct program is 2[S T] t 2[T S] s.

6. (a) No. The program draws this banner:

(b) Yes

(c) Yes

(d) No. The program draws this banner:

(e) Yes

(f) No. The program is invalid and will not draw any banner. The first instruction is the newif instruction, but there is no previously drawn shape that it can use in order to make adecision about what to draw next.

1

Page 9: CEMI a la maison 9e et 10e ann ee - Lundi 23 mars 2020 ... · 19.Un nombre de Fibonacci. 20.Un multiple de 11. 22.Le plus petit nombre de la grille. 23.Le troisi eme c^ot e d’un

CEMC.UWATERLOO.CA | le CENTRE d’ÉDUCATION en MATHÉMATIQUES et en INFORMATIQUE

Le CEMI a la maison presente Le probleme de la semaine

9e et 10e annee - jeudi 26 mars 2020

Quel est ce total ?

Nous connaissons le fait suivant au sujet des nombres a, b et c :

(a + b)2 = 9, (b + c)2 = 25, et (a + c)2 = 81.

Si a + b + c ≥ 1, determine le nombre de valeurs possibles de a + b + c.

Plus d’infos :

Consultez la page du CEMI a la maison, jeudi 2 avril, pour trouver la solution a ce probleme.Vous pouvez egalement vous inscrire au Probleme de la semaine en cliquant sur le lien ci-dessous etrecevoir la solution ainsi qu’un nouveau probleme, par courriel, jeudi 2 avril.

Cette ressource du CEMI a la maison correspond au Probleme de la semaine pour les 9e et 10eannees. Le Probleme de la semaine est une ressource gratuite. Chaque semaine, des problemesprovenant de divers domaines mathematiques sont publies en ligne et envoyes par courriel auxenseignants afin qu’ils les utilisent avec leurs etudiants. Les problemes sont disponibles pour lesetudiants de la 3e jusqu’a la 12e annee. Les solutions aux problemes sont envoyees une semaineapres, en meme temps que le nouveau Probleme de la semaine.

Pour plus d’informations et vous inscrire au Probleme de la semaine, rendez-vous sur :https://www.cemc.uwaterloo.ca/resources/potw-f.php

1

Page 10: CEMI a la maison 9e et 10e ann ee - Lundi 23 mars 2020 ... · 19.Un nombre de Fibonacci. 20.Un multiple de 11. 22.Le plus petit nombre de la grille. 23.Le troisi eme c^ot e d’un

Problem of the WeekProblem D and Solution

What’s that Total?

ProblemWe know the following about the numbers a, b and c:

(a+ b)2 = 9, (b+ c)2 = 25, and (a+ c)2 = 81.

If a+ b+ c ≥ 1, determine the number of possible values for a+ b+ c.

SolutionSince (a+ b)2 = 9, a+ b = ±3. Since (b+ c)2 = 25, b+ c = ±5. And since(a+ c)2 = 81, a+ c = ±9.

Now (a+ b) + (b+ c) + (a+ c) = 2a+ 2b+ 2c = 2(a+ b+ c). This quantity istwo times the value of the quantity we are looking for.

The following chart summarizes all possible combinations of values for a+ b, b+ c,and a+ c and the resulting values of 2a+2b+2c and a+ b+ c. The final columnof the chart states a yes or no answer to whether the value of a+ b+ c is ≥ 1.

a+ b+ c ≥ 1?a+ b b+ c a+ c 2a+ 2b+ 2c a+ b+ c (yes / no)3 5 9 17 8.5 yes3 5 −9 −1 −0.5 no3 −5 9 7 3.5 yes3 −5 −9 −11 −5.5 no

−3 5 9 11 5.5 yes−3 5 −9 −7 −3.5 no−3 −5 9 1 0.5 no−3 −5 −9 −17 −8.5 no

Therefore, there are three possible values of a+ b+ c such that a+ b+ c ≥ 1.

It should be noted that for each of the three possibilities, values for a, b, and c

which produce each value can be determined but that was not the question asked.

Page 11: CEMI a la maison 9e et 10e ann ee - Lundi 23 mars 2020 ... · 19.Un nombre de Fibonacci. 20.Un multiple de 11. 22.Le plus petit nombre de la grille. 23.Le troisi eme c^ot e d’un

CEMC.UWATERLOO.CA | le CENTRE d’ÉDUCATION en MATHÉMATIQUES et en INFORMATIQUE

CEMI a la maison

9e et 10e annee - Vendredi 27 mars 2020Hexaminos

Un hexamino est une forme geometrique composee de six carres de taille egale, connectes par un ouplusieurs cotes. Voici quelques exemples d’hexaminos. Essaye d’en dessiner d’autres par toi-meme.

Sur la derniere page, tu trouveras 35 hexaminos dessines et numerotes. Tous les autres hexaminospeuvent etre obtenus par translation, rotation ou symetrie d’un de ces 35 hexaminos, en utilisanteventuellement une combinaison de ces transformations. Dans les activites suivantes, tu pourras utili-ser au besoin des translations, rotations et symetries sur les 35 formes afin de resoudre les problemes.La collection de formes avec lesquelles nous allons travailler est parfois appelee les 35 hexaminos libres.

Questions :

1. Lequel ou lesquels de ces 35 hexaminos represente(nt) le patron d’un cube ? En d’autres termes,quels hexaminos forment un cube quand on les plie ? Pour t’aider a visualiser ce probleme, tupeux imprimer les hexaminos sur une feuille de papier, les decouper et les plier. Des tuilesmagnetiques fonctionneraient aussi tres bien.

2. Recouvre un rectangle de 3× 4 en utillisant deux copies de n’importe quel hexamino. Combiende solutions differentes peux-tu trouver ? (N’oublie pas que tu es libre d’utiliser des translations,rotations et/ou symetries sur la forme.)

1

Page 12: CEMI a la maison 9e et 10e ann ee - Lundi 23 mars 2020 ... · 19.Un nombre de Fibonacci. 20.Un multiple de 11. 22.Le plus petit nombre de la grille. 23.Le troisi eme c^ot e d’un

3. Recouvre le rectangle 6× 15 ci-dessous en utilisant n’importe quelle combinaison d’hexaminos.Peux-tu le faire en utilisant chaque hexamino au maximum une fois ?

4. Prends un carre de 15× 15 et retire un rectangle de 3× 5 au milieu. Recouvre les carres blancsrestants en utilisant exactement une fois chacun des 35 hexaminos.

2

Page 13: CEMI a la maison 9e et 10e ann ee - Lundi 23 mars 2020 ... · 19.Un nombre de Fibonacci. 20.Un multiple de 11. 22.Le plus petit nombre de la grille. 23.Le troisi eme c^ot e d’un

Plus d’infos :

Consulte la page du CEMI a la maison, vendredi 3 avril, pour trouver la solution d’Hexaminos.

Lorsque quatre carres de taille egale sont utilises au lieu de six, ces formes geometriques sont appeleestetrominos. Les tetrominos sont les blocs de construction du jeu original Tetris.

3

Page 14: CEMI a la maison 9e et 10e ann ee - Lundi 23 mars 2020 ... · 19.Un nombre de Fibonacci. 20.Un multiple de 11. 22.Le plus petit nombre de la grille. 23.Le troisi eme c^ot e d’un

CEMC at Home

Grade 9/10 - Friday, March 27, 2020

Hexominoes - Solution

Answers:

1. The following 11 hexominoes represent the net of a cube.

#12 #13 #14 #15 #16 #17 #20 #23 #25 #31 #35

2. There are five different solutions using hexominoes #6, #7, #22, #28, and #32.

3. Here is one possible solution.

1

Page 15: CEMI a la maison 9e et 10e ann ee - Lundi 23 mars 2020 ... · 19.Un nombre de Fibonacci. 20.Un multiple de 11. 22.Le plus petit nombre de la grille. 23.Le troisi eme c^ot e d’un

4. Here is one possible solution.

2

Page 16: CEMI a la maison 9e et 10e ann ee - Lundi 23 mars 2020 ... · 19.Un nombre de Fibonacci. 20.Un multiple de 11. 22.Le plus petit nombre de la grille. 23.Le troisi eme c^ot e d’un

CEMC.UWATERLOO.CA | le CENTRE d’ÉDUCATION en MATHÉMATIQUES et en INFORMATIQUE

CEMI a la maison

9e et 10e annee - Lundi 30 mars 2020Attention trombones

Tu auras besoin de :

• Deux joueurs

• 10 trombones(ou autres petits objets)

Comment jouer :

1. Commence avec une pile de 10 trombones.

2. Les joueurs alterneront les tours.

Decidez quel joueur commence (joueur 1) et quel joueur passe en second (joueur 2).

3. A ton tour, tu peux retirer 1, 2 ou 3 trombones de la pile.

4. Le joueur qui retire le dernier trombone, perd.

Jouez a ce jeu plusieurs fois. Alternez le joueur qui commence.

Peux-tu determiner une strategie∗ gagnante pour ce jeu?

La strategie gagnante depend-elle du fait d’etre le joueur 1 ou le joueur 2?

* Une strategie est un ensemble de regles predeterminees qu’un joueur utilisera pour jouer aujeu. La strategie dicte ce que le joueur fera pour chaque situation possible dans le jeu. Il s’agitd’une strategie gagnante si elle permet au joueur de toujours gagner, independamment de ceque fait l’autre joueur.

Y-a-t-il une connexion entre ce jeu et le jeu auquel nous avons joue le 23 mars (magicien de l’addition)?

Variantes :

A. Quel joueur a une strategie gagnante si le joueur qui retire le dernier trombone gagne (au lieude perdre)? Decris cette strategie gagnante.

B. Quel joueur a une strategie gagnante si, en plus de la variante A, les joueurs sont invites aprendre 1, 3 ou 4 trombones de la pile? Decris cette strategie gagnante.

C. Quel joueur a une strategie gagnante si, en plus des variantes A et B, la pile commence avec14 trombones? Decris cette strategie gagnante.

Plus d’infos :

Consulte la page internet du CEMI a la maison, lundi 6 avril, pour trouver la solution d’Attentiontrombones. Nous vous encourageons a discuter de vos idees en ligne en utilisant n’importe quel forumqui vous convient.

1

Page 17: CEMI a la maison 9e et 10e ann ee - Lundi 23 mars 2020 ... · 19.Un nombre de Fibonacci. 20.Un multiple de 11. 22.Le plus petit nombre de la grille. 23.Le troisi eme c^ot e d’un

CEMC at Home

Grade 9/10 - Monday, March 30, 2020

Careful Clipping - Solution

The Strategy

Let the two players be Player 1 and Player 2.

You likely noticed that the player that brings the number of paper clips in the pile to 1 is guaranteedto win the game, and the player that brings the number of paper clips in the pile to 2, 3, or 4 generallyloses the game. This is because the next player can bring the pile to 1 paper clip by removing 1, 2or 3 paper clips, respectively.

Using similar reasoning, we can show that the player that brings the number of paper clips to 5 isguaranteed to be able to bring the number to 1 on their next turn, and the player that brings thenumber of paper clips to 9 is guaranteed to be able to bring the number to 5 on their next turn. Thismeans that Player 1 has a winning strategy for this game and it goes as follows:

Start by removing 1 paper clip, reducing the total number of paper clips to 9. On your next turn,remove whatever number of paper clips are needed to bring the total to 5. On your turn after that,remove whatever number of paper clips are needed to bring the total to 1. (Our analysis aboveexplains why each of these moves will be possible within the rules of the game.)

Notice that the “target numbers” (9, 5, and 1) all differ by 4. We can instead describe the strategyas follows: Go first and start by removing 1 paper clip. For all turns that follow, if the other playerjust removed n paper clips, then you remove 4 − n paper clips. (These two turns, combined, willreduce the number of paper clips by 4.)

The Variations

Variation A

The player that reduces the pile to 1, 2, or 3 paper clips will lose the game since the next player canremove all of the remaining paper clips. Therefore, you want to be the player that reduces the pile to4 paper clips as you are guaranteed to be able to win the game on your next turn. Player 1 now hasthe following winning strategy: Start by removing 2 paper clips, reducing the pile to 8 paper clips.On your next turn, remove whatever number of paper clips are needed to bring the total to 4. Onyour turn after that, remove all remaining paper clips.

Variations B and C

In each of these variations, players can remove 1, 3, or 4 paper clips on their turn, and you win byremoving the last paper clip from the pile. Player 1 has a winning strategy starting from 10 paperclips (Variation B) and Player 2 has a winning strategy starting from 14 paper clips (Variation C).We outline these strategies in the table on the next page by analyzing how to win the game startingwith each of 1 through 14 paper clips, in turn. We give the first move(s) in each strategy and thengive guidance on how to use earlier rows in the table to fill in the rest of the strategy.

1

Page 18: CEMI a la maison 9e et 10e ann ee - Lundi 23 mars 2020 ... · 19.Un nombre de Fibonacci. 20.Un multiple de 11. 22.Le plus petit nombre de la grille. 23.Le troisi eme c^ot e d’un

Let the two players be Ally and Bri. In each game, Ally will go first.

Starting Pile Winner Reasoning

1 Player 1 Ally takes the clip and wins.

2 Player 2 Ally must take 1 clip. Bri takes the remaining clip and wins.

3 Player 1 Ally takes all 3 clips and wins.

4 Player 1 Ally takes all 4 clips and wins.

5 Player 1 Ally takes 3 clips, leaving 2 clips for Bri’s turn.As above, if a pile has 2 clips, the second player will win.Since it is Bri’s turn, the second player (starting from 2 clips) is Ally (Player 1).

6 Player 1 Ally takes 4 clips, leaving 2 clips for Bri’s turn.As above, if a pile has 2 clips, the second player will win.Since it is Bri’s turn, the second player (starting from 2 clips) is Ally (Player 1).

7 Player 2 Ally must take 1, 3, or 4 clips, leaving 6, 4, or 3 clips for Bri’s turn.As above, if a pile has 6, 4, or 3 clips, the first player will win.Since it is Bri’s turn, the first player is Bri (Player 2).

8 Player 1 Ally takes 1 clip, leaving 7 clips for Bri’s turn.As above, if a pile has 7 clips, the second player will win.Since it is Bri’s turn, the second player is Ally (Player 1).

9 Player 2 Ally must take 1, 3, or 4 clips, leaving 8, 6, or 5 clips for Bri’s turn.As above, if a pile has 8, 6, or 5 clips, the first player will win.Since it is Bri’s turn, the first player is Bri (Player 2).

10 Player 1 Ally takes 1 clip, leaving 9 clips for Bri’s turn.As above, if a pile has 9 clips, the second player will win.Since it is Bri’s turn, the second player is Ally (Player 1).

11 Player 1 Ally takes 4 clips, leaving 7 clips for Bri’s turn.As above, if a pile has 7 clips, the second player will win.Since it is Bri’s turn, the second player is Ally (Player 1).

12 Player 1 Ally takes 3 clips, leaving 9 clips for Bri’s turn.As above, if a pile has 9 clips, the second player will win.Since it is Bri’s turn, the second player is Ally (Player 1).

13 Player 1 Ally takes 4 clips, leaving 9 clips for Bri’s turn.As above, if a pile has 9 clips, the second player will win.Since it is Bri’s turn, the second player is Ally (Player 1).

14 Player 2 Ally must take 1, 3, or 4 clips, leaving 13, 11, or 10 clips for Bri’s turn.As above, if a pile has 13, 11, or 10 clips, the first player will win.Since it is Bri’s turn, the first player is Bri (Player 2).

2

Page 19: CEMI a la maison 9e et 10e ann ee - Lundi 23 mars 2020 ... · 19.Un nombre de Fibonacci. 20.Un multiple de 11. 22.Le plus petit nombre de la grille. 23.Le troisi eme c^ot e d’un

CEMC.UWATERLOO.CA | le CENTRE d’ÉDUCATION en MATHÉMATIQUES et en INFORMATIQUE

CEMI a la maison

9e et 10e annee - Mardi 31 mars 2020Separation de triangles

Le CEMI propose de nombreux concours pour inspirer la prochaine generation de mathematicien(ne)set informaticien(ne)s. Vous trouverez, ci-dessous, une question populaire tiree d’un concours Fryer(destine aux eleves de 9e annee). Au bas de la page, vous trouverez un lien vers une questionsupplementaire d’un ancien concours Galois (destine aux eleves de 10e annee).

Concours Fryer 2011, Question 2

Dans tout triangle isocele ABC dont AB = AC, la hauteur ADcoupe la base BC en son milieu, de maniere que BD = DC.

(a) (i) Dans le triangle ABC ci-contre, AB = AC = 25 etBC = 14. Determinez la longueur de la hauteur AD.

(ii) Determinez l’aire du triangle ABC.

A

C14

B

2525

D

(b) On coupe le triangle precedent ABC le long de sa hauteur, de A jusqu’a D (Figure 1). On faitsubir a chaque triangle qui en resulte une rotation de 90◦ de centre D jusqu’a ce que les pointsB et C se rencontrent au-dessous du point D (Figure 2). On obtient ainsi un nouveau triangleque l’on nomme PQR (Figure 3).

2525

B D C

Figure 1

25 25

BD

C

Figure 2

D

Figure 3

P

Q

R

(i) Determinez la longueur de la base PR du triangle PQR.

(ii) Determinez l’aire du triangle PQR.

(c) Il existe deux triangles isoceles differents dont les lon-gueurs de cotes sont des entiers et dont l’aire est egalea 120. Un de ces triangles, le triangle XY Z, est indiqueci-contre. Determinez les longueurs des trois cotes dudeuxieme triangle.

X Z

Y

30

1717

Plus d’infos :

Consultez la page internet du CEMI a la maison, mardi 7 avril, pour la solution de Separationde triangles. Pour une question supplementaire tiree du concours Galois, essayez la question 1 duConcours Galois 2016.

1

Page 20: CEMI a la maison 9e et 10e ann ee - Lundi 23 mars 2020 ... · 19.Un nombre de Fibonacci. 20.Un multiple de 11. 22.Le plus petit nombre de la grille. 23.Le troisi eme c^ot e d’un

CEMC at Home

Grade 9/10 - Tuesday, March 31, 2020

Splitting Triangles - Solution

Fryer 2011 Contest, Question 2

(a) (i) Since AB = AC, then 4ABC is isosceles. Therefore, the altitude AD bisects the base

BC so that BD = DC =14

2= 7. Since ∠ADB = 90◦, then 4ADB is right angled. By

the Pythagorean Theorem, 252 = AD2 + 72 or AD2 = 252 − 72 or AD2 = 625− 49 = 576,and so AD =

√576 = 24, since AD > 0.

(ii) The area of 4ABC is1

2×BC × AD or

1

2× 14× 24 = 168.

(b) (i) Through the process described, 4ADB is rotated 90◦ counter-clockwise about D to be-come 4PDQ. Similarly, 4ADC is rotated 90◦ clockwise about D to become 4RDQ.Through both rotations, the lengths of the sides of the original triangles remain unchanged.Thus, PD = AD = 24 and RD = AD = 24. Since P , D and R lie in a straight line, thenbase PR = PD +RD = 24 + 24 = 48.

(ii) When4ADC is rotated 90◦ clockwise about D, side DC becomes altitude DQ in4PQR.

Therefore, DQ = DC = 7. Thus, the area of 4PQR is1

2×PR×DQ or

1

2×48×7 = 168.

Note: The area of 4PQR is equal to the area of 4ABC from part (a)(ii). This is because4ABC is composed of 4ADB and 4ADC, and 4PQR is composed of rotated copies ofthese two right triangles.

(c) Since XY = Y Z, then 4XY Z is isosceles. Draw altitude YW from Y to W on XZ. Altitude

YW bisects the base XZ so that XW = WZ =30

2= 15, as shown. Since ∠YWX = 90◦, then

4YWX is right angled. By the Pythagorean Theorem, 172 = YW 2 +152 or YW 2 = 172− 152

or YW 2 = 289 − 225 = 64, and so YW =√64 = 8, since YW > 0. By reversing the process

described in part (b), we rotate 4XWY clockwise 90◦ about W and similarly rotate 4ZWYcounter-clockwise 90◦ about W . By the note at the end of the solution to part (b), the newisosceles triangle and the given isosceles triangle will have the same area. The new triangleformed has two equal sides of length 17 (since XY and ZY form these sides) and a third sidehaving length twice that of YW or 2 × 8 = 16 (since the new base consists of two copies ofYW ).

1

Page 21: CEMI a la maison 9e et 10e ann ee - Lundi 23 mars 2020 ... · 19.Un nombre de Fibonacci. 20.Un multiple de 11. 22.Le plus petit nombre de la grille. 23.Le troisi eme c^ot e d’un

CEMC.UWATERLOO.CA | le CENTRE d’ÉDUCATION en MATHÉMATIQUES et en INFORMATIQUE

CEMI a la maison

9e et 10e annee - Mercredi 1er avril 2020Ou suis-je ?

Jouons a un jeu ! Il s’agit d’un jeu sans strategie ce qui le distingue des autres jeux auxquels nousavons joue jusqu’a present. Tous nos mouvements seront decides en lancant une piece de monnaie.Pour les besoins de ce jeu, tu devras etiqueter quatre espaces differents : � chambre �, � salle debain �, � cuisine �et � salon �. Les espaces choisis peuvent etre de vraies pieces de la maison ouquatre morceaux de papier repartis dans une seule piece, chacun portant une de ces etiquettes. Tuauras egalement besoin d’une piece de monnaie.

Commence dans la chambre (ou debout sur le papier portant l’etiquette � chambre �) et lance lapiece de monnaie. En fonction du resultat du lancer, deplace-toi en suivant les regles suivantes :

— Si tu es dans la chambre et que la piece de monnaie tombe sur � face �, va dans le salon. Si lapiece tombe sur � pile �, va dans la salle de bain.

— Si tu es dans la salle de bain et que la piece de monnaie tombe sur � face �, va dans la cuisine.Si la piece tombe sur � pile �, va dans le salon.

— Si tu es dans la cuisine et que la piece de monnaie tombe sur � face �, va dans le salon. Si lapiece tombe sur � pile �, va dans la chambre.

— Si tu es dans le salon et que la piece de monnaie tombe sur � face �, va dans la cuisine. Si lapiece tombe sur � pile �, reste dans le salon.

Continue a lancer la piece de monnaie et a te deplacer de piece en piece jusqu’a ce que tu aies lancela piece de monnaie 10 fois. Le but de ce jeu est de terminer dans la cuisine. Dans laquelle des quatrepieces de la maison etais-tu a la fin du jeu ?

Les regles de ce jeu peuvent etre illustrees a l’aide du schema suivant :

chambre salle de bain cuisine

salon

P

FP

F

P

F

P

F

1

Page 22: CEMI a la maison 9e et 10e ann ee - Lundi 23 mars 2020 ... · 19.Un nombre de Fibonacci. 20.Un multiple de 11. 22.Le plus petit nombre de la grille. 23.Le troisi eme c^ot e d’un

CEMC.UWATERLOO.CA | le CENTRE d’ÉDUCATION en MATHÉMATIQUES et en INFORMATIQUE

Ce schema est connu sous le nom d’automate fini ou finite state machine (FSM) en anglais. Lescercles (pieces de la maison) sont les etats. Les fleches entre les cercles sont appeles transitions etdecrivent comment changer d’etat en fonction de la condition. La condition, dans notre jeu, est lapiece de monnaie qui a pour resultat soit � face �(F) ou � pile �(P). La fleche venant de � nullepart � et pointant vers un cercle indique l’etat de depart. Dans notre jeu, l’etat de depart est lachambre. Le double cercle indique un etat d’acceptation. Un etat d’acceptation identifie un resultatsouhaite. Dans notre jeu, le resultat souhaite est la cuisine. Note qu’en fonction de la sequence desconditions (les lancers de la piece de monnaie), tu pouvais ou non avoir terminer le jeu dans la cuisine.

Les automates finis sont des modeles. L’utilisation d’un automate fini pour modeliser un processuste permet d’analyser le processus sans avoir a le mettre en oeuvre.

Utilise le modele d’automate fini de notre jeu pour t’aider a repondre aux questions suivantes concer-nant le jeu.

Questions :

1. Biyu lance la piece de monnaie 6 fois avec les resultats suivants : P F P F P F. Dans quellepiece de la maison Biyu termine-t-il ?

2. Salmaan lance la piece de monnaie 10 fois avec les resultats suivants : P P F P F F P P F P.Dans quelle piece de la maison Salmaan termine-t-il ?

3. Leticia lance la piece de monnaie 7 fois avec les resultats suivants : F F P P ? P P. Si Leticiatermine dans la salle de bain, quel etait le resultat de son 5e lancer de la piece de monnaie ?

4. Pablo lance la piece de monnaie 12 fois. Son dernier lancer tombe sur � pile �. Pablo termine-t-ildans la cuisine ? Oui, non, ou peut-etre (en fonction de la sequence reelle) ?

5. Rashida lance la piece de monnaie 9 fois. Son dernier lancer tombe sur � face �. Rashidatermine-t-elle dans la cuisine ? Oui, non, ou peut-etre (en fonction de la sequence reelle) ?

6. Armando lance la piece de monnaie 3 fois. Dans quelle piece Armando ne peut-il pas terminer ?

Plus d’infos :

Consulte la page internet du CEMI a la maison, mercredi 8 avril, pour la solution de Ou suis-je ?

A un niveau tres abstrait, tous les ordinateurs sont des automates finis qui passent d’un etat a l’autreen fonction des donnees recues. Pour en savoir plus sur ce sujet, tu peux visionner les videos desCercles mathematiques, notamment celle-ci sur les Automates finis enregistree a l’automne 2018.

2

Page 23: CEMI a la maison 9e et 10e ann ee - Lundi 23 mars 2020 ... · 19.Un nombre de Fibonacci. 20.Un multiple de 11. 22.Le plus petit nombre de la grille. 23.Le troisi eme c^ot e d’un

CEMC at Home

Grade 9/10 - Wednesday, April 1, 2020

Where Am I? - Solution

bedroom bathroom kitchen

living room

T

H

T

H

T

H

T

H

Answers:

1.START T H T H T H

bedroom bathroom kitchen bedroom living room living room kitchen

Biyu finishes in the kitchen.

2.START T T H T H H T T H T

bed bath living kitchen bed living kitchen bed bath kitchen bed

Salmaan finishes in the bedroom.

3. After the first 4 coin tosses, Leticia must have ended up in the bathroom. Her 5th coin tossmust have landed either “heads” or “tails”. If it landed “heads”, then she would have movedto the kitchen, and from the kitchen, her remaining 2 coin tosses would have taken her tothe bathroom. If it landed “tails”, then she would have moved to the living room, and herremaining 2 coin tosses would have kept her in the living room. Since we are told that Leticiafinished in the bathroom, her 5th coin toss must have landed “heads”.

4. Notice that the only way to arrive in the kitchen is for a coin toss to land “heads”. SincePablo’s last coin toss landed on “tails”, it is not possible for Pablo to finish in the kitchen.

5. It is not possible to tell which room Rashida finishes in without knowing the results of herother coin tosses. For example, suppose her first 8 coin tosses all land “heads”. In this case,Rashida will not finish in the kitchen. However, if her first 8 coin tosses all land “tails”, thenRashida will finish in the kitchen.

6. Armando’s 3 coin tosses will result in one of 8 different combinations of “heads” and “tails”.Trying each combination and tracking which room Armando finishes in reveals that for nocombination does Armando finish in the bathroom.

1

Page 24: CEMI a la maison 9e et 10e ann ee - Lundi 23 mars 2020 ... · 19.Un nombre de Fibonacci. 20.Un multiple de 11. 22.Le plus petit nombre de la grille. 23.Le troisi eme c^ot e d’un

CEMC.UWATERLOO.CA | le CENTRE d’ÉDUCATION en MATHÉMATIQUES et en INFORMATIQUE

Le CEMI a la maison presente Le probleme de la semaine

9e et 10e annee - jeudi 2 avril 2020

Cerf-volant

Amanda aimerait faire voler un cerf-volant. Le cerf-volant est compose de deux tri-angles isoceles : 4ABD et 4BCD. La hauteur de 4BCD est 2 fois plus grandeque la hauteur de 4ABD et la largeur du cerf-volant BD est 1,5 fois la hauteur duplus grand triangle.

Si l’aire du cerf-volant est 1800 cm2, quel est le perimetre du cerf-volant ?

Savais-tu que dans un triangle isocele, la hauteur a partir du cote non congru bissectece cote non congru ?

Plus d’infos :

Consultez la page du CEMI a la maison, jeudi 9 avril, pour trouver la solution a ce probleme.Vous pouvez egalement vous inscrire au Probleme de la semaine en cliquant sur le lien ci-dessous etrecevoir la solution ainsi qu’un nouveau probleme, par courriel, jeudi 9 avril.

Cette ressource du CEMI a la maison correspond au Probleme de la semaine pour les 9e et 10eannees. Le Probleme de la semaine est une ressource gratuite. Chaque semaine, des problemesprovenant de divers domaines mathematiques sont publies en ligne et envoyes par courriel auxenseignants afin qu’ils les utilisent avec leurs etudiants. Les problemes sont disponibles pour lesetudiants de la 3e jusqu’a la 12e annee. Les solutions aux problemes sont envoyees une semaineapres, en meme temps que le nouveau Probleme de la semaine.

Pour plus d’informations et vous inscrire au Probleme de la semaine, rendez-vous sur :https://www.cemc.uwaterloo.ca/resources/potw-f.php

1

Page 25: CEMI a la maison 9e et 10e ann ee - Lundi 23 mars 2020 ... · 19.Un nombre de Fibonacci. 20.Un multiple de 11. 22.Le plus petit nombre de la grille. 23.Le troisi eme c^ot e d’un

Problem of the WeekProblem D and Solution

Go Fly a Kite

Problem

Amanda wants to fly a kite. The kite is composed of two isosceles triangles, 4ABD and4BCD. The height of 4BCD is 2 times the height of 4ABD, and the width of the kite, BD,is 1.5 times the height of the larger triangle. If the area of the kite is 1800 cm2, what is theperimeter of the kite?

SolutionLet the height of 4ABD be AE = x. Therefore, the height of 4BCD isCF = 2x. Also, the width of the kite is BD = 3x. Therefore, the base of eachtriangle is 3x.

The area of 4BCD =(3x)(2x)

2= 3x2 and the area of 4ABD =

(3x)(x)

2=

3x2

2.

Also,area of kite ABCD = area of 4BCD + area of 4ABD

= 3x2 +3x2

2

=9x2

2

Therefore, 9x2

2= 1800

9x2 = 3600

x2 = 400

x = 20, since x > 0

Page 26: CEMI a la maison 9e et 10e ann ee - Lundi 23 mars 2020 ... · 19.Un nombre de Fibonacci. 20.Un multiple de 11. 22.Le plus petit nombre de la grille. 23.Le troisi eme c^ot e d’un

Now, to find the perimeter of the kite, we need to find the lengths of the sides ofthe kite.Since 4ABD is isosceles, E will bisect BD and thereforeDE = BE = 1.5x = 30. This is shown below in the diagram to the left.Similarily, 4BCD is isosceles, F will bisect BD, and thereforeDF = BF = 1.5x = 30. This is shown below in the diagram to the right.

Using the Pythagorean Theorem in 4AED,AD2 = 202 + 302

= 400 + 900

= 1300

AD =√1300, since AD > 0.

Also AB = AD =√1300 cm.

Similiarily in 4DFC, DC2 = 302 + 402

= 2500

DC = 50, since DC > 0.

Also, BC = DC = 50 cm.

Now, the perimeter of the kite =√1300 +

√1300 + 50 + 50

= 2√1300 + 100

≈ 172.1

Therefore, the exact perimeter is 2√1300 + 100 cm or approximately 172.1 cm.

Note:The expression

√1300 can be simplified as follows:√

1300 =√100× 13 =

√100×

√13 = 10

√13.

Therefore, the exact perimeter is2√1300 + 100 = 2(10

√13) + 100 = 20

√13 + 100 cm.

Page 27: CEMI a la maison 9e et 10e ann ee - Lundi 23 mars 2020 ... · 19.Un nombre de Fibonacci. 20.Un multiple de 11. 22.Le plus petit nombre de la grille. 23.Le troisi eme c^ot e d’un

CEMC.UWATERLOO.CA | le CENTRE d’ÉDUCATION en MATHÉMATIQUES et en INFORMATIQUE

CEMI a la maison

9e et 10e annee - Vendredi 3 avril 2020

Fete surprise

Tu prepares une fete surprise pour ton amie Eve. Pour eviter qu’Eve ne decouvre les details de la fete,les organisateurs et toi decidez de communiquer en code. Tu as choisi de coder tes messages en utilisantun chiffrement par substitution connu sous le nom de chiffre de Cesar. Un chiffrement par substitutionfonctionne en remplacant systematiquement chaque lettre (ou symbole) d’un message par une lettre (ou unsymbole) different. Le chiffre de Cesar implique de � decaler � l’alphabet.

Afin de coder des messages en utilisant le chiffre de Cesar, ton groupe doit d’abord choisir un entier kcompris entre 1 et 25 inclusivement. Cet entier k est appele la cle de chiffrement et determine de combiende caracteres l’alphabet sera decale. Pour chiffrer un message (c’est-a-dire, pour passer du texte normalau code) chacune des lettres du message est remplacee par la lettre qui apparaıt k positions a droite dansl’alphabet. Par exemple, pour chiffrer le message F E T E en utilisant la cle 3, la lettre F est remplacee parI, qui est 3 positions plus loin a droite et le message original F E T E devient le message chiffre (ou code)I H W H.

Prends note que si tu ne peux pas deplacer de k positions vers la droite dans l’alphabet, alors tu retournesau debut. Par exemple, la lettre qui se trouve 3 positions a droite de Y est B.

Pour dechiffrer un message (c’est-a-dire, pour changer le code en texte normal) chaque lettre du messagecode est remplacee par la lettre qui apparaıt en deplacant k positions vers la gauche, en retournant au debutsi necessaire. Par exemple, pour dechiffrer le message F O R Z Q en utilisant la meme cle de chiffrement 3, lalettre F est remplacee par la lettre C et le message code F O R Z Q peut etre revele comme etant le messageC L O W N.

Pour les questions ci-dessous, cree ta propre roue de dechiffrement du chiffre de Cesar (voir derniere page)pour t’aider a chiffrer et dechiffrer. Par ailleurs, si tu as des connaissances en programmation, tu peux creerun programme informatique capable de chiffrer et dechiffrer des messages en fonction du texte et de la cledonnee.

Questions :

1. En utilisant la cle 6, chiffre le message L A F E T E C O M M E N C E A S E P T H E U R E S.

2. En utilisant la cle 24, dechiffre le message J C R F C K C C Q R D Y L R Y G Q G C.

3. Les autres organisateurs de la fete t’ont envoye le message suivant mais la cle a ete egaree. Peux-tu ledechiffrer ? Astuce : la lettre la plus utilisee dans la langue francaise est le E.

P F R P Y P L X P Y P C L W P R L E P L F P E W P D O P N Z C L E T Z Y D

Plus d’infos :

Consulte la page internet du CEMI a la maison, jeudi 9 avril, pour la solution de Fete surprise.

Pour un chiffrement par substitution un peu plus difficile, consulte le Systeme de chiffrement Vatsyayana.

1

Page 28: CEMI a la maison 9e et 10e ann ee - Lundi 23 mars 2020 ... · 19.Un nombre de Fibonacci. 20.Un multiple de 11. 22.Le plus petit nombre de la grille. 23.Le troisi eme c^ot e d’un

La roue de dechiffrement du chiffre de Cesar

Imprime et decoupe les deux cercles suivants. Superpose le plus petit cercle sur le plus grand et attache lesau centre en utilisant une attache parisienne.

Fais tourner les cercles pour que les A soit alignes. Ensuite, definis ta cle en faisant tourner le cercle interieurdans le sens inverse des aiguilles d’une montre. Sur le schema ci-dessous, la cle est definie a 3.

Tu es maintenant pret(e) a chiffrer et a dechiffrer ! Pour chiffrer, remplace chaque lettre de la roue exterieurepar sa lettre correspondante sur la roue interieure. Pour dechiffrer, remplace chaque lettre sur le cercleinterieur par sa lettre correspondante sur le cercle exterieur.

2

Page 29: CEMI a la maison 9e et 10e ann ee - Lundi 23 mars 2020 ... · 19.Un nombre de Fibonacci. 20.Un multiple de 11. 22.Le plus petit nombre de la grille. 23.Le troisi eme c^ot e d’un

CEMC at Home

Grade 9/10 - Friday, April 3, 2020

Surprise Party - Solution

Answers:

1. The encrypted message is V G X Z E Y Z G X Z Y G Z Y K B K T.

2. The decrypted message is T H E T H E M E I S F A N T A S Y.

3. One way to decode a message that is encrypted using a Caesar Cipher, when the key is unknown,is to try all possible keys until one key produces a message that makes sense. There are only25 possible keys, so this wouldn’t take too long.

A more clever way is to take advantage of letter frequencies in the English language. The mostcommon letter in the English language is E. The most common letter in the encrypted messageis P. This means that a good guess might be that the letter E has been shifted to the letter P.This would make the key equal to 11. Using a key of 11, the decrypted message is:

E U G E N E W I L L B R I N G T H E C A K E A N D D E C O R A T I O N S

Note: An attempt to break a substitution cipher by using knowledge of commonly used letters orphrases in a language, as we did above, is an example of what is called frequency analysis. Forfrequency analysis to be as reliable as possible, we want to study as much text, encrypted using thesame cipher, as we can. If we have only a short message to work with, then it is very possible thatthe letter E will not be the most frequently occurring letter in the original message (and we will betricked). If we have a very long message, or a very large quantity of messages, chances are good thatwithin a few tries we will have found the right match for the letter E.

1

Page 30: CEMI a la maison 9e et 10e ann ee - Lundi 23 mars 2020 ... · 19.Un nombre de Fibonacci. 20.Un multiple de 11. 22.Le plus petit nombre de la grille. 23.Le troisi eme c^ot e d’un

CEMC.UWATERLOO.CA | le CENTRE d’ÉDUCATION en MATHÉMATIQUES et en INFORMATIQUE

CEMI a la maison

9e et 10e annee - Lundi 6 avril 2020

Jeunes poussesTu auras besoin de :

— Deux joueurs

— Une feuille de papier et un crayon

Comment jouer :

1. Commence avec deux ou trois points sur la page, raisonnablement espaces.

2. Les joueurs alternent les tours. Decidez quel joueur commence.

3. A ton tour, fais les actions suivantes (si possible, en fonction des restrictions indiquees en 4) :

Trace une courbe reliant les deux points existants et ajoute un point a la courbe nouvellement tracee.Note que cette courbe peut etre tracee entre deux points differents ou sous la forme d’une boucle apartir d’un point vers lui-meme.

4. Voici les restrictions concernant les mouvements effectues en 3 :

— Tu ne peux pas tracer une courbe si cela a pour resultat qu’un point ait plus de trois segmentsde courbe qui entrent ou sortent du point. En particulier, tu ne peux pas tracer une boucle surun point qui a deja plus d’un segment de courbe entrant ou sortant.

— Tu ne peux pas tracer une courbe si elle doit croiser une courbe existante.

— Le point ajoute ne peut pas etre place sur un point existant.

5. La derniere personne qui reussit a tracer une nouvelle courbe, en suivant les regles, gagne la partie !

Exemple d’un jeu complet commencant avec 2 points :

Debut Joueur 1 Joueur 2 Joueur 1 Joueur 2relie A a B, ajoute C relie A a A (boucle), ajoute D relie C a B, ajoute E relie B a E, ajoute F

Remarque qu’apres ces quatre tours, le joueur 1 ne peut pas tracer de nouvelle courbe. Le joueur 1 ne peutpas tracer de courbe partant de A puisqu’il y a deja trois segments de courbe qui entrent ou sortent de A(avec les deux de la boucle). C’est la meme chose pour les points B, C, et E. Le joueur 1 ne peut pas relierD a F car la courbe croiserait une courbe existante et il ou elle ne peut pas dessiner de boucle sur D ou Fcar ils ont chacun deja deux segments de courbe entrant ou sortant. Par consequent, le joueur 2 gagne !

Jouez a ce jeu plusieurs fois, en commencant avec 2 points. Note le nombre total de tours qu’ilfaut pour gagner chaque partie. Y-a-t-il un certain nombre de tours apres lequel la partie est garantie d’etreterminee ?

Jouez a ce jeu plusieurs fois, en commencant avec 3 points. Y-a-t-il un certain nombre de toursapres lequel la partie est garantie d’etre terminee ? Comment compares-tu cette reponse a ta reponse pourle jeu commencant avec 2 points ?

Plus d’infos : Consulte la page internet du CEMI a la maison, mardi 14 avril, pour trouver une discussionsur Jeunes pousses.

1

Page 31: CEMI a la maison 9e et 10e ann ee - Lundi 23 mars 2020 ... · 19.Un nombre de Fibonacci. 20.Un multiple de 11. 22.Le plus petit nombre de la grille. 23.Le troisi eme c^ot e d’un

CEMC at Home

Grade 9/10 - Monday, April 6, 2020

Sprouts - Solution

Sprouts starting with 2 dots

While playing games of Sprouts starting with 2 dots, you may have noticed that each game endedafter at most 5 turns. Did you also notice that each game included at least 4 turns?

Sprouts starting with 3 dots

While playing games of Sprouts starting with 3 dots, you may have noticed that each game endedafter at most 8 turns. Did you also notice that each game included at least 6 turns?

We encourage you to think about each of these observations and see if you can explain why thishappened. We provide a discussion below to explain why any game starting with 3 dots must endafter at most 8 turns.

A game of Sprouts starting with 3 dots must end after at most 8 turns

First, we note that the game is over as soon as no dots in the gamecan be part of a newly drawn curve. Remember that all dots in thegame can have at most three curve segments attached to them. Wewill think of each dot as having three “slots” that can be filled. Curvesegments can be attached to a dot M in a few different ways as shownbelow.

Curve drawn from Mto another dot

This curve takes up oneslot on dot M

Loop drawn at M

This loop takes up twoslots on dot M

M added to a newlydrawn curve

This curve/loop takes up twoslots on dot M

The game starts with three dots and no curves. Since there are three dots in total, each having threeslots, the game starts with nine available slots.

After one turn is complete, one of two things has happened:

• a loop was added to one of the three dots, and a fourth dot was added to this loop, or

• a curve was drawn between two of the three dots, and a fourth dot was added to this curve.

In either case, after the first turn, there will be eight available slots remaining. Here we explain why:

1

Page 32: CEMI a la maison 9e et 10e ann ee - Lundi 23 mars 2020 ... · 19.Un nombre de Fibonacci. 20.Un multiple de 11. 22.Le plus petit nombre de la grille. 23.Le troisi eme c^ot e d’un

Drawing a loop on a dot fills two of the three available slots on that particular dot, reducing us to9− 2 = 7 slots available for the three original dots. However, a fourth dot is also added to this loop.Since two slots of this new dot are already taken, there is exactly one slot open on this new dot.This means there are 7 + 1 = 8 slots available among the four dots now in the game. Notice that thesituation is similar if a curve is drawn from one dot to another dot: we fill two slots (one on each dotat the ends of this curve), but gain one new slot from the fourth dot that is added.

In a similar way, we can argue that for each turn that follows, there is a net total loss of one slot perturn. After turn 2 there are 7 slots left, after turn 3 there are 6 slots left, and so on. If the gamemakes it to turn 8, then there can be only 1 slot left after turn 8 is complete. Since there must beat least 2 slots available in order for a new curve to be drawn, we can be sure that there is no legalmove to make on turn 9. Therefore, we see that any game starting with 3 dots must end after atmost 8 turns.

Did one of your games last exactly 8 turns? Note that our discussion above does not argue that agame can actually make it all the way to 8 turns, just that it is impossible for a game to make itto 9 turns. Below is an example of a game that lasted exactly 8 turns, which shows that 8 is themaximum number of turns attainable in a game starting with 3 dots.

Move Endpoints Added Point1 A and B D2 B and C E3 A and C F4 B and D G5 E and C H6 A and F I7 G and I J8 H and J K

Now try the following on your own:

• Explain why a game of Sprouts starting with 3 dots must last for at least 6 turns.

• Determine if a game of Sprouts starting with 3 dots can end in exactly 6 turns.

The game of Sprouts was invented by mathematicians John H. Conway (who died recently) andMichael S. Paterson at Cambridge University.

2

Page 33: CEMI a la maison 9e et 10e ann ee - Lundi 23 mars 2020 ... · 19.Un nombre de Fibonacci. 20.Un multiple de 11. 22.Le plus petit nombre de la grille. 23.Le troisi eme c^ot e d’un

CEMC.UWATERLOO.CA | le CENTRE d’ÉDUCATION en MATHÉMATIQUES et en INFORMATIQUE

CEMI a la maison

9e et 10e annee - Mardi 7 avril 2020Code additionnel

Dans ce casse-tete, chacune des lettres de l’alphabet represente un nombre entier different de 1 a26. Ta tache consiste a determiner quel nombre est associe a chaque lettre. Pour commencer, on tedonne : H = 20 et N = 17. Utilise les equations algebriques pour dechiffrer le code et trouver lesautres correspondances.

A B C D E F G H I J K L M

20

N O P Q R S T U V W X Y Z

17

1 2 3 4 5 6 7 8 9 10 11 12 13 14 15 16 ��17 18 19 ��20 21 22 23 24 25 26

Equations algebriques

E = D × D

B = E − D

H = E + D

B = T × D

H = D × C

J = C − T

V = C × C

Y × Y = P + I

Y + M = P − Y

P = V + 1

R = F − R

S = R − J

A = K + L

U = K × T

Z = O + W − K

O = W + C

X = T × C

Q = G − N + U

Plus d’infos :

Consulte la page internet du CEMI a la maison, mardi 14 avril, pour la solution de Code additionnel.

1

Page 34: CEMI a la maison 9e et 10e ann ee - Lundi 23 mars 2020 ... · 19.Un nombre de Fibonacci. 20.Un multiple de 11. 22.Le plus petit nombre de la grille. 23.Le troisi eme c^ot e d’un

CEMC at Home

Grade 9/10 - Tuesday, April 7, 2020

Sum Code - Solution

Answers

A B C D E F G H I J K L M

8 12 5 4 16 22 19 20 10 2 7 1 14

N O P Q R S T U V W X Y Z

17 18 26 23 11 9 3 21 25 13 15 6 24

Explanation

Since we are told that H = 20, and there are two equations involving the letter H, a good place tostart is with these equations.

E = D × D

B = E − D

H = E + D

B = T × D

H = D × C

J = C − T

V = C × C

Y × Y = P + I

Y + M = P − Y

P = V + 1

R = F − R

S = R − J

A = K + L

U = K × T

Z = O + W − K

O = W + C

X = T × C

Q = G − N + U

The equation H = D × C tells us that D and C are a factor pair of 20. This means they could be 2

and 10 (in some order) or 4 and 5 (in some order). Note that they cannot be 1 and 20. (Why not?)

The equations E = D × D and V = C × C tell us more about this factor pair. If the factor pair is 2and 10, then E and V are 4 and 100 (in some order). This is not possible since the numbers in thiscode only range from 1 to 26. Therefore, it must be the case that the factor pair D and C are 4 and5 (in some order) which means that E and V are 16 and 25 (in some order).

Suppose D = 5 and C = 4. Then E = 25 and V = 16. Using the equation H = E + D we get that H= 25 + 5 = 30 which we know must be false. Since this is not the correct order of the factor pair,we know we must have D = 4 and C = 5. In this case, we get E = 16 and V = 25. We confirm withequation H = E + D that we get H = 20 as expected.

We now know for certain the values of D, C, E and V. By substituting these values into all of therelevant equations above, we can also determine the values for B, T, J, P, and X.

To proceed further, consider the equation Y × Y = P + I. This tells us that P + I is a perfectsquare. What does this tell us about possible values for I and Y? What does this information,combined with the equation Y + M = P − Y, reveal about the value of M?

By substituting values we already know into equations, and combining equations that contain com-mon letters, we can proceed to crack the rest of the code, as indicated in the answer key above.

1

Page 35: CEMI a la maison 9e et 10e ann ee - Lundi 23 mars 2020 ... · 19.Un nombre de Fibonacci. 20.Un multiple de 11. 22.Le plus petit nombre de la grille. 23.Le troisi eme c^ot e d’un

CEMC.UWATERLOO.CA | le CENTRE d’ÉDUCATION en MATHÉMATIQUES et en INFORMATIQUE

CEMI a la maison

9e et 10e annee - mercredi 8 avril 2020Acheter local

La semaine derniere, cinq personnes (Charlie, Manuel, Priya, Sal, et Tina) ont fait leur courses aumarche de producteurs locaux. Chaque personne y est allee un jour different (de lundi a vendredi), aachete un article different (carottes, bleuets, tomates, pommes ou patates) et a depense une sommed’argent differente (1,50 $, 2,00 $, 2,50 $, 3,50 $, ou 3,75 $).

Utilise les indices ci-dessous pour determiner qui y est alle chaque jour, ce qu’ils ont achete et pourcombien.

1. Sal est alle au marche deux jours avant Priya.

2. Manuel a depense 3,75 $ au marche la veille du jour ou quelqu’un a achete des tomates.

3. Charlie a paye 2,50 $ pour des carottes le jour apres que quelqu’un ait depense 3,50 $.

4. La personne qui y est allee mercredi a achete des pommes.

5. Quelqu’un a achete des patates pour 2,00 $ lundi.

Le tableau suivant pourra etre utile pour organiser ta solution.

Lundi

Mardi

Mercredi

Jeudi

Vendredi

Carottes

Bleuets

Tom

ates

Pom

mes

Patates

1,50

$

2,00

$

2,50

$

3,50

$

3,75

$

Charlie

Manuel

Priya

Sal

Tina

1,50 $

2,00 $

2,50 $

3,50 $

3,75 $

Carottes

Bleuets

Tomates

Pommes

Patates

Plus d’infos :

Consulte la page internet du CEMI a la maison, mercredi 15 avril, pour une solution de Acheterlocal.

Ce type de casse-tete est connu sous le nom de casse-tete logique ou � puzzle logique � et nous enincluons parfois dans le Probleme de la semaine. En voici un qui s’appelle Un carnaval d’hiver.

1

Page 36: CEMI a la maison 9e et 10e ann ee - Lundi 23 mars 2020 ... · 19.Un nombre de Fibonacci. 20.Un multiple de 11. 22.Le plus petit nombre de la grille. 23.Le troisi eme c^ot e d’un

CEMC at Home

Grade 9/10 - Wednesday, April 8, 2020

Buying Local - Solution

Answer

• Charlie bought carrots for $2.50 on Friday

• Manuel bought apples for $3.75 on Wednesday

• Priya bought tomatoes for $3.50 on Thursday

• Sal bought blueberries for $1.50 on Tuesday

• Tina bought potatoes for $2.00 on Monday

Explanation

There are many different ways to arrive at the answers above. You may have used the chart providedwith the problem to keep track of matches that were confirmed or deemed impossible while examiningand combining the different clues. Below we present an explanation in words only. It may be helpfulto follow along by filling out the chart given with the problem as you read.

The apples were purchased on Wednesday (clue 4). The potatoes were purchased for $2.00 on Monday(clue 5). The carrots were not purchased on Tuesday because then the potatoes would cost $3.50instead of $2.00 (clue 3). So the carrots were purchased for $2.50 on either Thursday or Friday. Thetomatoes were not purchased on Tuesday because then the potatoes would cost $3.75 instead of $2.00(clues 2 and 5). The tomatoes were not purchased on Friday because if they were, then the carrotswere purchased on Thursday, and the carrots would cost $3.75 instead of $2.50 (clues 2 and 3).

So the potatoes were purchased for $2.00 on Monday, the blueberries were purchased on Tuesday,the apples were purchased on Wednesday, the tomatoes were purchased on Thursday, and the carrotswere purchased for $2.50 on Friday.

Manuel spent $3.75 on Wednesday purchasing apples (clue 2). Charlie spent $2.50 on Friday purchas-ing carrots, and someone spent $3.50 on Thursday purchasing tomatoes (clue 3). Since the potatoescost $2.00 (clue 5) someone spent $1.50 on Tuesday purchasing blueberries. Priya, Sal, and Tinawent to the market on Monday, Tuesday, and Thursday in some order. Since Sal went two daysbefore Priya (clue 1), then it must be the case that Sal went on Tuesday, Priya went on Thursday,and Tina went on Monday.

1

Page 37: CEMI a la maison 9e et 10e ann ee - Lundi 23 mars 2020 ... · 19.Un nombre de Fibonacci. 20.Un multiple de 11. 22.Le plus petit nombre de la grille. 23.Le troisi eme c^ot e d’un

CEMC.UWATERLOO.CA | le CENTRE d’ÉDUCATION en MATHÉMATIQUES et en INFORMATIQUE

Le CEMI a la maison presente Le probleme de la semaine

9e et 10e annee - jeudi 9 avril 2020

Des jetons jetes

Trois sacs contiennent des jetons. Le sac vert contient 22 jetons verts circulaires, chacun numeroteavec un nombre entier entre 1 et 22. Le sac rouge contient 15 jetons rouges triangulaires, chacunnumerote avec un nombre entier entre 1 et 15. Le sac bleu contient 10 jetons bleus carres, chacunnumerote avec un nombre entier entre 1 et 10.

Chaque jeton dans un des sacs a la meme probabilite d’etre pige qu’un autre jeton dans le meme sac.Il y a un total de 22 × 15 × 10 = 3300 combinaisons differentes de jetons creees en pigeant un jetonde chaque sac. Note que piger le jeton rouge avec le 7, le jeton bleu avec le 5 et le jeton vert avec le3 est une combinaison differente que piger le jeton rouge avec le 5, le jeton bleu avec le 7 et le jetonvert avec le 3. L’ordre n’est pas important.

Tu piges un jeton de chaque sac. Quelle est la probabilite que deux jetons ou plus soient numerotesavec un 5 ?

Plus d’infos :

Consultez la page du CEMI a la maison, jeudi 16 avril, pour trouver la solution a ce probleme.Vous pouvez egalement vous inscrire au Probleme de la semaine en cliquant sur le lien ci-dessous etrecevoir la solution ainsi qu’un nouveau probleme, par courriel, jeudi 16 avril.

Cette ressource du CEMI a la maison correspond au Probleme de la semaine pour les 9e et 10eannees. Le Probleme de la semaine est une ressource gratuite. Chaque semaine, des problemesprovenant de divers domaines mathematiques sont publies en ligne et envoyes par courriel auxenseignants afin qu’ils les utilisent avec leurs etudiants. Les problemes sont disponibles pour lesetudiants de la 3e jusqu’a la 12e annee. Les solutions aux problemes sont envoyees une semaineapres, en meme temps que le nouveau Probleme de la semaine.

Pour plus d’informations et vous inscrire au Probleme de la semaine, rendez-vous sur :https://www.cemc.uwaterloo.ca/resources/potw-f.php

1

Page 38: CEMI a la maison 9e et 10e ann ee - Lundi 23 mars 2020 ... · 19.Un nombre de Fibonacci. 20.Un multiple de 11. 22.Le plus petit nombre de la grille. 23.Le troisi eme c^ot e d’un

Problem of the WeekProblem D and Solution

Tokens Taken

ProblemThree bags each contain tokens. The green bag contains 22 round green tokens, each with adifferent integer from 1 to 22. The red bag contains 15 triangular red tokens, each with adifferent integer from 1 to 15. The blue bag contains 10 square blue tokens, each with adifferent integer from 1 to 10.Any token in a specific bag has the same chance of being selected as any other token from thatsame bag. There is a total of 22× 15× 10 = 3300 different combinations of tokens created byselecting one token from each bag. Note that selecting the 7 red token, the 5 blue token and 3green token is different than selecting the 5 red token, 7 blue token and the 3 green token. Theorder of selection does not matter.You select one token from each bag. What is the probability that two or more of the selectedtokens have the number 5 on them?SolutionSolution 1

There are 22 different numbers which can be chosen from the green bag, 15 different numberswhich can be chosen from the red bag, and 10 different numbers which can be chosen from theblue bag. So there are a total of 22× 15× 10 = 3300 different combinations of numbers whichcan be produced by selecting one token from each bag.

To count the number of possibilities for a 5 to appear on at least two of the tokens, we willconsider cases.

1. Each of the selected tokens has a 5 on it.This can only occur in 1 way.

2. A 5 appears on the green token and on the red token but not on the blue token.There are 9 choices for the blue token excluding the 5. A 5 can appear on the greentoken and on the red token but not on the blue token in 9 ways.

3. A 5 appears on the green token and on the blue token but not on the red token.There are 14 choices for the red token excluding the 5. A 5 can appear on the greentoken and on the blue token but not on the red token in 14 ways.

4. A 5 appears on the red token and on the blue token but not on the green token.There are 21 choices for the green token excluding the 5. A 5 can appear on the redtoken and on the blue token but not on the green token in 21 ways.

Summing the results from each of the cases, the total number of ways for a 5 to appear on atleast two of the tokens is 1 + 9 + 14 + 21 = 45. The probability of 5 appearing on at least twoof the tokens is 45

3300= 3

220.

Page 39: CEMI a la maison 9e et 10e ann ee - Lundi 23 mars 2020 ... · 19.Un nombre de Fibonacci. 20.Un multiple de 11. 22.Le plus petit nombre de la grille. 23.Le troisi eme c^ot e d’un

Solution 2

This solution uses a known result from probability theory. If the probability of event Aoccurring is a, the probability of event B occurring is b, the probability of event C occurring isc, and the results are not dependent on each other, then the probability of all three eventshappening is a× b× c.

The probability of a specific number being selected from the green bag is 122

and theprobability of any specific number not being selected from the green bag is 21

22.

The probability of a specific number being selected from the red bag is 115

and the probabilityof any specific number not being selected from the red bag is 14

15.

The probability of a specific number being selected from the blue bag is 110

and the probabilityof any specific number not being selected from the blue bag is 9

10.

In the following we will use P (p, q, r) to mean the probability of p being selected from thegreen bag, q being selected from the red bag, and r being selected from the blue bag. So,P (5, 5, not 5) means that we want the probability of a 5 being selected from the green bag, a 5being selected from the red bag, and anything but a 5 being selected from the blue bag.

Probability of 5 being selected from at least two of the bags= Probability of 5 from each bag+ Probability of 5 from exactly 2 bags= P(5, 5, 5) + P(5, 5, not 5) + P(5, not 5, 5) + P(not 5, 5, 5)

=1

22× 1

15× 1

10+

1

22× 1

15× 9

10+

1

22× 14

15× 1

10+

21

22× 1

15× 1

10

=1

3300+

9

3300+

14

3300+

21

3300

=45

3300

=3

220

The probability of 5 appearing on at least two of the tokens is3

220.

Page 40: CEMI a la maison 9e et 10e ann ee - Lundi 23 mars 2020 ... · 19.Un nombre de Fibonacci. 20.Un multiple de 11. 22.Le plus petit nombre de la grille. 23.Le troisi eme c^ot e d’un

CEMC.UWATERLOO.CA | le CENTRE d’ÉDUCATION en MATHÉMATIQUES et en INFORMATIQUE

CEMI a la maison

9e et 10e annee - mardi 14 avril 2020Quelques pieces de monnaie

Dans le pays peu connu de Galbrathia, il n’existe que deux types de monnaie : une piece de monnaiede 5 J et un billet de 9 J. En utilisant cette monnaie, tu peux representer certaines sommes, mais pasd’autres. Par exemple, tu peux representer 19 J en utilisant deux pieces de 5 J et un billet de 9 J, maisil n’est pas possible de representer 17 J en utilisant ces pieces de monnaie et ces billets.

Notre objectif est de determiner la plus grande somme d’argent qui ne peut pas etre representee en utilisantces billets et pieces de monnaie.

1. Montre comment tu peux representer 23 J en utilisant ces billets et pieces de monnaie.

2. Explique pourquoi tu ne peux pas representer 17 J en utilisant ces billets et pieces de monnaie.

3. Le tableau ci-dessous montre une maniere de representer chacune des valeurs de 40 J a 44 J, inclus :

Montant (en J) Nombre de pieces de monnaie de 5 J Nombre de billets de 9 J40 8 041 1 442 3 343 5 244 7 1

Explique comment utiliser ces informations pour representer chacune des valeurs de 45 J a 49 J,inclus.

Pour commencer, reflechis a la facon dont les informations contenues dans le tableau sur 40 Jpeuvent t’aider a representer 45 J.

4. Explique comment les informations de la question 3 nous permettent de dire que l’on peutrepresenter toutes sommes d’argent d’au moins 40 J.

Pour commencer, reflechis a la maniere dont tu pourrais representer les valeurs de 50 J a 54 J.

5. Quel est le montant le plus eleve N J qui ne peut pas etre represente en utilisant ces billets etpieces de monnaie ?

Pour repondre completement a cette question, tu devras faire quelques recherches pour determinerla valeur de l’entier positif N . Ensuite, tu devras demontrer que N J ne peut pas, en fait etrerepresente. Enfin, tu devras demontrer que toutes les valeurs a partir de (N + 1) J peuvent etrerepresentees. Tu vas probablement vouloir � travailler a rebours � en utilisant ce que tu as apprisdans les questions 3 et 4...

Extensions :A. Il existe trois facons de representer 90 J en utilisant ces billets et ces pieces de monnaie : en utilisant

18 pieces de monnaie, en utilisant 9 pieces de monnaie et 5 billets ou en utilisant 10 billets. (Pourpasser d’une facon a l’autre, on a echange 9 pieces de monnaie contre 5 billets, puisque ceux-ci ontla meme valeur.) Combien y a-t-il de facons de representer 900 J en utilisant ces billets et piecesde monnaie ?

B. Dans le pays voisin de Pnoll, les timbres sont emis pour des montants de 7 K, 9 K, et 11 K. Quelest le montant le plus eleve d’affranchissement qui ne peut pas etre effectue a l’aide de ces timbres ?

Plus d’infos :Consulte la page internet du CEMI a la maison, mardi 21 avril, pour une solution a Quelques pieces demonnaie.

1

Page 41: CEMI a la maison 9e et 10e ann ee - Lundi 23 mars 2020 ... · 19.Un nombre de Fibonacci. 20.Un multiple de 11. 22.Le plus petit nombre de la grille. 23.Le troisi eme c^ot e d’un

CEMC at Home

Grade 9/10 - Tuesday, April 14, 2020

Some Loose Change - Solution

1. Since 23 J = 2 × 9 J + 1 × 5 J, then we can can use 2 bills and 1 coin.

2. Since 2 × 9 is greater than 17, then to make 17 J, we must use either zero bills or one bill.If zero 9 J bills are used, then the entire 17 J must be made using 5 J coins.Since 17 is not divisible by 5, this is not possible.If one 9 J bill is used, then the remaining 8 J must be made using 5 J coins.Since 8 is not divisible by 5, this is not possible.Therefore, it is not possible to make 17 J using these bills and coins.

3. We start with the combinations of coins and bills that make up each of 40 J through 44 J,inclusive.If we add one 5 J coin to each of these combinations, we add 5 J to each total to get totalsfrom 45 J through 49 J, inclusive.We can summarize these in a table:

Amount (in J) # of 5 J coins # of 9 J bills40 8 041 1 442 3 343 5 244 7 145 9 046 2 447 4 348 6 249 8 1

4. Adding one 5 J coin to each of the combinations that make five consecutive amounts (like 40 Jto 44 J) allows us to make the next five consecutive amounts (in this case, 45 J to 49 J).Because we can continue to do this indefinitely, we can make any amount that is at least 40 J.Given any such amount, can you describe a quick way to determine a number of bills and coinsthat can be used to make this amount?

5. The table below shows that we can make each amount from 32 J to 36 J, inclusive:

Amount (in J) # of 5 J coins # of 9 J bills32 1 333 3 234 5 135 7 036 0 4

Repeating the argument from 4., this table shows that we can in fact make every amount thatis at least 32 J.

Page 42: CEMI a la maison 9e et 10e ann ee - Lundi 23 mars 2020 ... · 19.Un nombre de Fibonacci. 20.Un multiple de 11. 22.Le plus petit nombre de la grille. 23.Le troisi eme c^ot e d’un

It is impossible to make 31 J using only these coins and bills. (Can you modify the argumentfrom 2. to show this?)Since we cannot make 31 J and we can make every amount that is at least 32 J, then 31 J isthe largest amount that we cannot make using these bills and coins.

Extensions:

A. We can make 900 J using 100 bills and 0 coins.Since 5 bills and 9 coins have the same value, we can trade 5 bills for 9 coins.This means that we can make 900 J using 95 bills and 9 coins.How many times can you repeat this process before no further trades are possible?

B. Try working with just 7 K and 9 K stamps first and work through similar steps to 3., 4., and5. from earlier.Once you know the largest amount that cannot be made with just 7 K and 9 K stamps, tryintroducing 11 K stamps to see what amounts that you previously couldn’t make can now bemade.

Page 43: CEMI a la maison 9e et 10e ann ee - Lundi 23 mars 2020 ... · 19.Un nombre de Fibonacci. 20.Un multiple de 11. 22.Le plus petit nombre de la grille. 23.Le troisi eme c^ot e d’un

CEMC.UWATERLOO.CA | le CENTRE d’ÉDUCATION en MATHÉMATIQUES et en INFORMATIQUE

CEMI a la maison

9e et 10e annee - mercredi 15 avril 2020Retourner des verres d’eau

Dans cette activite, on retourne des verres d’eau selon un ensemble de regles afin d’atteindre certainsobjectifs.

Materiel : Trois verres d’eau vides.

Regles :— On doit retourner deux verres adjacents a chaque tour.— On obtient un verre d’eau a l’envers lorsqu’on retourne un verre

d’eau a l’endroit.— On obtient un verre d’eau a l’endroit lorsqu’on retourne un verre

d’eau a l’envers. a l’endroit a l’envers

Activites :1. Arrangez vos trois verres comme dans la figure ci-contre. Tout en

respectant les regles enoncees ci-dessus, est-il possible de retournerles verres de maniere qu’ils soient tous a l’endroit ? Si oui, combiende tours faut-il ?

2. Au depart, il y a huit arrangements de positions differentes pour les trois verres.(a) On voit deux tels arrangements dans le tableau ci-dessous. Completez le reste du tableau.

(b) Tout en respectant les regles, a partir duquel (ou desquels) des huit arrangements dedepart est-il possible de retourner les verres de maniere que les trois verres soient tous al’endroit ?

(c) Dans la partie (b), que remarquez-vous a propos du nombre initial de verres a l’envers dansles scenarios ou vous pouvez retourner les trois verres afin qu’ils soient tous a l’endroit ?

Extension : Dans la figure ci-contre, on voithuit verres dans un arrangement specifique.Tout en respectant les regles, est-il possible deretourner les verres de maniere qu’ils soient tousa l’endroit ?

Plus d’infos :

Consultez la page du CEMI a la maison jeudi, le 22 avril, pour la solution a ce probleme.

Vous pouvez modeliser vos solutions a l’aide d’un automate fini des memes manieres que dans laquestion Glasses du defi informatique Beaver de 2012 et dans la ressource Ou suis-je du 1 avril duCEMI a la maison.

1

Page 44: CEMI a la maison 9e et 10e ann ee - Lundi 23 mars 2020 ... · 19.Un nombre de Fibonacci. 20.Un multiple de 11. 22.Le plus petit nombre de la grille. 23.Le troisi eme c^ot e d’un

CEMC at Home

Grade 9/10 - Wednesday, April 15, 2020

Flipping Glasses - Solution

1. It is possible to flip the three glasses in two turns so that all three glasses are facing up.

Here are images describing one possible solution:

Start Turn 1 Turn 2

To visualize the transformation of the glasses after each turn (i.e. facing up or down), we canuse a finite state machine:

– Each circle represents a possible state of the three glasses, where D represents a glass facingdown and U represents a glass facing up.

– The starting state is the circle with the arrow “out of nowhere” pointing to it (DUD).

– To transition between states, follow an arrow by performing the action describing it.

– The desired state is the circle with the double border around it (final state).

DUD

UDD DDU

UUU

flipfirst

two glasse

s flip last two glasses

flip last two glasses flipfirst

two glasse

s

From the diagram above, we see that there are two different ways to arrive at the desired state (UUU)in exactly two turns (or two transitions from the starting state). One of these two solutions wasoutlined earlier (using images).

Note: If we flip the first and the third glass from the starting state, this would also result in thedesired state of UUU. However, this is not a valid transition since these glasses are not adjacent.

Page 45: CEMI a la maison 9e et 10e ann ee - Lundi 23 mars 2020 ... · 19.Un nombre de Fibonacci. 20.Un multiple de 11. 22.Le plus petit nombre de la grille. 23.Le troisi eme c^ot e d’un

2. (a) Here are the eight different starting positions for the three glasses.

1 2 3 4

5 6 7 8

(b) You are able to flip all three glasses so that they are facing up only if you start withposition 1, 5, 6, or 7. Starting position 1 already has all three glasses facing up, so wedon’t have to do anything. In question 1, we showed that the glasses starting in position7 can be flipped so that all three glasses are facing up. Positions 5 and 6 can be flippedto UUU in one turn. Another way to think about this is to notice that states 1, 5, 6, and7 are exactly the states in the finite state machine from Question 1. By following arrows,it is possible to move between any two of these four states. In particular, it is possible toget from any of states 5, 6, and 7 to state 1.

You may have also figured out that it is impossible to reach UUU from the starting states2, 3, 4, or 8. An explanation of this will be provided in part (c).

(c) In each of the starting positions for which it is possible to flip the glasses so that all threeare facing up, there are either 0 or 2 glasses facing down.

On each turn, two adjacent glasses are flipped causing one of the following three outcomes:

∗ If the two glasses were facing down, the number of glasses facing up increases by two.

∗ If the two glasses were facing up, the number of glasses facing up decreases by two.

∗ If one glass was facing up and the other was facing down, the number of glasses facingup does not change.

Therefore, on each turn, the number of glasses facing down either increases by two, de-creases by two, or stays the same. A consequence of this is that if the number of glassesfacing down at the beginning is odd, then the number of glasses facing down will alwaysbe odd, and hence can never be 0.

In starting positions 2, 3, 4, and 8, an odd number of glasses are facing down. This meansthat no sequence of valid flips can change any of them to the state UUU (which has 0 glassesfacing down). Put differently, a state must have an even number of glasses facing down forthere to be a chance to change it to UUU. In part (b), we we confirmed that it is actuallypossible to do this for all states starting with an even number of glasses facing down.

Extension: It is not possible to have all eight glasses facing up. We can generalize our reasoningfrom 2(c) beyond three glasses. The observation at the end of part (c) is true for any number ofglasses. That is, if there are an odd number of glasses facing down at the beginning, then there willalways be an odd number of glasses facing down. Since we start with an odd number of glasses (five)facing down, the number of glasses facing down will always be odd, and hence will never be 0. Atbest, we can reach a state where 1 glass is facing down and 7 glasses are facing up. Can you convinceyourself that it is indeed possible to reach a state where exactly one glass is facing down?

Page 46: CEMI a la maison 9e et 10e ann ee - Lundi 23 mars 2020 ... · 19.Un nombre de Fibonacci. 20.Un multiple de 11. 22.Le plus petit nombre de la grille. 23.Le troisi eme c^ot e d’un

CEMC.UWATERLOO.CA | le CENTRE d’ÉDUCATION en MATHÉMATIQUES et en INFORMATIQUE

Le CEMI a la maison presente Le probleme de la semaine

9e et 10e annee - jeudi 16 avril 2020

Des Ps, des Qs et des Rs

Des nombres a trois chiffres, PQR, QQP et PQQ sont formes en utilisant les chiffres P , Q et R defacon a ce que :

Determine toutes les valeurs possibles de P , Q et R qui vont satisfaire les deux equations.

Plus d’infos :

Consulte la page du CEMI a la maison, jeudi 23 avril, pour la solution a ce probleme.Tu peux egalement t’inscrire au Probleme de la semaine en cliquant sur le lien ci-dessous et recevoirla solution ainsi qu’un nouveau probleme, par courriel, jeudi 23 avril.

Cette ressource du CEMI a la maison correspond au Probleme de la semaine pour les 9e et 10eannees. Le Probleme de la semaine est une ressource gratuite. Chaque semaine, des problemesprovenant de divers domaines mathematiques sont publies en ligne et envoyes par courriel auxenseignants afin qu’ils les utilisent avec leurs etudiants. Les problemes sont disponibles pour lesetudiants de la 3e jusqu’a la 12e annee. Les solutions aux problemes sont envoyees une semaineapres, en meme temps que le nouveau Probleme de la semaine.

Pour plus d’informations et t’inscrire au Probleme de la semaine, rendez-vous sur :https://www.cemc.uwaterloo.ca/resources/potw-f.php

1

Page 47: CEMI a la maison 9e et 10e ann ee - Lundi 23 mars 2020 ... · 19.Un nombre de Fibonacci. 20.Un multiple de 11. 22.Le plus petit nombre de la grille. 23.Le troisi eme c^ot e d’un

Problem of the WeekProblem D and Solution

Mind Your Ps and Qs and Rs

Problem

The letters P, Q, and R represent single digits. Determine all possible combinations of valuesof P , Q and R, given that: PQR + 2 = PQQ and PQR× 2 = QQP

SolutionFrom PQR + 2 = PQQ, we note that the number PQQ is 2 more than the number PQR andthat the first two digits of each number are the same. From this, Q = R + 2 follows. We canprove this using place value. The number PQR = 100× P + 10×Q+R = 100P + 10Q+R.The number PQQ = 100× P + 10×Q+Q = 100P + 10Q+Q.Then,

(100P + 10Q+R) + 2 = 100P + 10Q+Q

100P + 10Q+R + 2 = 100P + 11Q

R + 2 = Q, as above.From this expression we can obtain a restriction on the possible integer values of R. R must bean integer from 0 to 7, inclusive. If R = 8, then R = Q+ 2 = 10 and Q is not a single digit.

From here, we will show two possible solutions.

Solution 1Representing PQR× 2 = QQP using place value,

(100P + 10Q+R)× 2 = 100Q+ 10Q+ P

200P + 20Q+ 2R = 110Q+ P

199P = 90Q− 2R

Substituting R + 2 for Q : 199P = 90(R + 2)− 2R

199P = 90R + 180− 2R

P =88R + 180

199

We are looking for an integer value of R from 0 to 7 such that 88R + 180 is a multiple of 199.The only value of R that produces a multiple of 199 when substituted into 88R+ 180 is R = 7.When R = 7, P = 88R+180

199= 88(7)+180

199= 4 and Q = R + 2 = 9.

Therefore, the only values that satisfy the system of equations are P = 4, Q = 9 and R = 7.

We can verify this result:PQR + 2 = 497 + 2 = 499 = PQQ and

PQR× 2 = 497× 2 = 994 = QQP.

Page 48: CEMI a la maison 9e et 10e ann ee - Lundi 23 mars 2020 ... · 19.Un nombre de Fibonacci. 20.Un multiple de 11. 22.Le plus petit nombre de la grille. 23.Le troisi eme c^ot e d’un

Solution 2If R must be an integer from 0 to 7, inclusive, then Q must be an integer from 2 to 9, inclusive.From the second equation, we also know that P would be the units digit of 2R.We can now use a table to determine all possible values for P ,Q and R

Q R 2R P PQR QQP Does 2× PQR = QQP?2 0 0 0 020 220 No3 1 2 2 231 332 No4 2 4 4 442 444 No5 3 6 6 653 556 No6 4 8 8 864 668 No7 5 10 0 075 770 No8 6 12 2 286 882 No9 7 14 4 497 994 Yes

We will now need to verify that the final row also works for PQR + 2 = PQQ.

PQR + 2 = 497 + 2 = 499 = PQQ.

Therefore, the only values that satisfy the system of equations are P = 4, Q = 9 and R = 7.

Page 49: CEMI a la maison 9e et 10e ann ee - Lundi 23 mars 2020 ... · 19.Un nombre de Fibonacci. 20.Un multiple de 11. 22.Le plus petit nombre de la grille. 23.Le troisi eme c^ot e d’un

CEMC.UWATERLOO.CA | le CENTRE d’ÉDUCATION en MATHÉMATIQUES et en INFORMATIQUE

Le CEMI a la maison

9e et 10e annee - vendredi 17 avril 2020Defis de cure-dents

Dans les puzzles de cure-dents suivants, on te presente des cure-dents ainsi que des instructions asuivre. Ta tache consiste a arranger, a rearranger ou a retirer des cure-dents afin d’obtenir le resultatsouhaite.

Exemple

Dans la figure ci-dessous, on voit 10 cure-dents dont la disposition cree 3 carres. Cree une figure quicontient exactement 2 carres en deplacant 2 cure-dents. (La solution se trouve sur la deuxieme page.)

Inverser le poisson

Dans la figure ci-dessous, on voit 8 cure-dents dont la disposition cree un poisson oriente vers lagauche. Cree un poisson identique dont l’orientation est vers la droite en deplacant seulement 3 cure-dents.

Supprimer des carres

Dans la figure ci-dessous, on voit 40 cure-dents dont la disposition cree un quadrillage 4 × 4. Onremarque que ce quadrillage contient de nombreux carres de differentes tailles. Retire exactement 9cure-dents afin qu’il ne reste aucun carre de quelconque taille.

Peux-tu compter le nombre total de carres dans cette figure ? Indice : Il y en a plus que 17.

1

Page 50: CEMI a la maison 9e et 10e ann ee - Lundi 23 mars 2020 ... · 19.Un nombre de Fibonacci. 20.Un multiple de 11. 22.Le plus petit nombre de la grille. 23.Le troisi eme c^ot e d’un

CEMC.UWATERLOO.CA | le CENTRE d’ÉDUCATION en MATHÉMATIQUES et en INFORMATIQUE

Triangles

Cree exactement 4 triangles equilateraux en utilisant exactement 6 cure-dents. Chacun des cotes destriangles doit avoir une longueur egale a celle d’un cure-dent.

Indice : Ta solution ne pourra etre posee a plat.

Solution a l’exemple

On a donc deux carres ; soit un grand carre 4 × 4 et un petit carre 1 × 1.

Plus d’infos :

Consulte la page du CEMI a la maison vendredi, le 24 avril, pour la solution a ce probleme.

2

Page 51: CEMI a la maison 9e et 10e ann ee - Lundi 23 mars 2020 ... · 19.Un nombre de Fibonacci. 20.Un multiple de 11. 22.Le plus petit nombre de la grille. 23.Le troisi eme c^ot e d’un

CEMC at Home

Grade 9/10 - Friday, April 17, 2020

Toothpick Challenges - Solutions

Reversing FishMove the circled toothpicks to their corresponding places noted.

Many Squares to No SquaresRemove the circled toothpicks.

TrianglesMake a triangular based pyramid. (A regular tetrahedron.)

1

Page 52: CEMI a la maison 9e et 10e ann ee - Lundi 23 mars 2020 ... · 19.Un nombre de Fibonacci. 20.Un multiple de 11. 22.Le plus petit nombre de la grille. 23.Le troisi eme c^ot e d’un

CEMC.UWATERLOO.CA | le CENTRE d’ÉDUCATION en MATHÉMATIQUES et en INFORMATIQUE

Le CEMI a la maison

9e et 10e annee - le lundi 20 avril 2020Splash

Materiel :— Deux joueurs— Un plateau de jeu � Splash � qui est un quadrillage rectangulaire dont le coin inferieur gauche

contient une tache (soit un � Splash �).

Comment y jouer :

1. Commencez avec un plateau de jeu � Splash �.

2. Les deux joueurs jouent a tour de role.Decidez quel joueur ira en premier (Joueur 1) et quel joueur ira en second (Joueur 2).

3. A chaque tour, le joueur doit selectionner un carre et doit ensuite supprimer ce carre et tousles carres situes au-dessus et a droite de ce dernier (voir l’exemple ci-dessous).

On ne peut pas choisir un carre qui a ete supprime (que l’on indique par un ) lors des tourssuivants.

4. Le joueur qui est oblige de supprimer le carre contenant la tache � Splash � perd la partie.

Vous pouvez jouer a ce jeu a plusieurs reprises en vous servant des plateaux de jeuci-dessous. A chaque partie, alternez le joueur qui commence la partie. Pour chacun des plateauxde jeu ci-dessous (Plateau de jeu 1 et Plateau de jeu 2), existe-t-il une strategie garantissant a l’undes joueurs (Joueur 1 ou Joueur 2) une victoire a chaque fois ?

Plateau de jeu 1 Plateau de jeu 2

Plus d’infos :

Consulte la page du CEMI a la maison lundi, le 27 avril, pour une strategie pour ce jeu. De plus, onvous encourage a partager vos idees en ligne dans des forums avec lesquels vous etes a l’aise.

Ce jeu est souvent appele Chomp.

1

Page 53: CEMI a la maison 9e et 10e ann ee - Lundi 23 mars 2020 ... · 19.Un nombre de Fibonacci. 20.Un multiple de 11. 22.Le plus petit nombre de la grille. 23.Le troisi eme c^ot e d’un

CEMC at Home

Grade 9/10 - Monday, April 20, 2020

Splat - Solution

Game Board 1

Player 1 has a winning strategy for this game board. Player 1 should choose the square one over andone up from the square with the Splat as shown in the diagram below.

From here, Player 1 will mimic whatever Player 2 does in the following way:

• If Player 2 chooses a square from the remaining column, then Player 1 will choose the corre-sponding square in the remaining row.

• If Player 2 chooses a square from the remaining row, then Player 1 will choose the correspondingsquare in the remaining column.

For example, if Player 2 chooses the square marked as P2 (in Diagram 1 below), then Player 1 shouldchose the square marked as P1. The resulting game board is shown in Diagram 2 below.

Diagram 1 Diagram 2

If Player 1 keeps mimicking Player 2 in this way, eventually the square with the Splat will be theonly square left after Player 1 makes a move. Then Player 2 must choose that square and lose thegame.

Note: This solution is valid for any game board with an equal number of rows and columns (exceptfor a 1 by 1 game board). To start, Player 1 should always choose the square one over and one upfrom the Splat. Then Player 1 mimics Player 2 as described above.

The strategy for Game Board 2 is on the next page.

Page 54: CEMI a la maison 9e et 10e ann ee - Lundi 23 mars 2020 ... · 19.Un nombre de Fibonacci. 20.Un multiple de 11. 22.Le plus petit nombre de la grille. 23.Le troisi eme c^ot e d’un

Game Board 2

It turns out that Player 1 has a winning strategy for this game board but it is not easy to find or todescribe. In one strategy, Player 1 can start with the following play.

From here, the strategy depends on what move Player 2 chooses to make. Player 1 will then have toreact to this move and there are many different ways this can unfold. We will not present the strategyhere but we encourage you to think about it and look into it further on your own. (Remember thatthis game is often called Chomp.)

It is actually possible to argue that Player 1 must have a winning strategy without actually findingone! An argument would involve explaining the following two facts:

1. Exactly one player (Player 1 or Player 2) must have a winning strategy for this game board.

2. It is not possible for Player 2 to have a winning strategy for this game board.

For point 1., it turns out that since there is no way for the game to end in a tie, we can argue thatat least one player must have a winning strategy (although this argument is not as easy to explaincarefully as you might think). By the definition of a winning strategy, it is not possible for bothplayers to have a winning strategy for the same game board.

To argue point 2. we can use what is sometimes called a strategy-stealing argument. The idea hereis as follows: If Player 2 had a winning strategy, then Player 1 could use that strategy before Player2 has a chance to use it. The name “strategy stealing argument” is slightly misleading. A winningstrategy must work regardless of the moves made by the other player and so cannot be “stolen” (atleast not without a mistake!). That Player 1 can “steal” the winning strategy simply means thatPlayer 1 must have had the winning strategy in the first place.

Let’s assume that Player 2 has a winning strategy. This means that no matter what first movePlayer 1 makes, Player 2 is guaranteed to win the game following this strategy.

In particular, this means that if Player 1 starts the game by selecting the top right hand square inthe grid, then Player 2 must have a winning strategy from this point. This means Player 2 has amove in response to Player 1 removing the top right hand square that leaves Player 1 with a gameboard from which they cannot possibly win.

No matter what the game board looks like after Player 2 responds, Player 1 could have made theirfirst move in a way that leaves this exact same game board! (Can you see why?) In this way, Player1 can “steal” the strategy by making their first move to leave Player 2 with this game board fromwhich they cannot win.

This argument may take a few times through to understand, but it achieves something quite remark-able. It explains that Player 1 has a winning strategy, but gives no hint whatsoever as to what thestrategy is. In fact, it doesn’t even tell us what the first move should be!

The above explanation that Player 2 cannot have a winning strategy involves a type of argument calleda proof by contradiction.

Page 55: CEMI a la maison 9e et 10e ann ee - Lundi 23 mars 2020 ... · 19.Un nombre de Fibonacci. 20.Un multiple de 11. 22.Le plus petit nombre de la grille. 23.Le troisi eme c^ot e d’un

CEMC.UWATERLOO.CA | le CENTRE d’ÉDUCATION en MATHÉMATIQUES et en INFORMATIQUE

Le CEMI a la maison

9e et 10e annee - le mardi 21 avril 2020Comptons

Considere la fourchette d’entiers de 10 a 30.

Si tu ecrivais tous ces entiers, tu ecrirais le chiffre 2 un total de douze fois ; soit une fois pour le chiffredes unites de 12, dix fois pour le chiffre des dizaines de chacun des entiers de 20 a 29 et une fois pourle chiffre des unites de 22.

Si on te demandait combien des entiers de 10 a 30 contiennent le chiffre 2, ta reponse serait onze etnon douze. Bien que le nombre 22 soit compose de deux 2, on ne compte ce nombre qu’une seule fois.

Resous les problemes suivants dans lesquels on compte des chiffres et des nombres dans une longueliste d’entiers.

Probleme 1 : Considere la fourchette d’entiers de 300 a 600.(a) Si tu ecrivais tous les entiers que comprend cette fourchette, combien de fois ecrirais-tu

le chiffre 4 ?(b) Combien des entiers compris dans cette fourchette contiennent le chiffre 4 ?

Probleme 2 : Si tu ecrivais tous les entiers de 300 a 600, quelle serait la somme de tous leschiffres que tu aurais ecrits ?

Discussion du probleme 2 : Pour repondre a la partie (a) ci-dessus, tu dois compter le nombre de foisque tu ecrirais le chiffre 4 si tu ecrivais tous les entiers de 300 a 600. Puisqu’il y a 301 entiers dansla fourchette d’entiers de 300 a 600, il ne serait pas tres efficace de tous les ecrire.

On rencontre le meme probleme dans le probleme 2. Une facon de resoudre ce probleme serait d’ecrireles 301 entiers de 300 a 600 puis d’additionner les 903 chiffres dont sont composes ces 301 entiers.Cependant, on risque d’y passer beaucoup de temps. Il y a des facons plus efficaces de resoudrece probleme. Vois-tu comment ton travail dans le probleme 1 pourrait t’aider a repondre a cettequestion ?

Afin de t’aider dans la resolution de ce probleme, considere la liste d’entiers de notre premier exemple,soit la liste d’entiers de 10 a 30 :

10, 11, 12, 13, 14, 15, 16, 17, 18, 19, 20, 21, 22, 23, 24, 25, 26, 27, 28, 29, 30

Que remarques-tu lorsque tu calcules la somme des 42 chiffres de cette liste d’entiers ? Il te seraitpeut-etre utile de considerer separement les 21 chiffres des unites et les 21 chiffres des dizaines.

Plus d’infos :

Consulte la page du CEMI a la maison mardi, le 28 avril, pour la solution a ce probleme.

1

Page 56: CEMI a la maison 9e et 10e ann ee - Lundi 23 mars 2020 ... · 19.Un nombre de Fibonacci. 20.Un multiple de 11. 22.Le plus petit nombre de la grille. 23.Le troisi eme c^ot e d’un

CEMC at Home

Grade 9/10 - Tuesday, April 21, 2020

Some Counting - Solution

Problem 1: Consider the integers from 300 to 600, inclusive.

(a) If you wrote down all of the integers from 300 to 600, inclusive, how many times wouldyou write the digit 4?

(b) How many of the integers from 300 to 600, inclusive, contain the digit 4?

Solution to Problem 1:

(a) There are a number of ways to approach this question, but we will use an organized count.

First, consider the hundreds digits of the 301 integers from 300 to 600. None of the integersfrom 300 to 399 have a 4 in the hundreds position. The 100 integers from 400 to 499 each havea 4 in the hundreds position. None of the integers from 500 to 600 have a 4 in the hundredsposition. So there are a total of 100 4s appearing in the hundreds position in the list of integersfrom 300 to 600.

Next, consider the tens digits. Only the integers from 340 to 349, 440 to 449, and 540 to 549have a 4 in the tens position. So there are 3 × 10 = 30 4s appearing in the tens position in thelist of integers from 300 to 600.

Finally, consider the units (ones) digits. In the integers from 300 to 399 there are 10 integerswith a 4 in the units position. These integers are as follows:

304, 314, 324, 334, 344, 354, 364, 374, 384, 394

Similarly, there are 10 integers from 400 to 499 with a 4 in the units position and 10 integersfrom 500 to 600 with a 4 in the units position. So there are 3 × 10 = 30 4s appearing in theunits position in the list of integers from 300 to 600.

Combining the counts for the three different positions, when writing the integers from 300 to600 the digit 4 will be written a total of 100 + 30 + 30 = 160 times.

(b) In our solution to part (a), we determined that there are 100 integers from 300 to 600 with a 4in the hundreds position, 30 integers with a 4 in the tens position, and 30 integers with a 4 inthe units position, but we note that 100 + 30 + 30 = 160 is not a correct count of the numberof integers that contain a 4. Some of these integers have more than one digit equal to 4 and sowere counted more than once in the total of 160.

We now need to determine how many integers were counted more than once, and exactly howmany times they were counted.

How many of the integers have a 4 in both the hundreds and the tens positions? These integerswill begin with 44, and thus are from 440 to 449, and there are 10 of them. These 10 integershave been counted at least twice.

How many of the integers have a 4 in both the hundreds and the units positions? These integerswill begin with a 4 and end with a 4. These integers have the form 4X4, where the X can beany of the ten digits, and there are 10 of them. These 10 integers have been counted at leasttwice.

Page 57: CEMI a la maison 9e et 10e ann ee - Lundi 23 mars 2020 ... · 19.Un nombre de Fibonacci. 20.Un multiple de 11. 22.Le plus petit nombre de la grille. 23.Le troisi eme c^ot e d’un

How many of the integers have a 4 in both the tens and the units positions? These integershave the form X44, where X can be 3, 4 or 5. Thus there are 3 of these integers. These 3integers have been counted at least twice.

There is only one integer with a 4 in all three positions, 444, and it was counted three timesbecause it belongs to each of the three groups we just examined. All other integers from thesegroups were counted exactly two times because they each belong to two of the three groups.

To count the number of integers that contain at least one digit 4 we do the following calculation.Start with the total of 160 and subtract the number of integers that were counted at least twice(due to having a digit 4 in at least two positions). This results in the following:

160 − 10 − 10 − 3 = 137

But there is one last thing to consider before we obtain the final answer. The integer 444 wascounted (or included) three times and then removed (or excluded) three times in our calculationabove. (Do you see why?) This means it is not included in the count of 137. Adding the integer444 back into our count we get a final answer of

160 − (10 + 10 + 3) + 1 = 138

Therefore, there are 138 integers between 300 and 600 that contain the digit 4.

Note: In this solution, the method of inclusion-exclusion has been demonstrated.

Problem 2: If you wrote down all of the integers from 300 to 600, inclusive, what is the sumof all of the digits that you would write?

Solution to Problem 2:

In the solution to this problem, it is again necessary to be organized in your approach. We give twopossible ways to calculate this sum. The second approach uses our work from Problem 1(a).

Approach 1:

First, consider the units digits of the integers from 300 to 399. In each set of ten consecutive integersin this range, each of the digits 0, 1, 2, 3, 4, 5, 6, 7, 8, 9 appears once (in the units position) and theirsum is 45. There are ten distinct sets of 10 consecutive integers in the range from 300 to 399, and sothe sum of the units digits in this range is 10 × 45 = 450. Similarly, in both the ranges from 400 to499 and 500 to 599, the sum of the units digits will be 450. Thus, from 300 to 599, the sum of theunits digits will be 3 × 450 = 1350.

Now consider the tens digits of the integers from 300 to 399. We note that each of the digits0, 1, 2, 3, 4, 5, 6, 7, 8, 9 appears 10 times in the tens position in this list of 100 integers. For example,the digit 0 appears as the tens digit in the integers from 300 to 309. The sum of these tens digits is10 × 45 = 450. As with the units digits, the same number of each digit occurs again in the ranges400 to 499 and 500 to 599. Thus, from 300 to 599, the sum of the tens digits will be 3× 450 = 1350.

In the range 300 to 599 there are 100 integers with a 3 in the hundreds position, 100 integers witha 4 in the hundreds position, and 100 integers with a 5 in the hundreds position. The sum of these300 digits is 100 × 3 + 100 × 4 + 100 × 5 = 100 × (3 + 4 + 5) = 100 × 12 = 1200.

Note that the only digits we have not yet included in our sum are the digits of the integer 600.Therefore, the sum of all digits used in writing down the integers from 300 to 600, inclusive, is

1350 + 1350 + 1200 + 6 + 0 + 0 = 3906

Page 58: CEMI a la maison 9e et 10e ann ee - Lundi 23 mars 2020 ... · 19.Un nombre de Fibonacci. 20.Un multiple de 11. 22.Le plus petit nombre de la grille. 23.Le troisi eme c^ot e d’un

Approach 2:

You may have noticed that you can use your work from Problem 1(a) to calculate the sum inProblem 2. We outline how to do this here.

Recall that when writing the integers from 300 to 600, inclusive, the digit 4 will be written a total of160 times. A similar argument can be used to show that the digits 3 and 5 will also appear a totalof 160 times. Convince yourself that the digits 3 and 5 will appear exactly as often as the digit 4 ineach of the three positions: hundreds, tens, and units.

The counts for the remaining digits are a bit different because of the particular range of integers.Notice that the digits 1, 2, 7, 8 and 9 do not appear as hundreds digits, and the digit 6 appears onlyonce as a hundreds digit (in 600).

Using the work done in Problem 1(a), we can see that the digit 9 will appear 30 + 30 = 60 times.This is because the digit 9 appears exactly as often as the digit 4 in the units position and in the tensposition, but does not appear at all in the hundreds position. (Remember that the digit 4 appears30 times in the tens position and 30 times in the units position.) Similar reasoning shows that thedigits 1, 2, 7, and 8 also appear 60 times each, and that the digit 6 appears 61 times. (Rememberthat the digit 6 appears once in the hundreds position.)

When we add up all of the digits, each digit 4 contributes 4 to the sum, and so the 160 4s contributea total of 160 × 4 = 640 to the sum of the digits. Dealing with the other digits in a similar way, wecan calculate the sum of all of the digits as follows:

60 × 1 + 60 × 2 + 160 × 3 + 160 × 4 + 160 × 5 + 61 × 6 + 60 × 7 + 60 × 8 + 60 × 9 = 3906

Page 59: CEMI a la maison 9e et 10e ann ee - Lundi 23 mars 2020 ... · 19.Un nombre de Fibonacci. 20.Un multiple de 11. 22.Le plus petit nombre de la grille. 23.Le troisi eme c^ot e d’un

CEMC.UWATERLOO.CA | le CENTRE d’ÉDUCATION en MATHÉMATIQUES et en INFORMATIQUE

Le CEMI a la maison

9e et 10e annee - le mercredi 22 avril 2020Tresor englouti

VideoVisionne la presentation du probleme d’optimisation connu sous le nom de Probleme du sac a dos :

https://youtu.be/qihjUx8Qakk (des sous-titres en francais sont disponibles)

Les deux scenarios de la presentation, ainsi que les questions, sont inclus a titre de reference.

Questions :

1. Quelle est la valeur maximale que peut contenir le sac a dos ?

2. Pour atteindre cette valeur maximale, quel sous-ensemble d’objets le sac devrait-il contenir ?

3. Quel a ete ton processus ?

4. Comment peux-tu t’assurer d’avoir obtenu la bonne solution ?

Tresor englouti (A) (avec l’application interactive : https://www.geogebra.org/m/hvbnqhzg)

Tresor englouti (B) (avec l’application interactive : https://www.geogebra.org/m/dnrgpjcd)

Plus d’infos :

Consulte la page du CEMI a la maison mercredi, le 29 avril, pour la solution a ce probleme.

Page 60: CEMI a la maison 9e et 10e ann ee - Lundi 23 mars 2020 ... · 19.Un nombre de Fibonacci. 20.Un multiple de 11. 22.Le plus petit nombre de la grille. 23.Le troisi eme c^ot e d’un

CEMC at Home

Grade 9/10 - Wednesday, April 22, 2020

Sunken Treasure - Solution

Recall the set up for the activity Sunken Treasure (B):

What is the maximum knapsack value you can achieve?

The maximum knapsack value you can achieve is 11 550 gold, although this value may have beendifficult for you to obtain.

If you tried to fill your knapsack by choosing items with the greatest rate of gold per gram, then youmost likely achieved a value of 11 500.

If you calculate each item’s rate of gold per gram, sort the items from largest to smallest rate, andthen place items in your bag in this same order as long as they fit in your knapsack, then you willend up with the following subset of the available items:

This algorithm, where you choose the best item at each step in the hope of getting the best outcomeoverall, is known as a greedy algorithm. Greedy algorithms do not look at the bigger picture and donot plan ahead.

Page 61: CEMI a la maison 9e et 10e ann ee - Lundi 23 mars 2020 ... · 19.Un nombre de Fibonacci. 20.Un multiple de 11. 22.Le plus petit nombre de la grille. 23.Le troisi eme c^ot e d’un

If we step back and look at the bigger picture, we can see that if we remove the coins and the operaglasses and instead take the teapot, we can increase the knapsack’s value by 50 gold as shown below.

It turns out that this solution is optimal, but how do we know for sure that this is the case? Wecould check the value attained by every possible subset of items, and make sure our value is at leastas high as all of the rest. This approach is often called a brute force algorithm.

How long would this approach take? When forming a subset, each item is either in the subset ornot. So there are 2 possibilities for each of the 15 available items: in or out. This means there are215 = 32 768 subsets we would have to check.

Of course, there are many subsets that we can eliminate immediately, or at least very quickly, becauseit is clear they will not produce an optimal result.

Checking all of these subsets is not feasible by hand, but suppose a computer can check 1 millionsubsets every second. In this case, it would take a computer less than 1 second to check all of thesubsets and find the optimal solution. The problem is, this approach does not scale well. If thenumber of available items is increased from 15 to 100, then there would be 2100 subsets to checkwhich would take a computer approximately 40 quadrillion years to consider.

Using brute force to find the optimal solution is often not feasible, but unfortunately the best knownalgorithms to solve this type of problem are also not always feasible. This explains why the knapsackproblem is a hard problem in general. For hard problems, it is often good enough to find a solutionwhich is close to the optimal solution.

Can you find a way to reason that the value given above is optimal without the help of a computer?If you have some programming experience, can you write a computer program that can check this foryou?

Page 62: CEMI a la maison 9e et 10e ann ee - Lundi 23 mars 2020 ... · 19.Un nombre de Fibonacci. 20.Un multiple de 11. 22.Le plus petit nombre de la grille. 23.Le troisi eme c^ot e d’un

CEMC.UWATERLOO.CA | le CENTRE d’ÉDUCATION en MATHÉMATIQUES et en INFORMATIQUE

Le CEMI a la maison presente Le probleme de la semaine

9e et 10e annee - le jeudi 23 avril 2020

Des points d’intersection

Deux droites differentes sont tracees de facon a ce que la premiere droite croise l’axe des y au pointP et la deuxieme droite croise l’axe des x au point Q. Le segment de droite PQ est perpendiculaireaux deux droites.

Si l’equation de la droite qui croise P est y = mx + k, determine l’ordonnee a l’origine de l’autredroite, en termes de m et k.

Suggestion : Si tu trouves le probleme general un peu difficile a commencer, essaye d’abord de resoudreun probleme ayant une equation precise pour la droite qui croise le point P , comme y = 4x + 3 etensuite tente de resoudre le probleme general.

Plus d’infos :

Consulte la page du CEMI a la maison jeudi, le 30 avril, pour la solution a ce probleme.Tu peux egalement t’inscrire au Probleme de la semaine en cliquant sur le lien ci-dessous et recevoirla solution ainsi qu’un nouveau probleme, par courriel jeudi, le 30 avril.

Cette ressource du CEMI a la maison correspond au Probleme de la semaine pour les 9e et 10eannees. Le Probleme de la semaine est une ressource gratuite. Chaque semaine, des problemesprovenant de divers domaines mathematiques sont publies en ligne et envoyes par courriel auxenseignants afin qu’ils les utilisent avec leurs etudiants. Les problemes sont disponibles pour lesetudiants de la 3e jusqu’a la 12e annee. Les solutions aux problemes sont envoyees une semaineapres, en meme temps que le nouveau Probleme de la semaine.

Pour plus d’informations et t’inscrire au Probleme de la semaine, rendez-vous sur :https://www.cemc.uwaterloo.ca/resources/potw-f.php

1

Page 63: CEMI a la maison 9e et 10e ann ee - Lundi 23 mars 2020 ... · 19.Un nombre de Fibonacci. 20.Un multiple de 11. 22.Le plus petit nombre de la grille. 23.Le troisi eme c^ot e d’un

Problem of the WeekProblem D and Solution

Crossing Points in General

Problem

Two distinct lines are drawn such that the first line passes through point P on the y-axis andthe second line passes through point Q on the x-axis. Line segment PQ is perpendicular toboth lines. If the line through P has equation y = mx+ k, then determine the y-intercept ofline through Q in terms of m and k.

SolutionFor ease of reference, we will call the first line l1 and the second line l2.

Let b represent the y-intercept of l2.

Since l1 has equation y = mx+ k, we know that the slope of l1 is m and they-intercept is k. Therefore, P is the point (0, k).

Since PQ is perpendicular to both lines, it follows that l1 is parallel to l2. Also,the slope of PQ is the negative reciprocal of the slope of l1. Therefore,slope(PQ) = − 1

m . Since k is the y-intercept of the perpendicular segment PQ

and the slope of PQ is − 1m , the equation of the line through PQ is y = − 1

mx+ k.

To find the x-intercept of y = − 1mx+ k, set y = 0 and solve for x. If y = 0, then

0 = − 1mx+ k and 1

mx = k. The result x = mk follows. Therefore, the x-interceptof y = − 1

mx+ k is mk and the coordinates of Q are (mk, 0).

We can now find the y-intercept of l2 since we know Q(mk, 0) is on l2 and theslope of l2 is m. Substituting into the slope-intercept form of the line,y = mx+ b, we obtain 0 = (m)(mk) + b which simplifies to b = −m2k.

Therefore, the y-intercept of l2, the line through Q, is −m2k.

For the student who solved the problem using y = 4x+ 3 as the equation of l1,you should have obtained the answer −48 for the y-intercept of l2, the linethrough Q. A full solution to this problem is provided on the next page.

Page 64: CEMI a la maison 9e et 10e ann ee - Lundi 23 mars 2020 ... · 19.Un nombre de Fibonacci. 20.Un multiple de 11. 22.Le plus petit nombre de la grille. 23.Le troisi eme c^ot e d’un

Let l1 represent the line y = 4x+ 3. Let l2 represent the second line, the linethrough Q.

From the equation of l1 we know that the slope is 4 and the y-intercept is 3.Therefore P is the point (0,3).

Since PQ ⊥ l1 and PQ ⊥ l2, it follows that l1 ‖ l2. Also, the slope of PQ is thenegative reciprocal of the slope of l1. Therefore, slope(PQ) = −1

4 . Since 3 is they-intercept of PQ and the slope of PQ is −1

4 , the equation of the line throughPQ is y = −1

4x+ 3.

The x-intercepts of the line through perpendicular PQ and the line l2 are thesame since both lines intersect at Q on the x-axis. To find this x-intercept, sety = 0 in y = −1

4x+ 3. Then 0 = −14x+ 3 and 1

4x = 3. The result, x = 12,follows. The x-intercept of the line through perpendicular PQ and the line l2 is12 and point Q is (12, 0).

We can now find equation of l2 since Q(12, 0) is on l2 and the slope of l2 is 4.Substituting x = 12, y = 0 and m = 4 into y = mx+ b, we obtain0 = (4)(12) + b which simplifies to b = −48. The equation of l2 is y = 4x− 48and the y-intercept is −48.

This is the same result we obtained from the general solution on the previouspage.

Page 65: CEMI a la maison 9e et 10e ann ee - Lundi 23 mars 2020 ... · 19.Un nombre de Fibonacci. 20.Un multiple de 11. 22.Le plus petit nombre de la grille. 23.Le troisi eme c^ot e d’un

CEMC.UWATERLOO.CA | le CENTRE d’ÉDUCATION en MATHÉMATIQUES et en INFORMATIQUE

Le CEMI a la maison

9e et 10e annee - le vendredi 24 avril 2020S’amuser en coloriant

Materiel :

— Morceaux de papier— Crayons de couleur ou marqueurs a colorier

Colorier des figures

Dans les activites suivantes, on coloriera des figures en utilisant differentes couleurs. Dans les activitesd’aujourd’hui, une figure aura une coloration dite valide si :

— Toutes les regions de la figure sont coloriees.— Tout couple de regions adjacentes de la figure (c’est-a-dire les regions partageant un bord) est

colorie de deux couleurs differentes.— Les regions qui ne se rencontrent qu’en un seul point peuvent etre coloriees de la meme couleur.

Considerons l’exemple dans la figure ci-contre. Puisqu’il y a six regionsdifferentes dans le cercle, un moyen facile de produire une colorationvalide de la figure est d’utiliser six couleurs differentes et de colorierchaque region en utilisant l’une de ces couleurs. Cependant, on peutcolorier la figure en utilisant moins de six couleurs tout en respectantles regles de coloration valide. Peux-tu determiner combien de couleurssont absolument necessaires pour colorier cette figure ?

Il s’avere qu’il existe des colorations valides de cette figure qui n’uti-lisent que trois couleurs differentes (aucune coloration valide ne peutcontenir moins de trois couleurs). Cela veut dire qu’on a besoin d’aumoins 3 couleurs pour colorier cette figure. On voit dans la figure ci-contre une telle coloration qui ne contient que trois couleurs. Peux-tuexpliquer pourquoi on ne peut pas colorier cette figure en n’utilisantque deux couleurs ?

Activite 1 : Trouve une coloration valide pour chacune des figures ci-dessous en utilisant le moinsde couleurs possible. En gardant en tete ce nombre minimal de couleurs que tu as identifie, peux-tuexpliquer pourquoi tu ne peux pas utiliser encore moins de couleurs pour colorier la figure ?

(a) (b)

1

Page 66: CEMI a la maison 9e et 10e ann ee - Lundi 23 mars 2020 ... · 19.Un nombre de Fibonacci. 20.Un multiple de 11. 22.Le plus petit nombre de la grille. 23.Le troisi eme c^ot e d’un

CEMC.UWATERLOO.CA | le CENTRE d’ÉDUCATION en MATHÉMATIQUES et en INFORMATIQUE

Modeliser des figures a l’aide de graphiques

A mesure que les figures deviennent plus complexes, il est de plus en plus difficile de determinerle nombre minimal de couleurs necessaires pour les colorier. Afin de faciliter cette tache, on peutconvertir les informations necessaires quant au coloriage de la figure initiale en une figure plus simpleappelee graphique. Autrement dit, on modelise la figure a l’aide d’un graphique. On colorie ensuitele graphique plutot que la figure. Par la suite, on applique la coloration du graphique a la figure.Examinons cette idee en utilisant le cercle a six regions de la page precedente.

Pour modeliser une figure a l’aide d’un graphique, suis les etapessuivantes :

i. Attribue un entier strictement positif distinct a chaque region de lafigure.

ii. Modelise la figure de i. a l’aide d’un graphique. Les graphiques sontcomposes de points (soit des sommets) et de segments de droites (soitdes aretes). Cree un point (un sommet) dans le graphique pour cha-cune des regions de la figure. On represente ces points par des cerclescontenant les entiers qui representent chacune des regions. Ensuite, onrelie uniquement les sommets representant des regions adjacentes pardes segments de droite (des aretes).

Par exemple, puisque les regions 1 et 2 sont adjacentes dans la figure,les sommets 1 et 2 sont relies par une arete dans le graphique. Deplus, puisque les regions 4 et 5 ne sont pas adjacentes dans la figure,les sommets 4 et 5 ne sont pas relies par une arete dans le graphique.

iii. Colorie tous les sommets du graphique de maniereque deux sommets adjacents (sommets relies parune arete) n’aient pas la meme couleur.

iv. Transfere les couleurs des sommets de ton gra-phique aux regions correspondantes de la figureoriginale. Ta figure aura donc une colorationvalide. (Vois-tu pourquoi ?)

Puisque l’objectif initial etait de colorier les figures en utilisant le moins de couleurs possible, l’objectiflors de la modelisation de ces problemes avec des graphiques est de colorier les graphiques en utilisantle moins de couleurs possible.

Par exemple, voici la methode qu’on a utilisee pour colorier le graphique ci-dessus : On commence enattribuant la couleur rose au sommet 1. Ensuite, on identifie tous les sommets qui sont adjacents ace dernier : 2, 4 et 6. Si possible, on colorie tous ces sommets de la meme deuxieme couleur (jaune).Si ce n’est pas possible, on colorie autant de sommets que possible de cette meme deuxieme couleur.Dans notre exemple, on colorie les sommets 2 et 4 en jaune mais non le sommet 6 car ce dernier estrelie au sommet 4. Il nous faut donc une troisieme couleur (bleu). Il s’avere qu’on peut colorier tousles sommets restants avec ces trois couleurs.

Par ailleurs, on pourrait aussi argumenter qu’on ne peut avoir moins de trois couleurs. Remarquequ’on n’a pas d’autre choix que d’utiliser trois couleurs differentes pour les sommets 2, 3 et 5. Il nousfaut deux couleurs differentes pour les sommets 2 et 3 et aucune de ces couleurs ne peut etre utiliseepour le sommet 5.

2

Page 67: CEMI a la maison 9e et 10e ann ee - Lundi 23 mars 2020 ... · 19.Un nombre de Fibonacci. 20.Un multiple de 11. 22.Le plus petit nombre de la grille. 23.Le troisi eme c^ot e d’un

CEMC.UWATERLOO.CA | le CENTRE d’ÉDUCATION en MATHÉMATIQUES et en INFORMATIQUE

Activite 2 :

(a) Considerons les deux figures ci-dessous. Suis les etapes suivantes pour chacune des figures :

— Modelise la figure a l’aide d’un graphique tel qu’indique dans i. et ii. de la page precedente.

— Colorie le graphique tel qu’indique dans iii. de la page precedente de maniere a utiliser lemoins de couleurs possible.

— Ayant determine le nombre minimal de couleurs a utiliser, explique pourquoi tu ne peuxpas utiliser encore moins de couleurs pour colorier le graphique.

Lorsque tu auras complete ces etapes, tu auras determine le nombre minimal de couleursnecessaires afin que chaque figure ait une coloration valide. Tu pourras donc colorier les fi-gures originales a l’aide de ton graphique !

(b) Peux-tu creer une figure a deux dimensions dont une coloration valide necessiterait au minimum5 couleurs ? Cela est-il possible ? Penses-y bien.

La modelisation dans la vie de tous les jours !

On peut convertir de nombreux problemes de la vie quotidienne en problemes de colorationde graphiques. Ces problemes paraissent souvent sous forme de ressources (couleurs) et deconflits (deux regions qui ne peuvent pas etre coloriees de la meme couleur). On te charge donct’attribuer les ressources de maniere optimale (en utilisant le moins de couleurs possible) touten t’assurant qu’il n’y ait aucun conflit. Ce type de probleme survient souvent lorsqu’on tentede creer des horaires ; plus l’horaire a creer est grand, plus le probleme est difficile !

Consulte les problemes Timetabling et Aircraft Scheduling des concours precedents du defiinformatique Beaver. Lis l’enonce du probleme et essaie de determiner comment tu pourraisrepresenter ce probleme sous la forme d’un graphique a colorier. Pose-toi les questions sui-vantes :

— Que devraient representer les sommets ?— Comment decider si une arete devrait relier un couple de sommets ?— Que representent les couleurs ?— Une coloration valide comprenant un minimum de couleurs represente-t-elle une solution

au probleme ?

Plus d’infos :

Consulte la page du CEMI a la maison vendredi, le 1 mai, pour la solution a ce probleme.

3

Page 68: CEMI a la maison 9e et 10e ann ee - Lundi 23 mars 2020 ... · 19.Un nombre de Fibonacci. 20.Un multiple de 11. 22.Le plus petit nombre de la grille. 23.Le troisi eme c^ot e d’un

CEMC at Home

Grade 9/10 - Friday, April 24, 2020

Colouring Fun - Solution

1(a) The colouring to the right shows one way this figurecan be coloured using three colours. Is there a validcolouring that uses fewer colours? Consider the regionin the bottom left corner that is coloured red. Thisregion has exactly two neighbours and these twoneighbours are also neighbours of each other. Thus,in a valid colouring, these three regions must all begiven different colours. That is, at least three coloursare required. Since at least three colours are requiredand we have found a colouring that uses exactly threecolours, the minimum number of colours required tocolour this figure is three.

1(b) The colouring to the right shows one way this figurecan be coloured using four colours. Is there a validcolouring that uses fewer colours? Consider thefollowing four regions: the circle coloured pink alongwith the green region to its left, the yellow region toits bottom-left, and the blue region below it. Noticethat every pair of regions in this group of four areneighbours. This means that all four of these regionsmust be given a different colour in a valid colouring.That is, at least four colours are required.Since at least four colours are required and we havefound a colouring that uses exactly four colours, theminimum number of colours required to colour thisfigure is four.

Graph colouring is the assignment of labels, in this case colours, to each vertex of a graph, G,such that no adjacent vertices (i.e. connected by an edge) share the same colour. The goal isto colour G with the minimum number of colours possible. This minimum number is known asthe chromatic number of G.

There are many applications of graph colouring in the real world. Here are a few examples:

• Scheduling: classes, exams, meetings, sports, flights

• Creating and solving Sudoku puzzles

• Internet bandwidth allocation

1

Page 69: CEMI a la maison 9e et 10e ann ee - Lundi 23 mars 2020 ... · 19.Un nombre de Fibonacci. 20.Un multiple de 11. 22.Le plus petit nombre de la grille. 23.Le troisi eme c^ot e d’un

2(a) We first work through the activity for the first figure.

i. Label the regions ii. Translate into a graph

iii. Colour the graph

The colouring to the left shows one way this graph canbe coloured using four colours. Is there a valid colouringthat uses fewer colours? Notice that the group of verticeslabelled 5, 10, 11, and 12 has the property that all pairsof vertices from this group are adjacent. Therefore,vertices 5, 10, 11, and 12 all require a different colour ina valid colouring, so at least four colours are required.Since at least four colours are required and we have founda colouring that uses exactly four colours, the minimumnumber of colours required to colour this graph is four.That is, the chromatic number of this graph is 4.

Finally, we use translate the colouring of the graph to a colouring of the figure:

2

Page 70: CEMI a la maison 9e et 10e ann ee - Lundi 23 mars 2020 ... · 19.Un nombre de Fibonacci. 20.Un multiple de 11. 22.Le plus petit nombre de la grille. 23.Le troisi eme c^ot e d’un

Next we work through the activity for the second figure.

i. Label the regions ii. Translate into a graph

iii. Colour the graph

The colouring above shows one way this graph can be coloured using four colours. Is there a validcolouring that uses fewer colours? You can spend a bit of time trying to find a group of fourvertices for which each pair of vertices is adjacent, but how do you proceed if you cannot finda group like this? (Note that vertices 1, 6, 7, and 8 satisfy this property, but we will proceed asthough we have not spotted these.)

Let’s try to colour this graph using only three colours: pink, yellow, and blue. Consider the vertexlabelled 4 and assign this vertex a colour, say pink. (Note that exactly what colour we choose isnot important here.) The vertex 4 is adjacent to the vertices labelled 2, 3, 5, and 12 and so thesevertices cannot be coloured pink. Looking at the connections between these four vertices, convinceyourself that since only two colours remain, vertices 2 and 5 must be coloured with the same colour,say blue, and vertices 3 and 12 must be coloured with the other colour, yellow. Now consider thevertex numbered 11, which is adjacent to vertex 5 (coloured blue) and vertex 12 (coloured yellow),and so must be coloured pink. Finally, consider the vertex numbered 1. This vertex is adjacentto vertex 2 (coloured blue), vertex 12 (coloured yellow), and vertex 11 (coloured pink). In orderto produce a valid colouring, vertex 1 must be assigned a colour, but that colour cannot be pink,yellow, or blue! This means we cannot possibly finish our colouring without adding a fourth colour.

Since at least four colours are required and we have found a colouring that uses exactly four colours,the minimum number of colours required to colour this graph is four. That is, the chromatic num-ber of this graph is 4.

3

Page 71: CEMI a la maison 9e et 10e ann ee - Lundi 23 mars 2020 ... · 19.Un nombre de Fibonacci. 20.Un multiple de 11. 22.Le plus petit nombre de la grille. 23.Le troisi eme c^ot e d’un

Finally, we use translate the colouring of the graph to a colouring of the figure:

2(b) It turns out that, as hard as you try, you will not be able to create a two-dimensional figure thatrequires five colours in order to achieve a valid colouring. This result follows from a famous theoremin graph theory known as the Four Colour Theorem. The Four Colour Theorem states that anytwo-dimensional map, like the figures we have been colouring in this activity, always has a validcolouring with four colours. That is, it can always be coloured with four or fewer colours.

The proof of the Four Colour Theorem is not trivial. In fact, this theorem was first conjectured in1852, but was not proven until the 1970s, and the proof required the aid of a computer!

4

Page 72: CEMI a la maison 9e et 10e ann ee - Lundi 23 mars 2020 ... · 19.Un nombre de Fibonacci. 20.Un multiple de 11. 22.Le plus petit nombre de la grille. 23.Le troisi eme c^ot e d’un

CEMC.UWATERLOO.CA | le CENTRE d’ÉDUCATION en MATHÉMATIQUES et en INFORMATIQUE

Le CEMI a la maison

9e et 10e annee - le lundi 27 avril 2020Piste de course

Materiel :

— Un a quatre joueurs— Une piste de course sur du papier quadrille

On vous fournit une piste de course sur l’avant-derniere page. On vous fournit egalement du papierquadrille vierge sur la derniere page afin que vous puissiez creer votre propre piste !

— Chaque joueur doit avoir un stylo ou un crayon de couleur differente.Afin de pouvoir jouer a ce jeu a plusieurs reprises, on vous suggere de placer la piste de course al’interieur d’un protege-feuille transparent et d’utiliser des marqueurs effacables a sec pour y jouer. Sivous n’avez pas un protege-feuille transparent, recouvrez votre piste avec du ruban adhesif transparentafin de creer une surface effacable.

Comment jouer :

1. Preparez votre piste de course.

2. Les joueurs jouent a tour de role. Decidez quel joueur ira en premier, quel joueur ira en deuxieme etainsi de suite.

3. Pour commencer le jeu, chaque joueur doit placer sa � voiture � a un endroit different sur la lignede depart. Le premier joueur placera sa voiture sur la ligne de depart en premier suivi du deuxiemejoueur qui placera a son tour sa voiture et ainsi de suite.Les lignes du papier quadrille se coupent toutes en des points que l’on appellera desormais des � pointsde grille �. On place sa � voiture � sur la ligne de depart en choisissant un point de grille sur cetteligne. Chaque joueur doit placer sa voiture sur un point de grille different. Vous pouvez placer votrevoiture sur un point de grille sur les bords de la piste.

4. A chaque tour, le joueur deplacera sa voiture selon les regles de deplacements permis dans le jeu.On deplace sa voiture en choisissant un nouveau point de grille sur la piste. Les regles des deplacementspermis sont decrites ci-dessous.

5. Le gagnant est le premier joueur a completer un tour de la piste, c’est-a-dire le premier joueur dont lavoiture franchit la ligne d’arrivee.

Les deplacements permis

Un deplacement peut s’effectuer que d’un point de grille a un autre point de grille. Dansla figure ci-contre, on voit que chaque point de grille a huit points de grilles avoisinants.Pour le premier deplacement, chaque joueur doit deplacer sa voiture de son point degrille sur la ligne de depart a l’un des huit points de grilles avoisinants.

Pour tous les deplacements ulterieurs, soit les joueurs deplacent leur voiture de la mememaniere que celle du deplacement precedent (c’est-a-dire un deplacement de la memedistance et dans la meme direction), soit ils deplacent leur voiture a l’un des huit pointsde grilles avoisinant cette position finale. Par exemple, soit la fleche AB dans la figure ci-contre le deplacement precedent d’un joueur. Pour son prochain deplacement, le joueurpeut deplacer sa voiture soit au point indique par le × en rouge, soit a l’un des huitpoints avoisinant le × en rouge (ces points avoisinants sont indiques par des × en bleu).Remarquez que le deplacement du point B au × en rouge est represente par une flechedont la longueur et la direction sont identiques a celles de la fleche representant ledeplacement du point A au point B.Une voiture ne peut pas etre deplacee vers un point de grille ou se trouve une autrevoiture.

Page 73: CEMI a la maison 9e et 10e ann ee - Lundi 23 mars 2020 ... · 19.Un nombre de Fibonacci. 20.Un multiple de 11. 22.Le plus petit nombre de la grille. 23.Le troisi eme c^ot e d’un

CEMC.UWATERLOO.CA | le CENTRE d’ÉDUCATION en MATHÉMATIQUES et en INFORMATIQUE

Voici un exemple d’un jeu de piste de course qui comprend deux joueurs.

Le Joueur 1 (J1) joue en premier.Le Joueur 2 (J2) gagne apres 6 deplacements.

Voici une explication des quatre premiers deplacements du Joueur 2 dans l’exemple de jeu ci-dessus.

Le premier deplacementJ2 peut deplacer sa voiture a l’un des huit points indiques parles × en bleu. J2 choisit de deplacer sa voiture d’un point degrille vers la droite. Remarquez que J2 pourrait reculer. Or,un tel choix ne serait pas a son avantage s’il espere completerun tour de la piste en aussi peu de deplacements que possible.(On ne peut gagner en reculant de la ligne de depart demaniere a atteindre la ligne d’arrivee sans avoir completeun tour de la piste !)

Le deuxieme deplacementPuisque J2 a deplace sa voiture d’un point vers la droitedans son deplacement precedent, J2 peut deplacer sa voituresoit de la meme maniere que celle du deplacement precedent(c’est-a-dire un deplacement de la meme distance et dans lameme direction ; que l’on indique par le × en rouge), soit ildeplace sa voiture a l’un des huit points avoisinant le × enrouge (que l’on indique par des × en bleu). J2 deplace savoiture de deux points de grille vers la droite.

Le troisieme deplacementPuisque J2 a deplace sa voiture de deux points vers la droitedans son deplacement precedent, J2 peut deplacer sa voituresoit de la meme maniere que celle du deplacement precedent(c’est-a-dire un deplacement de la meme distance et dans lameme direction ; que l’on indique par le × en rouge), soitil deplace sa voiture a l’un des huit points avoisinant le ×en rouge (que l’on indique par des × en bleu). J2 deplacesa voiture de trois points vers la droite et d’un point vers lebas.

Le quatrieme deplacementPuisque J2 a deplace sa voiture de trois points vers la droiteet d’un point vers le bas dans son deplacement precedent,J2 peut deplacer sa voiture soit de la meme maniere quecelle du deplacement precedent (c’est-a-dire un deplacementde la meme distance et dans la meme direction ; que l’onindique par le × en rouge), soit il peut deplacer sa voiturea quelques-uns des points avoisinant le × en rouge (que l’onindique par des × en bleu). J2 deplace sa voiture de quatrepoints vers la droite.

Que faire quant aux bords de la piste ?Lors du jeu, il peut arriver qu’un joueur n’ait pas d’autre choix que de deplacer savoiture de maniere qu’elle soit situee sur les bords de la piste ou meme a l’exterieur dela piste (comme dans la figure ci-contre). Dans ce cas, le joueur doit placer sa voiture surle point de grille le plus pres de l’endroit ou son deplacement coupe le bord de la piste(ce point est indique par le point rouge). A son prochain tour, le joueur doit deplacersa voiture a l’un des points situes a l’interieur de la piste et avoisinant le point rouge(on indique ces points par des points noirs).

Jouons !

Jouez a ce jeu autant de fois que vous le souhaitez en utilisant la piste de course sur la pagesuivante. A chaque partie, alternez le joueur qui commence la partie. Avez-vous reussi a developper unestrategie qui vous permettra d’eviter de heurter les bords de la piste de course ?

Page 74: CEMI a la maison 9e et 10e ann ee - Lundi 23 mars 2020 ... · 19.Un nombre de Fibonacci. 20.Un multiple de 11. 22.Le plus petit nombre de la grille. 23.Le troisi eme c^ot e d’un

Plus d’infos : Un vecteur est une quantite possedant une magnitude et une direction. Dans l’activite de lapiste de course, chaque deplacement peut etre represente par un vecteur. Pour en savoir plus sur les vecteurs,voir cette lecon des Cercles de mathematiques.

Exemple de piste de course

Page 75: CEMI a la maison 9e et 10e ann ee - Lundi 23 mars 2020 ... · 19.Un nombre de Fibonacci. 20.Un multiple de 11. 22.Le plus petit nombre de la grille. 23.Le troisi eme c^ot e d’un

Creez votre propre piste de course !

Vous pouvez utiliser votre propre papier quadrille ou le papier quadrille ci-dessous. Ajoutez des coins pointusa votre piste afin d’augmenter le niveau de difficulte du defi !

print-graph-paper.com

Page 76: CEMI a la maison 9e et 10e ann ee - Lundi 23 mars 2020 ... · 19.Un nombre de Fibonacci. 20.Un multiple de 11. 22.Le plus petit nombre de la grille. 23.Le troisi eme c^ot e d’un

CEMC.UWATERLOO.CA | le CENTRE d’ÉDUCATION en MATHÉMATIQUES et en INFORMATIQUE

Le CEMI a la maison

9e et 10e annee - le mardi 28 avril 2020Comptons encore

Dans certains domaines des mathematiques, on etudie la notion de permutation. Une permutationd’un ensemble d’objets est une disposition ordonnee de tous les objets de cet ensemble.

Par exemple, considerons les entiers 1, 2 et 3. On peut disposer ces trois entiers de six manieresdifferentes. Il existe donc six permutations de cet ensemble d’objets, soit :

(1, 2, 3) (1, 3, 2) (3, 1, 2) (3, 2, 1) (2, 1, 3) (2, 3, 1)

Dans les problemes ci-dessous, on travaillera avec des permutations d’entiers consecutifs. De plus, onexaminera un type particulier de permutation, soit les permutations VALROBSAR.

Une permutation est une permutation VALROBSAR si aucun entier dans la permutation adeux entiers avoisinants qui lui sont tous deux inferieurs.

Deux entiers dans la permutation sont � voisins � s’ils paraissent directement l’un a cote del’autre.

Dans l’exemple ci-dessus, les permutations (1, 3, 2) et (2, 3, 1) ne sont pas des permutationsVALROBSAR car dans chacune, l’entier 3 a un entier inferieur a sa droite ainsi qu’a sa gauche.C’est-a-dire que l’entier 3 a deux entiers avoisinants qui lui sont inferieurs.

Les quatre autres permutations indiquees ci-dessus sont des permutations VALROBSAR. Regardonspar exemple la permutation (3, 1, 2). Les entiers 3 et 2 n’ont chacun qu’un seul voisin et n’ontdonc pas deux voisins inferieurs a eux-memes tandis que l’entier 1 a deux voisins dont aucun ne luiest inferieur. Regardons la permutations (3, 2, 1) comme second exemple. Les entiers 3 et 1 n’ontchacun qu’un seul voisin et n’ont donc pas deux voisins inferieurs a eux-memes tandis que l’entier 2a deux voisins dont un seul lui est inferieur. A toi : verifie que les deux permutations restantes, soit(1, 2, 3) et (2, 1, 3), sont bel et bien des permutations VALROBSAR.

Problemes :

1. Dresse la liste de toutes les permutations des entiers 1, 2, 3 et 4.

2. Combien des permutations des entiers 1, 2, 3 et 4 sont des permutations VALROBSAR ?

3. Combien des permutations des entiers 1, 2, 3, 4 et 5 sont des permutations VALROBSAR ?

4. Combien des permutations des entiers 1, 2, 3, 4, 5 et 6 sont des permutations VALROBSAR ?

Extension : Vois-tu une regularite dans tes solutions aux problemes 1. a 4. ? Soit n un entierstrictement positif qui verifie n ≥ 2 et considerons les permutations des entiers 1, 2, 3, 4, . . . ,n. Quepeux-tu dire par rapport au nombre de permutations VALROBSAR de ces entiers ?

Plus d’infos :

Consulte la page du CEMI a la maison mardi, le 5 mai, pour les solutions a ces problemes.

Page 77: CEMI a la maison 9e et 10e ann ee - Lundi 23 mars 2020 ... · 19.Un nombre de Fibonacci. 20.Un multiple de 11. 22.Le plus petit nombre de la grille. 23.Le troisi eme c^ot e d’un

CEMC at Home

GRADE 9/10 - Tuesday, April 28, 2020

More Counting - Solution

A permutation will be called a VALROBSAR permutation if no integer in the permutationhas two neighbours that both are less than it.

Two integers in the permutation are neighbours if they appear directly beside each other.

Problems:

1. List all permutations of the integers 1, 2, 3, and 4.

2. How many of the permutations of the integers 1, 2, 3, and 4 are VALROBSAR permutations?

3. How many VALROBSAR permutations are there of the integers 1, 2, 3, 4, and 5?

4. How many VALROBSAR permutations are there of the integers 1, 2, 3, 4, 5, and 6?

Solutions:

1. There are 24 permutations of the four integers:

1 2 3 4 1 2 4 3 1 3 2 4 1 3 4 2 1 4 2 3 1 4 3 2

2 1 3 4 2 1 4 3 2 3 1 4 2 3 4 1 2 4 1 3 2 4 3 1

3 1 2 4 3 1 4 2 3 2 1 4 3 2 4 1 3 4 1 2 3 4 2 1

4 1 2 3 4 1 3 2 4 2 1 3 4 2 3 1 4 3 1 2 4 3 2 1

2. There are 8 VALROBSAR permutations. They are the red permutations in the table below:

1 2 3 4 1 2 4 3 1 3 2 4 1 3 4 2 1 4 2 3 1 4 3 2

2 1 3 4 2 1 4 3 2 3 1 4 2 3 4 1 2 4 1 3 2 4 3 1

3 1 2 4 3 1 4 2 3 2 1 4 3 2 4 1 3 4 1 2 3 4 2 1

4 1 2 3 4 1 3 2 4 2 1 3 4 2 3 1 4 3 1 2 4 3 2 1

Page 78: CEMI a la maison 9e et 10e ann ee - Lundi 23 mars 2020 ... · 19.Un nombre de Fibonacci. 20.Un multiple de 11. 22.Le plus petit nombre de la grille. 23.Le troisi eme c^ot e d’un

3. Looking at the solution to 2., you might have noticed that all of the VALROBSAR permutationshave the number 4 at one of the two ends. It will be helpful to think about why this mustbe true. In order to be a VALROBSAR permutation, every number must have at most oneneighbour that is smaller than it. Since 4 is the largest among 1, 2, 3, and 4, if a permutationhas 4 in one of the two middle positions, then 4 is guaranteed to have two neighbours that aresmaller than it. Hence, a permutation cannot be a VALROBSAR permutation unless 4 is atone of the ends. It is worth noting that there are permutations with a 4 at the end that fail tobe VALROBSAR permutations, for example, 4132.

We now focus our attention on the VALROBSAR permutations of 1, 2, 3, 4, and 5. By thesame reasoning as in the previous paragraph, a VALROBSAR permutation of these integersmust have 5 at one of the ends. The next key observation is that if we “remove” this 5 fromthe end, what remains will be a VALROBSAR permutation of 1, 2, 3, and 4. This is becauseremoving a number on the end of a permutation does not introduce any new neighbouringpairs, and so cannot cause a failure of the VALROBSAR condition.

This means all of the VALROBSAR permutations of 1, 2, 3, 4, and 5 take the form 5abcd orabcd5 where abcd is a VALROBSAR permutation of 1, 2, 3, and 4. On the other hand, if wetake a VALROBSAR permutation of 1, 2, 3, and 4 and place a 5 on either end, what results isa VALROBSAR permutation of 1, 2, 3, 4, and 5. To see this, suppose abcd is a VALROBSARpermutation of 1, 2, 3, and 4 and consider the permutation 5abcd. The neighbours of b, c, andd in 5abcd are the same as they are in the permutation abcd. Since we are assuming abcd is aVALROBSAR permutation, each of a, b, and c has at most one neighbour smaller than it inabcd, and hence, has at most one neighbour smaller than it in 5abcd. We know that a is equalto one of 1, 2, 3, and 4, so a < 5, which means a has at most one neighbour smaller than it in5abcd (namely, b could be smaller than a). The number 5 is at the end of the permutation, soit cannot possibly cause a failure of the VALROBSAR condition.

We are now able to quickly count the number of VALROBSAR permutations of 1, 2, 3, 4,and 5. Using the discussion above, we get all of these VALROBSAR permutations by takinga VALROBSAR permutation of 1, 2, 3, and 4 and placing a 5 on one of the two ends. Thereare 8 VALROBSAR permutations of 1, 2, 3, and 4, so this gives 2 × 8 = 16 VALROBSARpermutations of 1, 2, 3, 4, and 5. Furthermore, each of these 16 VALROBSAR permutationsmust be different. Can you see why?

The VALROBSAR permutations of 1, 2, 3, 4, and 5 are given below:

1 2 3 4 5 2 1 3 4 5 3 1 2 4 5 3 2 1 4 5

4 1 2 3 5 4 2 1 3 5 4 3 1 2 5 4 3 2 1 5

5 1 2 3 4 5 2 1 3 4 5 3 1 2 4 5 3 2 1 4

5 4 1 2 3 5 4 2 1 3 5 4 3 1 2 5 4 3 2 1

Note: There are more direct ways of counting these permutations without building on the per-mutations of 1, 2, 3, and 4. (An idea of this form will be discussed in the Extension on thelast page.) The method presented above doesn’t just give us an easy to count the “next order”of VALROBSAR permutations, but also gives an easy way to list them (based on the “previ-ous list”). When using permutations in mathematics, sometimes we are interested in only thecount, and sometimes we are interested in the actual list of permutations. It is often helpful tothink about building them in “stages” like we have done here.

Page 79: CEMI a la maison 9e et 10e ann ee - Lundi 23 mars 2020 ... · 19.Un nombre de Fibonacci. 20.Un multiple de 11. 22.Le plus petit nombre de la grille. 23.Le troisi eme c^ot e d’un

4. Similar to the argument in the previous solution, a VALROBSAR permutation of 1, 2, 3, 4,5, and 6 must have the 6 at one of the ends and what remains after removing the 6 mustbe a VALROBSAR permutation of 1, 2, 3, 4, and 5. Furthermore, we get a VALROBSARpermutation of 1, 2, 3, 4, 5, and 6 by taking any VALROBSAR permutation of 1, 2, 3, 4,and 5 and adding a 6 to either end of the permutation. For every choice of a VALROBSARpermutation of 1, 2, 3, 4, and 5 and choice of which side to add the 6, we get a VALROBSARpermutation of 1, 2, 3, 4, 5, and 6. There are 16 VALROBSAR permutations of 1, 2, 3, 4, and5, so this means there are 2 × 16 = 32 VALROBSAR permutations of 1, 2, 3, 4, 5, and 6.

Extension: Can you see a pattern forming based on your work in problems 1. to 4.? Suppose thatn is a positive integer satisfying n ≥ 2 and consider the permutations of the integers 1, 2, 3, 4, . . . ,n.What can you say about the number of VALROBSAR permutations of these integers?

Discussion:

To recap, we found the following counts of the VALROBSAR permutations:

• n = 4: 23 = 8 VALROBSAR permutations

• n = 5: 24 = 16 VALROBSAR permutations

• n = 6: 25 = 32 VALROBSAR permutations

You might guess from this pattern that the number of VALROBSAR permutations of 1, 2, . . . , n is2n−1, in general. In fact, the arguments in 3. and 4. actually showed why the number of VALROBSARpermutations seemed to double at each stage, and these arguments can be used to justify this formula.

Here is a more direct way to count the number of VALROBSAR permutations of 1, 2, . . . , n. Let’sbuild such a permutation by placing each integer in turn and keeping track of how many choices wehave at each stage. (This argument could have been used in 3. and 4. as well.)

First, consider n, the largest integer. No matter where you place it in the permutation it will belarger than its neighbours, and so it must have only one neighbour. The integer n must be placed atan end of the permutation, either first or last, which means you have 2 choices.

n . . . or . . . n

Once you place n, you have (n− 1) places left for the remaining integers 1, 2, . . . , n− 1. As with nabove, you have 2 choices for where to place the integer n − 1: beside n or at the other end of thepermutation.

For example, if we placed n as in the left image shown above, then the two leftmost images belowshow the choices for where to place n − 1. Can you see why n − 1 must be placed like this in orderto form a VALROBSAR permutation? In the other case above, the choices are shown below on theright.

n (n− 1) . . . or n . . . (n− 1) or (n− 1) . . . n or . . . (n− 1) n

After each placement of the largest remaining integer, you then have 2 choices for where to place thenext largest integer. This continues until you have placed the integer 2, and then the integer 1 mustgo in the only remaining place.

Thus, for each of the first n − 1 integers, n, (n − 1), (n − 2), (n − 3), . . . , 2, you have 2 choices ofwhere each is placed, and then the 1 goes in the last open place. Thus, the number of VALROBSARpermutations of the integers 1, 2, . . . , n is 2n−1.

Page 80: CEMI a la maison 9e et 10e ann ee - Lundi 23 mars 2020 ... · 19.Un nombre de Fibonacci. 20.Un multiple de 11. 22.Le plus petit nombre de la grille. 23.Le troisi eme c^ot e d’un

CEMC.UWATERLOO.CA | le CENTRE d’ÉDUCATION en MATHÉMATIQUES et en INFORMATIQUE

Le CEMI a la maison

9e et 10e annee - le mercredi 29 avril 2020Des tapis magiques

Dans un chateau magique, il y a des tapis magiques et un nombre illimite de figurines de chats et dechiens. Les tapis magiques ont les comportements suivants :

∗ Si deux chats vont sur un tapis magique vert, un seul en sort.

∗ Si un quelconque autre couple d’animaux va sur un tapis magique vert, un chien en sort.

∗ Si deux chiens vont sur un tapis magique bleu sarcelle, un seul en sort.

∗ Si un quelconque autre couple d’animaux va sur un tapis magique bleu sarcelle, un chat ensort.

∗ Si un chat va sur un tapis magique rose, un chien en sort. Si un chien va sur un tapis magiquerose, un chat en sort.

Questions :

Dans les figures des questions suivantes, une ligne reliant deux tapis signifie que l’animal qui sort dutapis gauche est le meme animal qui va ensuite sur le tapis droit.

1. Dans la figure ci-dessous, on voit la disposition des tapis magiques au rez-de-chaussee duchateau. Si des chats et des chiens vont sur les tapis comme indique, quel animal sortira dutapis situe le plus a droite ?

Page 81: CEMI a la maison 9e et 10e ann ee - Lundi 23 mars 2020 ... · 19.Un nombre de Fibonacci. 20.Un multiple de 11. 22.Le plus petit nombre de la grille. 23.Le troisi eme c^ot e d’un

CEMC.UWATERLOO.CA | le CENTRE d’ÉDUCATION en MATHÉMATIQUES et en INFORMATIQUE

2. Dans la figure ci-dessous, on voit la disposition des tapis magiques au premier etage du chateau.Si des chats et des chiens vont sur les tapis comme indique et qu’un chien sort du tapis situe leplus a droite, identifie la couleur du tapis magique inconnu.

3. Dans la figure ci-dessous, on voit la disposition des tapis magiques au deuxieme etage duchateau. Si un chat sort du tapis situe le plus a droite, quels sont les quatre animaux qui sontalles sur les tapis ?

4. Au troisieme etage du chateau, le proprietaire souhaite disposer des tapis magiques commedans la figure ci-dessous. Cependant, les tapis magiques bleu sarcelle sont tres chers ! Suggereune nouvelle disposition, en utilisant uniquement des tapis verts et roses, qui aura le memecomportement que la disposition que souhaite le proprietaire. Autrement dit, pour chaquecombinaison possible de trois animaux qui pourraient aller sur les tapis les plus a gauche,l’animal qui sortira du tapis le plus a droite sera le meme dans les deux dispositions.

Plus d’infos :

Consulte la page du CEMI a la maison mercredi, le 6 mai, pour les solutions a ces problemes.

Les tapis magiques vert, bleu sarcelle et rose ont, respectivement la meme forme et le meme compor-tement que les portes logiques ET, OU et NON. Consulte Escape Room du defi informatique Beaverde 2019 Beaver Computing Challenge pour un probleme similaire et pour plus d’informations sur lesportes logiques.

Page 82: CEMI a la maison 9e et 10e ann ee - Lundi 23 mars 2020 ... · 19.Un nombre de Fibonacci. 20.Un multiple de 11. 22.Le plus petit nombre de la grille. 23.Le troisi eme c^ot e d’un

CEMC at Home

Grade 9/10 - Wednesday, April 29, 2020

Magic Carpets - Solution

1. A dog will walk off the green carpet and a cat will walk off the pink carpet. Therefore, a dog anda cat will walk on the teal carpet, and so a cat will walk off the teal carpet (rightmost carpet).

2. It must be the case that a cat walks off the missing carpet since only a cat walking on therightmost (pink) carpet will cause a dog to walk off. What animals walk on the missing carpet?At the top-left, a cat walks off the pink carpet and so two cats walk on the teal carpet. Thismeans one cat walks off the teal carpet and approaches the missing carpet. At the bottom-left,a dog walks off the green carpet and approaches the missing carpet. So a cat and a dog walkon the missing carpet and a cat walks off. This can only happen if the missing carpet is teal.

3. From top to bottom the animals are cat, dog, dog, and dog.

One way to obtain this answer is to try all possible combinations of animals until you find onethat results in a cat walking off the rightmost carpet. How many combinations would you haveto try? Since there are 4 spots for animals, and each spot could be a cat or a dog, there are24 = 16 combinations. Testing each combination in turn may not be the best way to proceed.

An alternative way is to work backwards. Imagine that you have a video of the animals walkingacross the carpets, and you play the video in reverse. Here is a series of images that show thecat on the right moving backwards over the carpets.

The only way a cat can walkoff a pink carpet is if a dogwalks on.

The only way a dog can walkoff a teal carpet is if two dogswalk on.

The only way a dog can walkoff a pink carpet is if a catwalks on.

AND

The only way a dog can walkoff a teal carpet is if two dogswalk on.

1

Page 83: CEMI a la maison 9e et 10e ann ee - Lundi 23 mars 2020 ... · 19.Un nombre de Fibonacci. 20.Un multiple de 11. 22.Le plus petit nombre de la grille. 23.Le troisi eme c^ot e d’un

The only way a cat can walkoff a green carpet is if twocats walk on.

The only way a cat can walkoff a pink carpet is if a dogwalks on.

Therefore, this shows theonly option for the startinganimals.

4. The given arrangement with the teal carpet

is equivalent to the following arrangement that uses no teal carpets:

You may have found a different arrangement than this that also works.

2

Page 84: CEMI a la maison 9e et 10e ann ee - Lundi 23 mars 2020 ... · 19.Un nombre de Fibonacci. 20.Un multiple de 11. 22.Le plus petit nombre de la grille. 23.Le troisi eme c^ot e d’un

CEMC.UWATERLOO.CA | le CENTRE d’ÉDUCATION en MATHÉMATIQUES et en INFORMATIQUE

Le CEMI a la maison presente Le probleme de la semaine

9e et 10e annee - le jeudi 30 avril 2020

Une machine lumineuse

Une machine possede 2020 lumieres et 1 bouton. Chaque fois qu’on appuie sur le bouton, l’etat de 3lumieres est change. C’est-a-dire, si une lumiere est allumee, elle s’eteint et si une lumiere est eteinte,elle s’allume. Avant d’appuyer sur le bouton, l’utilisateur choisit les lumieres qui changeront d’etat.

Avant de commencer, toutes les lumieres sont eteintes. Quel est le plus petit nombre de fois que lebouton doit etre appuye afin d’allumer toutes les lumieres ?

Astuce : Commence en imaginant une machine avec moins de lumieres.

Plus d’infos :

Consulte la page du CEMI a la maison jeudi, le 7 mai, pour la solution a ce probleme.Tu peux egalement t’inscrire au Probleme de la semaine en cliquant sur le lien ci-dessous et recevoirla solution ainsi qu’un nouveau probleme, par courriel jeudi, le 7 mai.

Cette ressource du CEMI a la maison correspond au Probleme de la semaine pour les 9e et 10eannees. Le Probleme de la semaine est une ressource gratuite. Chaque semaine, des problemesprovenant de divers domaines mathematiques sont publies en ligne et envoyes par courriel auxenseignants afin qu’ils les utilisent avec leurs etudiants. Les problemes sont disponibles pour lesetudiants de la 3e jusqu’a la 12e annee. Les solutions aux problemes sont envoyees une semaineapres, en meme temps que le nouveau Probleme de la semaine.

Pour plus d’informations et t’inscrire au Probleme de la semaine, rendez-vous sur :https://www.cemc.uwaterloo.ca/resources/potw-f.php

1

Page 85: CEMI a la maison 9e et 10e ann ee - Lundi 23 mars 2020 ... · 19.Un nombre de Fibonacci. 20.Un multiple de 11. 22.Le plus petit nombre de la grille. 23.Le troisi eme c^ot e d’un

Problem of the WeekProblem D and SolutionA De-Light-Ful Machine

ProblemA machine has 2020 lights and 1 button. Each button press changes the state of exactly 3 ofthe lights. That means if the light is currently on, it turns off, and if the light is currently off,it turns on. Before each button press, the user selects which 3 lights will change their state. Tobegin with, all the lights on the machine are off. What is the fewest number of button pressesrequired in order for all the lights to be on?

Hint: Start by thinking about a machine with fewer lights.

SolutionTo turn on all the lights with the fewest number of button presses, we shouldturn on 3 lights with each button press, and not turn any lights off.

• The first button press would turn on 3 lights.

• The second button press would turn on 3 more lights, bringing the total to 6lights on.

• The third button press would turn on 3 more lights, so now there would be 9lights on.

• And so on . . .

Continuing in this way we can see that the total number of lights on wouldalways be a multiple of 3. However, since 2020 is not a multiple of 3, this tells usthat at least 1 button press must turn some lights off. Since we want to press thebutton the fewest number of times, that means we want the fewest number ofbutton presses to turn lights off.

Now suppose the button was pressed 671 times, and each time 3 lights turned on.Then there would be 671× 3 = 2013 lights on in total. Let’s look at theremaining 7 lights that are still off. We can draw a diagram to show all thepossible outcomes for the next button presses until all 7 lights are on.

Note that the order of the lights does not matter. We are interested in how manylights are on, not which particular lights are on. To simplify our diagram, at eachstep we have moved all of the lights that are on to the left.

Page 86: CEMI a la maison 9e et 10e ann ee - Lundi 23 mars 2020 ... · 19.Un nombre de Fibonacci. 20.Un multiple de 11. 22.Le plus petit nombre de la grille. 23.Le troisi eme c^ot e d’un

Note that if a button press reverses the press that was just made, we did notinclude this in our diagram, as this will not give us the fewest number of buttonpresses.

We can see in the diagram that the shortest sequence of steps to get all the lightson would be:

1. Turn 3 lights on.

2. Turn 1 light off and 2 lights on.

3. Turn 3 lights on.

This takes 3 steps, which means 3 button presses. If we add this to the 671button presses to get to this point, that tells us there are 671 + 3 = 674 buttonpresses in total. We note that only 1 of these 674 button presses turns lights off.Since we know that at least 1 button press must turn some lights off, that tells uswe cannot turn all the lights on using fewer button presses.

Therefore, 674 button presses are required to turn on all the lights.

Page 87: CEMI a la maison 9e et 10e ann ee - Lundi 23 mars 2020 ... · 19.Un nombre de Fibonacci. 20.Un multiple de 11. 22.Le plus petit nombre de la grille. 23.Le troisi eme c^ot e d’un

CEMC.UWATERLOO.CA | le CENTRE d’ÉDUCATION en MATHÉMATIQUES et en INFORMATIQUE

Le CEMI a la maison

9e et 10e annee - le vendredi 1er mai 2020Fleurs, lettres et droites

Instructions : On decrit onze droites a la page suivante a l’aide d’equations ou d’autres renseigne-ments. A l’aide d’une regle, trace soigneusement ces droites sur la grille ci-dessous. Chaque droitedevrait passer par une seule fleur et une seule lettre. Associe chaque fleur a la lettre situee surla meme droite pour repondre a l’enigme ci-dessous. Sers-toi du tableau a la page suivante pourorganiser ton travail.

Enigme : Meme si l’etymologie de mon nom rappelle la basse-cour, jesuis haut en couleur au printemps dans les champs, et en novembre

sur tous les coeurs. Que suis-je?

Reponse :

!

1

Page 88: CEMI a la maison 9e et 10e ann ee - Lundi 23 mars 2020 ... · 19.Un nombre de Fibonacci. 20.Un multiple de 11. 22.Le plus petit nombre de la grille. 23.Le troisi eme c^ot e d’un

CEMC.UWATERLOO.CA | le CENTRE d’ÉDUCATION en MATHÉMATIQUES et en INFORMATIQUE

Droites :

y = 5x y = 19x− 10 x + 2y − 16 = 0

y + 15 = −13(x− 15) y = 17

2x− 5y − 10 = 0

Une droite ayant pour pente−2 et pour ordonnee a

l’origine 10

Une droite qui passepar les points

(−15, 40) et (−5,−20)

Une droite ayant pour pente−1 et qui passe par

le point (9,−8)

y + 4 = 3(x− 1)Une droite verticale qui passe

par le point (−13, 0)

Il est possible que tu doives recrire une/des equation(s) sous une autre forme ou que tu doives effectuerdes calculs supplementaires a l’aide des renseignements afin de representer les droites plus facilement.

Sers-toi du tableau suivant pour organiser ton travail.

Fleur

Lettre

Plus d’infos :

Consulte la page du CEMI a la maison vendredi, le 8 mai, pour la solution a ce probleme.

Pour plus de pratique avec la representation graphique des equations lineaires, consulte cette lecondans le didactitiel du CEMI. D’autres lecons te sont aussi accessibles dans l’unite des relationslineaires.

2

Page 89: CEMI a la maison 9e et 10e ann ee - Lundi 23 mars 2020 ... · 19.Un nombre de Fibonacci. 20.Un multiple de 11. 22.Le plus petit nombre de la grille. 23.Le troisi eme c^ot e d’un

CEMC.UWATERLOO.CA | le CENTRE d’ÉDUCATION en MATHÉMATIQUES et en INFORMATIQUE

Le CEMI a la maison

9e et 10e annee - le vendredi 1er mai 2020Fleurs, lettres et droites - Solution

On ecrit l’equation de chacune des droites sous la forme y = mx + b :

1. y = 5x 2. y = 19x− 10 3.

x + 2y − 16 = 0y = −1

2x + 8

4.y + 15 = −1

3(x− 15)

y = −13x− 10

5. y = 172

6.x− 5y − 10 = 0

y = 15x− 2

7.

Une droite ayantpour pente −2

et pour ordonneea l’origine 10y = −2x + 10

8.

Une droite qui passepar les points

(−15, 40) et (−5,−20)y = −6x− 50

9.

Une droite ayantpour pente −1et qui passe parle point (9,−8)y = −x + 1

10.y + 4 = 3(x− 1)

y = 3x− 7 11.

Une droite verticalequi passe par

le point (−13, 0)x = −13

Apres avoir trace la droite de chacune des equations lineaires (voir le graphique a la page suivante),on voit que chaque droite passe par une seule fleur et une seule lettre. On remplit le tableau suivanta l’aide du graphique:

Fleur

Lettre T O J E S I U C N Q L

Equation 5. 6. 3. 4. 10. 7. 11. 9. 1. 8. 2.

A l’aide du tableau ci-dessus, on peut repondre a l’enigme � Mon premier est un oiseau de basse-cour.On dort sur mon deuxieme. Mon troisieme est le debut du chant de mon premier. Mon tout est unefleur rouge des champs. Que suis-je? �

J E S U I S U N C O Q U E L I C O T !

1

Page 90: CEMI a la maison 9e et 10e ann ee - Lundi 23 mars 2020 ... · 19.Un nombre de Fibonacci. 20.Un multiple de 11. 22.Le plus petit nombre de la grille. 23.Le troisi eme c^ot e d’un

CEMC.UWATERLOO.CA | le CENTRE d’ÉDUCATION en MATHÉMATIQUES et en INFORMATIQUE

Clique sur le graphique pour l’explorer!

2

Page 91: CEMI a la maison 9e et 10e ann ee - Lundi 23 mars 2020 ... · 19.Un nombre de Fibonacci. 20.Un multiple de 11. 22.Le plus petit nombre de la grille. 23.Le troisi eme c^ot e d’un

CEMC.UWATERLOO.CA | le CENTRE d’ÉDUCATION en MATHÉMATIQUES et en INFORMATIQUE

Le CEMI a la maison

9e et 10e annee - le lundi 4 mai 2020Concours - jour 1

La ressource d’aujourd’hui presente deux questions des concours de mathematiques 2020 du CEMI.

Concours Euclide 2020, nº2(a)

Le nombre impair m est un entier strictement positif compose de trois chiffres distincts. Sachant quele chiffre des centaines est egal au produit des chiffres des unites et des dizaines, quel est l’entier m ?

Concours de mathematiques canadien par equipe 2020, probleme d’equipe nº6

Tous les vendredis, le prix du billet d’entree au musee est de 9 $. Un samedi, le musee a accueilli200 visiteurs, soit deux fois plus que la veille. Le montant total des recettes provenant de la ventedes billets ce samedi-la etait de 4

3plus eleve que la veille. Le prix des billets le samedi est de k $.

Determinez la valeur de k.

Plus d’infos :

Consulte la page du CEMI a la maison lundi, le 11 mai, pour les solutions aux problemes de ConcoursJour 1.

Page 92: CEMI a la maison 9e et 10e ann ee - Lundi 23 mars 2020 ... · 19.Un nombre de Fibonacci. 20.Un multiple de 11. 22.Le plus petit nombre de la grille. 23.Le troisi eme c^ot e d’un

CEMC.UWATERLOO.CA | le CENTRE d’ÉDUCATION en MATHÉMATIQUES et en INFORMATIQUE

Le CEMI a la maison

9e et 10e annee - le lundi 4 mai 2020Concours, jour 1 - solutions

Voici les solutions aux deux problemes de concours. La solution au premier probleme est accompagneed’une video.

Concours Euclide 2020, nº2(a)

Le nombre impair m est un entier strictement positif compose de trois chiffres distincts. Sachant quele chiffre des centaines est egal au produit des chiffres des unites et des dizaines, quel est l’entier m ?

Solution :

Supposons que a, b et c sont respectivement les chiffres des centaines, dizaines et des unites de m.D’apres le probleme : a, b et c sont trois chiffres distincts, a, b et c sont chacun inferieurs a 10, a = bcet c est impair (puisque m est impair).L’entier m = 623 remplit toutes ces conditions. Etant donne qu’il n’existe qu’un tel nombre, 623 estdonc la seule reponse.Pourquoi est-ce la seule valeur possible de m ?On remarque qu’on ne peut avoir b = 1 ou c = 1, sinon a = c ou a = b.Donc, b ≥ 2 et c ≥ 2.Puisque c ≥ 2 et que c est impair, alors c peut etre egal a 3, a 5, a 7 ou a 9.Puisque b ≥ 2 et a = bc, alors a serait superieur a 10 (ce qui est impossible) si c est egal a 5, a 7 oua 9.Donc, c = 3.Puisque b ≥ 2 et b 6= c, alors b = 2 ou b ≥ 4.Si b ≥ 4 et c = 3, alors a > 10, ce qui est impossible.On doit donc avoir c = 3 et b = 2, d’ou a = 6.

Video

Clique le lien suivant pour une discussion sur cette solution : https://youtu.be/dJ6d0ILAGwE

Concours de mathematiques canadien par equipe 2020, probleme d’equipe nº6

Tous les vendredis, le prix du billet d’entree au musee est de 9 $. Un samedi, le musee a accueilli200 visiteurs, soit deux fois plus que la veille. Le montant total des recettes provenant de la ventedes billets ce samedi-la etait de 4

3plus eleve que la veille. Le prix des billets le samedi est de k $.

Determinez la valeur de k.

Solution :Il y avait 200 visiteurs le samedi, alors il y en avait 100 la veille. Puisque les billets coutent 9 $ lesvendredis, le montant total collecte le vendredi s’elevait a 900 $. Par consequent, la somme d’argentcollectee lors de la vente des billets le samedi etait de 4

3(900 $) = 1200 $. Comme il y avait 200

visiteurs le samedi, le prix des billets pour ce samedi etait de 1200 $200

= 6 $.

En fait, tu n’as pas besoin de connaıtre le nombre de visiteurs du musee le samedi pour resoudre ceprobleme. Si ce nombre (200) change, mais toutes les autres conditions restent les memes, alors lareponse sera toujours k = 6. Pouvez-vous percevoir pourquoi ?

Page 93: CEMI a la maison 9e et 10e ann ee - Lundi 23 mars 2020 ... · 19.Un nombre de Fibonacci. 20.Un multiple de 11. 22.Le plus petit nombre de la grille. 23.Le troisi eme c^ot e d’un

CEMC.UWATERLOO.CA | le CENTRE d’ÉDUCATION en MATHÉMATIQUES et en INFORMATIQUE

Le CEMI a la maison

9e et 10e annee - le mardi 5 mai 2020Les regularites dans l’arithmetique

La recherche de regularite est une strategie que tu peux utiliser pour resoudre les problemes de cetteactivite.

Probleme 1 : L’assistant d’un sorcier est paye d’une maniere inhabituelle. A la fin de sapremiere semaine, l’assistant ne recoit qu’un seul dollar. A la fin de chaque semaine apres lapremiere, l’assistant recoit le montant qu’il a recu la semaine precedente plus deux dollars pourchaque semaine travaillee depuis qu’il a ete embauche. Quel montant l’assistant recevra-t-il ala fin de sa cinquante-deuxieme semaine de travail ?

Pour commencer, calcule le montant que l’assistant recevra a la fin de l’une des semaines anterieures(par exemple la 9e ou la 10e semaine) tout en recherchant une regularite dans ton travail.

Probleme 2 : Suppose que l’entier N correspond a la valeur de la somme ci-dessous (contenant52 termes) :

1 + 11 + 101 + 1001 + 10001 + · · · +

50 zeros

1︷ ︸︸ ︷000 . . . 0001

Lorsqu’on calcule N et qu’on l’exprime sous la forme d’un seul entier, quelle est la somme deses chiffres ?

Pour commencer, considere la forme des termes de la somme.

Probleme 3 : On cree une suite de chiffres en utilisant uniquement les chiffres 1, 2, 3, 4 et 5.Voici le debut de la suite :

1, 2, 2, 3, 3, 3, 4, 4, 4, 4, 5, 5, 5, 5, 5, 1, 1, 1, 1, 1, 1, 2, 2, 2, 2, 2, 2, 2, . . .

La suite commence par un 1, suivi de deux 2, puis de trois 3, puis de quatre 4, puis de cinq 5,puis de six 1, puis de sept 2, et ainsi de suite. Quel est le 1000e terme de la suite ?

Certaines regularites sont difficiles a decrire en peu de mots. Il est probable que la description ci-dessus soit suffisante afin de te communiquer la maniere dont la suite est definie. Or, cette descriptionne definit pas les termes restants de la suite de maniere precise. Peux-tu definir la suite d’une maniereplus formelle ?

Tu peux arriver aux solutions a ces problemes si tu decouvres les bonnes regularites. Reflechis a lafacon dont tu pourrais justifier chacune de ces regularites.

Plus d’infos :

Consulte la page du CEMI a la maison jeudi, le 12 mai, pour les solutions a ces problemes.

Page 94: CEMI a la maison 9e et 10e ann ee - Lundi 23 mars 2020 ... · 19.Un nombre de Fibonacci. 20.Un multiple de 11. 22.Le plus petit nombre de la grille. 23.Le troisi eme c^ot e d’un

CEMC at Home

Grade 9/10 - Tuesday, May 5, 2020

Patterns in Arithmetic - Solution

Problem 1: A Wizard’s assistant is paid in an unusual way. The assistant’s paycheque for the firstweek is one dollar. At the end of each week after the first week, the assistant is paid the amount ofmoney earned the previous week plus two dollars for every week worked so far. What is the assistant’spaycheque, in dollars, for the fifty-second week?

Solution:

Looking at the first few weeks’ salaries we have:

Week Pay ($)1 12 1 + 2(2) = 5

5 + 2(3) = 1111 + 2(4) = 19

34...

...

The above table may not provide enough information, so let’s expand things a bit in another table:

Week Pay ($)1 12 1 + 2(2) = 53 1 + 2(2) + 2(3) = 114 1 + 2(2) + 2(3) + 2(4) = 19...

...

At this point, it may be easier to see a pattern beginning to emerge. We might guess that the next rowin the table will have 1+2(2)+2(3)+2(4)+2(5) = 19+10 = 29 in the right column. Indeed, the payin the 5th week is 2×5 dollars more than it was on the 4th week so will be 1+2(2)+2(3)+2(4)+2(5).This pattern will continue and so we can deduce a formula for calculating any week’s pay directly.

In particular, the pay for the 52nd week can be calculated as follows:

1 + 2(2) + 2(3) + · · ·+ 2(51) + 2(52) = 1 + 2(2 + 3 + 4 + · · ·+ 51 + 52)

We are left with the problem of computing the sum

S = 2 + 3 + 4 + · · ·+ 51 + 52

where S represents the sum of the integers from 2 to 52. This can be done in a number of ways.

If you know the formula for the sum of the integers from 1 to n (it is said that Gauss found thisformula on his own as a child) then you could use it here. Alternatively, you could notice that thenumbers in the sum form an arithmetic sequence and use a formula for summing such a list ofnumbers.

Page 95: CEMI a la maison 9e et 10e ann ee - Lundi 23 mars 2020 ... · 19.Un nombre de Fibonacci. 20.Un multiple de 11. 22.Le plus petit nombre de la grille. 23.Le troisi eme c^ot e d’un

We will calculate S as follows: First write out the sum S twice, with the second sum written withthe terms in the opposite order.

S = 2 + 3 + 4 + · · ·+ 50 + 51 + 52+S = 52 + 51 + 50 + · · ·+ 4 + 3 + 22S = 54 + 54 + 54 + · · ·+ 54 + 54 + 54

Adding the terms on the left side gives 2S. Adding the terms on the right side gives

54 + 54 + 54 + · · ·+ 54 + 54 + 54,

the sum of 51 copies of 54. Therefore,

2S = 54 + 54 + 54 + · · ·+ 54 + 54 + 54 = 51(54) = 2754

This means the assistant’s pay for the 52nd week is

1 + 2(2 + 3 + 4 + · · ·+ 51 + 52) = 1 + 2S = 1 + 2754 = 2755

dollars.

Problem 2: Suppose that the integer N is the value of the following sum (with 52 terms):

1 + 11 + 101 + 1001 + 10001 + · · ·+50 zeroes

1︷ ︸︸ ︷000 . . . 0001

When N is calculated and written as a single integer, what is the sum of its digits?

Solution:

There are 52 numbers to be added together, and all of them have a 1 in the units position. One wayto approach this problem is to first compute N − 52 by subtracting 1 from each term. That is,

N − 52 = (1− 1) + (11− 1) + (101− 1) + (1001− 1) + · · ·+ (1

50 zeroes︷ ︸︸ ︷000 . . . 0001− 1)

= 0 + 10 + 100 + 1000 + · · ·+ 1

51 zeroes︷ ︸︸ ︷000 . . . 000

=

51 ones︷ ︸︸ ︷111 . . . 1110

We can now simply add 52 to both sides to get

N =

50 ones︷ ︸︸ ︷111 . . . 11162

Therefore, the sum of the digits of N is 50× 1 + 6 + 2 = 58.

Page 96: CEMI a la maison 9e et 10e ann ee - Lundi 23 mars 2020 ... · 19.Un nombre de Fibonacci. 20.Un multiple de 11. 22.Le plus petit nombre de la grille. 23.Le troisi eme c^ot e d’un

Problem 3: Using only the digits 1, 2, 3, 4, and 5, a sequence is created. The beginning of thesequence in shown below.

1, 2, 2, 3, 3, 3, 4, 4, 4, 4, 5, 5, 5, 5, 5, 1, 1, 1, 1, 1, 1, 2, 2, 2, 2, 2, 2, 2, . . .

The sequence starts with one 1, followed by two 2s, then three 3s, four 4s, five 5s, six 1s, seven 2s,and so on. What is the 1000th term in the sequence?

Solution:

Let’s think of the sequence as being in “blocks”. That is, the first block consists of 1s, the secondblock consists of 2s, the third block consists of 3s, and so on.To answer the question, we first need to determine in which block of digits the 1000th term lies.Note that each block consists of one more term than the previous block and that the first blockconsists of one term. Convince yourself that after you have written down the first n blocks in thesequence, you will have written down exactly

1 + 2 + 3 + · · ·+ (n− 1) + n

terms in total. How many blocks must be written down before you reach the block containing the1000th term? To answer this, we need to find when the sum above reaches 1000.

In the solution to Problem 1, we mentioned, but did not use, the formula for the sum of the positiveintegers from 1 to n. We shall make use of it here. The formula is:

1 + 2 + 3 + · · ·+ (n− 1) + n =n(n + 1)

2

What is the smallest positive integer n satisfyingn(n + 1)

2≥ 1000? Equivalently, what is the smallest

positive integer n satisfying n(n + 1) ≥ 2000? We could test values of n starting at n = 1 until wefind such an integer n, but this might take quite a while. We also “know” that n cannot be too small.For example, your intuition probably tells you that n is at least 10, or maybe even at least 20, soyou might not start at n = 1 and instead start at some larger integer.

Here is a way to choose a “good” starting integer. Note that for any positive integer n, (n + 1)2 islarger than n(n+1), so the n we seek must satisfy (n+1)2 ≥ 2000. Using a calculator, you can checkthat

√2000 ≈ 44.7. Since n + 1 is an integer, this means n + 1 ≥ 45, so n ≥ 44. Thus, we need not

check any values of n that are less than 44. With n = 44, we have n(n + 1) = 44(45) = 1980 whichis not ≥ 2000. With n = 45, we have n(n + 1) = 45(46) = 2070, which is ≥ 2000. This means thesmallest positive integer n with n(n + 1) ≥ 2000 is n = 45, and we only had to check two values ofn to find it!

We can now see that the first 44 blocks (in total) contain44(45)

2= 990 terms, and the first 45 blocks

(in total) contain45(46)

2= 1035 terms. This means the 1000th term occurs in the 45th block.

To determine what the 1000th term will be, we need to determine what digit makes up the 45th block.Note that the 1st block, 6th block, 11th block, and so on, consist of 1s.The 2nd block, 7th block, 12th block, and so on, consist of 2s.The 3rd block, 8th block, 13th block, and so on, consist of 3s.The 4th block, 9th block, 14th block, and so on, consist of 4s.The 5th block, 10th block, 15th block, and so on, consist of 5s.

In particular, when n is a multiple of 5, the nth block consists of 5s. Since the 1000th term is in the45th block, the 1000th term will be the digit 5.

Page 97: CEMI a la maison 9e et 10e ann ee - Lundi 23 mars 2020 ... · 19.Un nombre de Fibonacci. 20.Un multiple de 11. 22.Le plus petit nombre de la grille. 23.Le troisi eme c^ot e d’un

CEMC.UWATERLOO.CA | le CENTRE d’ÉDUCATION en MATHÉMATIQUES et en INFORMATIQUE

Le CEMI a la maison

9e et 10e annee - le mercredi 6 mai 2020Les coquillages de Sheldon

Sheldon marche le long de la plage en ramassant des coquillages. Les coquillages sont disperses surla plage dans differentes zones comme dans la figure ci-dessous.

En ramassant les coquillages, Sheldon emprunte un chemin qui commence dans la zone � A � etqui se termine dans la zone � Y �. Apres avoir ramasse tous les coquillages dans une zone, Sheldonse deplace a la zone suivante en allant soit vers le haut soit vers la droite. Sheldon ne se deplacerajamais vers le bas ou vers la gauche.

Questions :

1. En commencant dans la zone A, Sheldon peut atteindre la zone situee a 2 deplacements versla droite et a 1 deplacement vers le haut (la zone qui contient deux coquillages bleus) enempruntant trois chemins differents. Quel est le plus grand nombre de coquillages que Sheldonpourrait ramasser en allant de la zone A a cette zone (excluant les deux coquillages bleus) ?

2. Lors d’un voyage de A a Y, Sheldon s’arrete en chemin et remarque qu’il a ramasse 8 coquillages(y compris ceux dans la zone dans laquelle il s’est arrete). Quelles sont toutes les zones possiblesdans lesquelles Sheldon aurait pu s’arreter ?

3. Considere tous les chemins possibles que Sheldon pourrait emprunter en allant de A a Y. Quelest le plus grand nombre de coquillages qu’il pourrait ramasser sur son chemin en allant de Aa Y ?

Plus d’infos :

Consulte la page du CEMI a la maison mercredi, le 13 mai, pour les solutions a ces problemes.

Si tu as apprecie ce probleme, consulte le probleme Coins and Monsters du defi informatique Bea-ver de 2019. Quoique ce dernier soit semblable au probleme d’aujourd’hui, il contient un elementsupplementaire !

Page 98: CEMI a la maison 9e et 10e ann ee - Lundi 23 mars 2020 ... · 19.Un nombre de Fibonacci. 20.Un multiple de 11. 22.Le plus petit nombre de la grille. 23.Le troisi eme c^ot e d’un

CEMC at Home

Grade 9/10 - Wednesday, May 6, 2020

Sheldon’s Shells - Solution

Label the areas of the beach as shown:

Recall that Sheldon only moves either up or right on his way from area A to area Y.

1. On his way from area A to the area with the two identical blue shells, Sheldon can collect2 + 3 = 5 shells (up, right, right), 4 + 3 = 7 shells (right, up, right) or 4 + 2 = 6 shells (right,right, up). This means the largest number of shells he can collect on this trip is 7.

2. Sheldon stops after he has collected 8 shells. One way to figure out where Sheldon might havestopped is to build a tree starting with A and illustrating all the possible paths Sheldon couldtake. Since from A Sheldon can either move up to F (where he collects 2 shells) or right to B(where he collects 4 shells), the tree begins like this:

A

F/2 B/4

From F Sheldon can either move up to K (where he collects 5 more shells for a total of 7) orright to G (where he collects 3 more shells for a total of 5). From B Sheldon can either moveup to G (where he collects 3 more shells for a total of 7) or right to C (where he collects 2 moreshells for a total of 6). Those branches are added to the tree like this:

A

F/2

K/7 G/5

B/4

G/7 C/6

Page 99: CEMI a la maison 9e et 10e ann ee - Lundi 23 mars 2020 ... · 19.Un nombre de Fibonacci. 20.Un multiple de 11. 22.Le plus petit nombre de la grille. 23.Le troisi eme c^ot e d’un

From left to right we can continue to add branches representing Sheldon’s possible paths andcumulative amount of shells collected:

A

F/2

K/7

P/8 L/8

G/5

L/6 H/7

B/4

G/7

L/8 H/9

C/6

H/8 D/10

The areas shaded in red are areas on the path where Sheldon will have collected exactly 8shells. We do not need to add any new branches from these areas since any further movementwill result in collecting more than 8 shells. We also do not need to add any new branches fromthe yellow diamond areas since Sheldon has already exceeded 8 shells along these paths.

A

F/2

K/7

P/8 L/8

G/5

L/6

Q/9 M/10

H/7

M/11 I/8

B/4

G/7

L/8 H/9

C/6

H/8 D/10

From this tree, we can see that the only areas Sheldon could have stopped in (after collectingexactly 8 shells) are P, L, I, or H.

3. Sheldon can collect 25 shells by following the path “A B G H M R S T Y”. It turns out thatthis is the maximum number of shells that he can collect. How can we be sure of this?

One way to figure out the maximum number of shells that Sheldon could collect on his wayfrom A to Y is to continue building the tree from the previous question until all paths reachY. Then the bottom row of the tree can be scanned for the largest number.

How many possible paths from A to Y are there? To get from A to Y Sheldon must move upfour times and right four times, in some sequence. There are 70 different ways to arrange four“up”s and four “right”s, so there are 70 possible paths. That’s going to make a big tree!

Page 100: CEMI a la maison 9e et 10e ann ee - Lundi 23 mars 2020 ... · 19.Un nombre de Fibonacci. 20.Un multiple de 11. 22.Le plus petit nombre de la grille. 23.Le troisi eme c^ot e d’un

There are ways to simplify the tree if you would like to carry through with this approach. Oneway to simplify the tree is to “abandon” a path when a better path is found. For example,consider the beginning of the tree:

A

F/2

K/7 G/5

B/4

G/7 C/6

There are two ways Sheldon can get to area G. Taking the path “A B G” is better than takingthe path “A F G” since it results in more shells. We can abandon path “A F G” by not addingany more branches to it.

A

F/2

K/7

P/8 L/8

G/5

B/4

G/7

L/8 H/9

C/6

H/8 D/10

In addition, we can simplify the tree by “merging” paths. Since two different paths to L resultin the same number of shells (8), we do not lose any information by merging the paths together.

If we continue to expand, abandon, and merge paths we will eventually get a diagram whichshows the maximum number of shells Sheldon can collect.

See the next page for the completed diagram for the tree approach.

Another way to approach a solution to this problem is to do calculations in a grid using thefollowing observation:

The maximum number of shells that can be collected by Sheldon upon reaching a par-ticular area of the beach is the number of shells in this area plus the maximum of

• the total number of shells that can be collected upon reaching the section to theleft, and

• the total number of shells that can be collected upon reaching the section below.

We can compute each of these maximum numbers of shells starting at the bottom left area andworking our way through the grid.

See the next page for the completed grid with these calculations.

Page 101: CEMI a la maison 9e et 10e ann ee - Lundi 23 mars 2020 ... · 19.Un nombre de Fibonacci. 20.Un multiple de 11. 22.Le plus petit nombre de la grille. 23.Le troisi eme c^ot e d’un

Grid approach

In the grid below, the number of shells in each area is displayed in a small box in the bottom leftcorner of the corresponding square. There is only one way to get to each of areas F and B andthe maximum numbers of shells you can collect upon reaching these areas are 2 and 4, respectively.These numbers are entered in the corresponding squares in the grid below. There are two ways toget to area G: through F (from the left) or through B (from below). We see that to collect themaximum number of shells upon reaching G, we want to travel through B, and in doing so we cancollect 4+3 = 7 shells. See if you can work your way through the rest of the calculations in the table.

Grid approach

In the grid below, the number of shells in each area is displayed in a small box in the bottom rightcorner of the corresponding square. There is only one way to get to areas F and B and the maximumnumber of shells you can collect upon reaching these are 2 and 4, respectively. There are two waysto get to area G: through F (from the left) and through B (from below). We see that to collectthe maximum shells upon reaching G, we want to travel through B and in doing so we can collect4 + 3 = 7 shells. See if you can work your way through the rest of the calculations in the table.

0 4 2 4 5

2 3 2 1 2

5 1 4 3 1

1 3 5 2 5

3 4 3 2 0

A 4 4 + 2 = 6 6 + 4 = 10 10 + 5 = 15

2 4 + 3 = 7 7 + 2 = 9 10 + 1 = 11 15 + 2 = 17

2 + 5 = 7 7 + 1 = 8 9 + 4 = 13 13 + 3 = 16 17 + 1 = 18

7 + 1 = 8 8 + 3 = 11 13 + 5 = 18 18 + 2 = 20 20 + 5 = 25

8 + 3 = 11 11 + 4 = 15 18 + 3 = 21 21 + 2 = 23 Y

Each arrow pointing to an area indicates whether the maximum comes from the section to the leftor comes from the section below. When we reach the top right area, we compute the value 25 whichtells us that 25 is the maximum number of shells among all possibilities.

To collect this maximum number of shells, Sheldon should follow the path shown by the red arrows.

Tree approach

Each arrow pointing to an area indicates whether the maximum comes from the section to the leftor comes from the section below (or both). When we reach the top right area, we compute the value25 which tells us that 25 is the maximum number of shells among all possibilities.

To collect this maximum number of shells, Sheldon should follow the path shown by the red arrows.

Tree approach

Grid approach

GRID

Each arrow pointing to an area indicates whether the maximum comes from the section to theleft or comes from the section below. When we reach the top right area, we compute the value25 which tells us that 25 is the maximum number of shells among all possibilities.

To collect this maximum number of shells, Sheldon should follow the path shown by the redarrows.

Tree approach

A

F/2

K/7

P/8

U/11

V/15

W/18

Q/11

R/16

L/8

M/12

G/5

B/4

G/7

H/9

M/13

R/18

W/21

X/23

Y/23

S/20

X/22 T/25

Y/25

N/16

S/18 O/17

I/10

C/6

H/8 D/10

I/11

N/14 J/13

E/15

J/17

O/18

T/23

Looking at the bottom row, we see that the maximum number of shells is 25 and the path to getthere is “A B G H M R S T Y”.

Looking at the bottom row, we see that the maximum number of shells is 25.

We can also see that the path Sheldon should take to collect 25 shells is “A B G H M R S T Y”.

Can you see how the grid and the tree are related?

Page 102: CEMI a la maison 9e et 10e ann ee - Lundi 23 mars 2020 ... · 19.Un nombre de Fibonacci. 20.Un multiple de 11. 22.Le plus petit nombre de la grille. 23.Le troisi eme c^ot e d’un

CEMC.UWATERLOO.CA | le CENTRE d’ÉDUCATION en MATHÉMATIQUES et en INFORMATIQUE

Le CEMI a la maison presente Le probleme de la semaine

9e et 10e annee - le jeudi 7 mai 2020

Un cercle de nombres

Les nombres, 1, 6, 8, 13, 15, et 20 peuvent etre places une seule fois chacun, dans le cercle ci-dessous defacon a ce que la somme de chaque paire de nombres adjacents dans le cercle soit un multiple de sept.

En fait, il existe plus d’une facon d’arranger ces nombres dans le cercle tout en respectant ces regles.Determine tous les differents arrangements possibles. Note que deux arrangements sont consideresidentiques si l’un d’entre eux peut etre obtenu par une serie de reflexions et de rotations de l’autre.

13

1

20

15

6

8

Plus d’infos :

Consulte la page du CEMI a la maison jeudi, le 14 mai, pour la solution a ce probleme.Tu peux egalement t’inscrire au Probleme de la semaine en cliquant sur le lien ci-dessous et recevoirla solution par courriel jeudi, le 14 mai.

Cette ressource du CEMI a la maison correspond au Probleme de la semaine pour les 9e et 10eannees. Le Probleme de la semaine est une ressource gratuite. Chaque semaine, des problemesprovenant de divers domaines mathematiques sont publies en ligne et envoyes par courriel auxenseignants afin qu’ils les utilisent avec leurs etudiants. Les problemes sont disponibles pour lesetudiants de la 3e jusqu’a la 12e annee. Les solutions aux problemes sont envoyees une semaineapres, en meme temps que le nouveau Probleme de la semaine.

Pour plus d’informations, t’inscrire au Probleme de la semaine et de nombreux problemes dejapublies, visite : https://www.cemc.uwaterloo.ca/resources/potw-f.php

1

Page 103: CEMI a la maison 9e et 10e ann ee - Lundi 23 mars 2020 ... · 19.Un nombre de Fibonacci. 20.Un multiple de 11. 22.Le plus petit nombre de la grille. 23.Le troisi eme c^ot e d’un

Problem of the WeekProblem D and Solution

A Circle of Numbers

Problem

13

1

20

15

6

8

The numbers 1, 6, 8, 13, 15, and 20 can be placed in the circle above, each exactly once, sothat the sum of each pair of numbers adjacent in the circle is a multiple of seven. In fact, thereis more than one way to arrange the numbers in such a way in the circle. Determine alldifferent arrangements. Note that we will consider two arrangements to be the same if one canbe obtained from the other by a series of reflections and rotations.

SolutionWe will start by writing down all the pairs of numbers that add to a multiple of 7.

Sum of 7 Sum of 14 Sum of 21 Sum of 28 Sum of 351,6 1,13 6,15 13,15 15,20

6,8 8,13 8,201,20

To show these connections visually, we can write the numbers in a circle anddraw a line connecting numbers that add to a multiple of 7.

8

6

1

20

15

13

We will now determine all the different arrangements by looking at various cases.Note that in order for two arrangements to be different, at least some of thenumbers need to be adjacent to different numbers.

Now, consider the possibilities for the numbers adjacent to 1. Since 6, 13, and 20are the only numbers in our list that add with 1 to make a multiple of 7, thereare three possible cases: 1 adjacent to 6 and 20, 1 adjacent to 6 and 13, and 1adjacent to 13 and 20. We consider each case separately.

Page 104: CEMI a la maison 9e et 10e ann ee - Lundi 23 mars 2020 ... · 19.Un nombre de Fibonacci. 20.Un multiple de 11. 22.Le plus petit nombre de la grille. 23.Le troisi eme c^ot e d’un

Case 1: 1 is adjacent to 6 and 20

In this case, we can see from our table that 13 must be adjacent to 15 and 8,since 1 is no longer available. The two different ways to write such a circle areshown below.

13

1

20

15

6

8

15

6

1

20

8

13

8

20

1

6

15

13

Case 2: 1 is adjacent to 6 and 13

In this case, we can see from our table that 20 must be adjacent to 15 and 8,since 1 is no longer available. The two different ways to write such a circle areshown below.

13

1

20

15

6

8

15

6

1

13

8

20

8

13

1

6

15

20

Case 3: 1 is adjacent to 13 and 20

In this case, we can see from our table that 6 must be adjacent to 15 and 8, since1 is no longer available. The two different ways to write such a circle are shownbelow.

13

1

20

15

6

8

15

13

1

20

8

6

8

20

1

13

15

6

Therefore, we have found that there are 6 different arrangements. These are thearrangements shown in Cases 1, 2 and 3 above.

Page 105: CEMI a la maison 9e et 10e ann ee - Lundi 23 mars 2020 ... · 19.Un nombre de Fibonacci. 20.Un multiple de 11. 22.Le plus petit nombre de la grille. 23.Le troisi eme c^ot e d’un

CEMC.UWATERLOO.CA | le CENTRE d’ÉDUCATION en MATHÉMATIQUES et en INFORMATIQUE

Le CEMI a la maison

9e et 10e annee - le vendredi 8 mai 2020Les problemes non resolus

Nombres parfaits impairs

Un nombre parfait est un entier strictement positif egal a la somme de ses diviseurs positifs propres. Undiviseur positif est dit propre s’il est inferieur au nombre lui-meme. Par exemple, les diviseurs positifspropres de 6 sont 1, 2 et 3. Puisque 1 + 2 + 3 = 6, 6 est un nombre parfait. Les diviseurs propres de28 sont 1, 2, 4, 7 et 14. Puisque 1 + 2 + 4 + 7 + 14 = 28, 28 est egalement un nombre parfait.

(a) Est-ce que 120 est un nombre parfait ?

(b) Est-ce que 496 est un nombre parfait ?

(c) Y a-t-il des nombres parfaits impairs ?

Presentement, la reponse a la question (c) est : On ne sait pas ! L’existence d’un nombre parfait impairest un probleme non resolu. A ce jour, les mathematiciens ont verifie tous les entiers strictement positifsjusqu’a au moins 101500 et n’ont toujours pas trouve un seul entier strictement positif qui soit a la foisimpair et parfait. Est-ce a dire qu’aucun tel nombre n’existe ? Pour resoudre de maniere concluantece probleme de l’existence ou non d’un nombre parfait impair, on doit soit trouver un exemple d’unnombre parfait impair, soit elaborer une justification (une demonstration) que tous les nombres parfaitssont pairs. A ce jour, nous n’avons ni l’un ni l’autre.

La conjecture de Goldbach

Considere l’entier 16. On peut l’exprimer comme la somme de deux nombres premiers : 16 = 5 + 11.

Il y a plus d’une seule facon d’exprimer 16 comme la somme de deux nombres premiers. Quellessont-elles ?

N’oublie pas qu’un nombre premier est un entier p superieur a 1 qui n’admet que deux diviseurspositifs, soit 1 et p (lui-meme). Citons quelques nombres premiers : 2, 3, 5, 7, 11, . . .

Quels nombres peuvent etre exprimes comme la somme de deux nombres premiers d’au moins unemaniere ?

(a) Exprime 34 comme la somme de deux nombres premiers.

(b) Exprime 52 comme la somme de deux nombres premiers.

(c) Explique pourquoi on ne peut exprimer 41 comme la somme de deux nombres premiers.

(d) Choisis un entier pair superieur a 2 et exprime-le comme la somme de deux nombres premiers.

La conjecture de Goldbach s’enonce comme suit : On peut exprimer tout entier pair superieur a 2comme la somme de deux nombres premiers. Quoique toutes les preuves decouvertes jusqu’a presentsuggerent qu’elle est vraie, la conjecture de Goldbach est une conjecture et non un theoreme puis-qu’aucune demonstration n’a encore ete decouverte. A ce jour, les mathematiciens ont verifie tous lesentiers pairs superieurs a 2 jusqu’a environ 1018 et ont pu exprimer chacun comme la somme de deuxnombres premiers. Est-ce a dire que tous les entiers pairs superieurs a 2 doivent avoir cette propriete ?Pour resoudre ce probleme de maniere concluante, on doit soit trouver un nombre pair superieur a 2qu’on ne peut exprimer comme la somme de deux nombres premiers, soit elaborer une justification(une demonstration) que tous le nombres pairs superieurs a 2 peuvent etre exprimes de cette maniere.A ce jour, nous n’avons ni l’un ni l’autre.

Page 106: CEMI a la maison 9e et 10e ann ee - Lundi 23 mars 2020 ... · 19.Un nombre de Fibonacci. 20.Un multiple de 11. 22.Le plus petit nombre de la grille. 23.Le troisi eme c^ot e d’un

CEMC.UWATERLOO.CA | le CENTRE d’ÉDUCATION en MATHÉMATIQUES et en INFORMATIQUE

La suite de Syracuse

Considere une suite dont le premier terme est un entier strictement positif n et dans laquelle chaqueterme, apres le premier, est obtenu comme suit :

— Si le terme precedent est pair, le terme suivant sera la moitie de son predecesseur.

— Si le terme precedent est impair, le terme suivant sera un de plus que trois fois son predecesseur.

Cette suite est connue sous le nom de suite de Syracuse. La conjecture de Syracuse s’enonce commesuit : Quel que soit l’entier strictement positif n choisi comme premier terme d’une suite Syracuse,la suite atteindra toujours 1. Quoique maints mathematiciens y croient, la conjecture de Syracuse estune conjecture et non un theoreme puisqu’aucune demonstration n’a encore ete decouverte.

Si tu as de l’experience en programmation, essaie d’ecrire un programme informatique qui prend commedonnee d’entree un entier strictement positif n et qui genere une suite de Syracuse comme donnees desortie (s’arretant lorsque la suite atteint 1).

Si le programme ne s’arrete pas, peux-tu conclure que la suite n’atteint jamais 1 ? Pourquoi ou pourquoipas ?

Par ailleurs, tu peux experimenter avec la suite en utilisant un programme informatique que nousavons ecrit dans le langage Python. Suis les instructions ci-dessous.

Instructions :

1. Ouvre cette page web dans un onglet de ton navigateur Internet. Tu devrais y voir du codePython.

2. Ouvre cet interpreteur Python gratuit dans un autre onglet. Tu devrais y voir un panneaudu milieu designe par main.py.

3. Copie le code et colle-le dans le panneau du milieu de l’interpreteur.

4. Appuie sur run. Cela te permet d’interagir avec le programme dans le panneau noir adroite. Entre un entier strictement positif comme donnee et observe la suite de Syracusegeneree.

5. Si tu rencontres une erreur ou si tu veux explorer une suite differente, tu peux appuyer surrun pour recommencer.

Plus d’infos :

Consulte la page du CEMI a la maison vendredi, le 15 mai, pour une discussion de ces questions.

Page 107: CEMI a la maison 9e et 10e ann ee - Lundi 23 mars 2020 ... · 19.Un nombre de Fibonacci. 20.Un multiple de 11. 22.Le plus petit nombre de la grille. 23.Le troisi eme c^ot e d’un

CEMC at Home

Grade 9/10 - Friday, May 8, 2020

Unsolved Problems - Solution

Odd Perfect Numbers

(a) Is 120 a perfect number?

The proper divisors of 120 are 1, 2, 3, 4, 5, 6, 8, 10, 12, 15, 20, 24, 30, 40, and 60.Since 1 + 2 + 3 + 4 + 5 + 6 + 8 + 10 + 12 + 15 + 20 + 24 + 30 + 40 + 60 = 240 and not 120,120 is not a perfect number.

(b) Is 496 a perfect number?

The proper divisors of 496 are 1, 2, 4, 8, 16, 31, 62, 124, and 248.Since 1 + 2 + 4 + 8 + 16 + 31 + 62 + 124 + 248 = 496, 496 is a perfect number.

(c) Are there any odd perfect numbers?

The answer to this question is currently unknown. It is known that there are no odd perfectnumbers between 1 and 101500.

The Goldbach Conjecture

(a) Express 34 as the sum of two prime numbers.

There are four different ways to express 34 as the sum of two prime numbers:

34 = 3 + 31 = 5 + 29 = 11 + 23 = 17 + 17

(b) Express 52 as the sum of two prime numbers.

There are three different ways to express 52 as the sum of two prime numbers:

52 = 5 + 47 = 11 + 41 = 23 + 29

(c) Explain why 41 cannot be written as the sum of two prime numbers.

You could write out a list of all primes less than 41 and check that no pair adds to 41. A fasterway to approach this problem is as follows: Since 41 is odd, if 41 can be written as a sum oftwo primes then it must be an even prime plus an odd prime. The only even prime is 2 andso the only possibility is 41 = 2 + p where p is an odd prime. But we can see that p must be41 − 2 = 39 which is not prime.

(d) Choose any even integer greater than 2 and express it as the sum of two prime numbers.

It is not known whether there exists a general strategy that can be followed to write any eveninteger greater than 2 as the sum of two prime numbers. It is known that there is a way towrite any even integer from 4 to around 1018 as the sum of two primes numbers.

1

Page 108: CEMI a la maison 9e et 10e ann ee - Lundi 23 mars 2020 ... · 19.Un nombre de Fibonacci. 20.Un multiple de 11. 22.Le plus petit nombre de la grille. 23.Le troisi eme c^ot e d’un

Hailstone Sequence

Consider a computer program that takes as input a positive integer, n, and outputs the hailstonesequence, stopping when the sequence reaches 1. If the program doesn’t stop, can you conclude thesequence never reaches 1? Why or why not?

If the program doesn’t stop you cannot conclude that the sequence never reaches 1. That might bethe reason why it never stopped, but there are two other possibilities as well.

• There might be an error in the program. For example, the sequence might not be generatedcorrectly.

• You might not have waited long enough. It is possible that the number 1 is a term in the se-quence but it does not appear for a very long time. Perhaps the program would stop eventually,given enough time.

You may have noticed that mathematicians have checked many more numbers in an effort to find anodd perfect number (up to around 101500) than they have checked to see if the Goldbach conjecture istrue (up to around 1018). You may want to do some research to get an idea of why this might be thecase. Is there some reason to believe that it is much easier to check if a number is perfect than tocheck if a number is the sum of two prime numbers? How difficult is it to check these conditions forvery large numbers?

2

Page 109: CEMI a la maison 9e et 10e ann ee - Lundi 23 mars 2020 ... · 19.Un nombre de Fibonacci. 20.Un multiple de 11. 22.Le plus petit nombre de la grille. 23.Le troisi eme c^ot e d’un

CEMC.UWATERLOO.CA | le CENTRE d’ÉDUCATION en MATHÉMATIQUES et en INFORMATIQUE

Le CEMI a la maison

9e et 10e annee - le lundi 11 mai 2020Concours - jour 2

La ressource d’aujourd’hui presente deux questions des concours de mathematiques 2020 du CEMI.

Concours de mathematiques canadien par equipe 2020, probleme d’equipe nº9

Combien de fois le chiffre 0 apparaıt-il dans l’entier egal a 2010 ?

Concours de mathematiques canadien par equipe 2020, probleme individuel nº7

Vingt-sept cubes unites sont chacun colores soit completement en noir ou completement en rouge.Les cubes unites sont assembles en un cube plus grand. Si 1

3de l’aire de la surface du grand cube est

rouge, quel est le plus petit nombre de cubes unites qui auraient pu etre colores en rouge ?

Plus d’infos :

Consulte la page internet du CEMI a la maison jeudi, le 21 mai, pour les solutions aux problemes deConcours Jour 2.

Page 110: CEMI a la maison 9e et 10e ann ee - Lundi 23 mars 2020 ... · 19.Un nombre de Fibonacci. 20.Un multiple de 11. 22.Le plus petit nombre de la grille. 23.Le troisi eme c^ot e d’un

CEMC.UWATERLOO.CA | le CENTRE d’ÉDUCATION en MATHÉMATIQUES et en INFORMATIQUE

Le CEMI a la maison

9e et 10e annee - le lundi 11 mai 2020Concours, jour 2 - solutions

Voici les solutions aux deux problemes de concours. La solution du second probleme est accompagneed’une video.

Concours de mathematiques canadien par equipe 2020, probleme d’equipe no 9

Combien de fois le chiffre 0 apparaıt-il dans l’entier egal a 2010 ?

Solution :

En factorisant et en appliquant les lois des exposants, on a 2010 = (2× 10)10 = 210 × 1010.Donc, 2010 = 1024× 1010, soit l’entier 1024 suivi de dix zeros.Donc, le chiffre 0 parait onze fois dans 2010 ; 10 zeros proviennent de la fin de l’entier et 1 zero provientdu 1024 au debut.

Concours de mathematiques canadien par equipe 2020, probleme individuel no 7

Vingt-sept cubes unites sont chacun colores soit completement en noir ou completement en rouge.Les cubes unites sont assembles en un cube plus grand. Si 1

3de l’aire de la surface du grand cube est

rouge, quel est le plus petit nombre de cubes unites qui auraient pu etre colores en rouge ?

Solution :

Puisque 3√

27 = 3, le grand cube doit avoir pour dimensions 3× 3× 3.Donc, chacune des faces du grand cube a une aire de 3× 3 = 9.Un cube a 6 faces, donc l’aire totale du cube est composee de 9 × 6 = 54 carres 1 × 1 (ces derniersetant les faces des cubes unites).Puisque 1

3de l’aire totale du grand cube est rouge, alors 54

3= 18 de ces carres unites sont de couleur

rouge.

Aucune des faces du cube unite au centre du grand cube n’est visible. Les 6 cubes unites au centredes faces du grand cube n’ont chacun qu’une seule face visible. Les 12 cubes unites situes le long desaretes (et n’etant pas situes aux extremites memes de ces dernieres) ont chacun deux faces visibles.Les 8 cubes unites situes aux 8 coins du grand cube ont chacun trois faces visibles.Chacun des 27 cubes unites a soit 0, 1, 2 ou 3 de ses faces visibles sur les faces du grand cube.Cela signifie qu’au plus trois faces d’un cube unite se trouvent sur les faces du grand cube. Donc, afind’avoir 18 carres unites rouges sur les faces du grand cube, il doit y avoir au moins 6 cubes unitesrouges.

Il y a 8 cubes unites situes aux 8 coins du grand cube. Donc, si on colorie exactement 6 cubes unitesen rouge et les 21 cubes unites restants en noir et qu’on arrange ces cubes en un cube plus grand dedimensions 3× 3× 3 de maniere que les 6 cubes unites rouges soient situes aux coins du grand cube,alors il y aura exactement 18 carres unites rouges sur les faces du grand cube.Donc, la reponse est 6.

Video

Clique sur le lien suivant pour une explication de la solution a ce deuxieme probleme :https://youtu.be/K9ax9uQESME

Page 111: CEMI a la maison 9e et 10e ann ee - Lundi 23 mars 2020 ... · 19.Un nombre de Fibonacci. 20.Un multiple de 11. 22.Le plus petit nombre de la grille. 23.Le troisi eme c^ot e d’un

CEMC.UWATERLOO.CA | le CENTRE d’ÉDUCATION en MATHÉMATIQUES et en INFORMATIQUE

Le CEMI a la maison

9e et 10e annee - le mardi 12 mai 2020Des cercles ombres

Pour chaque probleme, utilise les informations et la figure donnees pour determiner l’aire de laregion ombree. Exprime tes reponses sous forme de nombres exacts reduits tels que π+1 et 1−

√2.

Probleme 1

Dans la figure ci-contre, on voit deux cercles de centres C et O.Le cercle de centre O a pour diametre AB. Le cercle de centreC a pour diametre OB. Le cercle de centre O a un diametre de 20.

Probleme 2

Dans la figure ci-contre, le cercle de centre O a un rayon de2. Les points A et B sont situes sur le cercle tels que ∠AOB = 90◦.

25

Probleme 3

Dans la figure ci-contre, le carre ABCD est inscrit dans le cerclede centre O et de rayon 5.

Un carre est inscrit dans un cercle si les quatre sommets du carresont situes sur le cercle.

25

Probleme 4

Dans la figure ci-contre, deux cercles de centres C et O ont chacunun rayon de 10. Les cercles se coupent en A et B tels que∠AOB = 60◦.

Peux-tu visualiser a quoi ressemblerait cette figure si on separaitles cercles ?

Plus d’infos :

Consulte la page du CEMI a la maison mardi, le 19 mai, pour les solutions a ces problemes.

Pour reviser comment calculer l’aire d’un triangle et l’aire d’un cercle, visionne les videos suivantesdans le didactitiel du CEMI : l’aire d’un triangle et l’aire d’un cercle.

Page 112: CEMI a la maison 9e et 10e ann ee - Lundi 23 mars 2020 ... · 19.Un nombre de Fibonacci. 20.Un multiple de 11. 22.Le plus petit nombre de la grille. 23.Le troisi eme c^ot e d’un

CEMC at Home

Grade 9/10 - Tuesday, May 12, 2020

Shady Circles - Solution

Problem 1 Solution

We will find the area of the shaded region by subtracting the areaof the smaller circle from the area of the larger circle.Let R be the radius of the larger circle and r be the radius of thesmaller circle.

Since the diameter of the smaller circle is the radius of the largercircle, we have R = 20

2= 10 and r = 10

2= 5.

Alarger = πR2 = π(10)2 = 100πAsmaller = πr2 = π(5)2 = 25π

Therefore, the area of the shaded region is 100π − 25π = 75π.

Problem 2 Solution

To find the area of the shaded region, we will find the area of the sector of the circle with arc ABand subtract the area of 4AOB. Note that the triangle is a right isosceles triangle and therefore,the base and height are both equal to the radius which is 2.

AwholeCircle = πr2 = π(2)2 = 4π

Asector =

(90

360

)4π = π

Atriangle =bh

2=

(2)(2)

2= 2

Therefore, the area of the shaded region is π − 2.

Problem 3 Solution

There are a few different ways to approach this problem. We will outline two approaches. Each ofthese approaches relies upon the following facts that we will not prove:

1) The two diagonals of the inscribed square intersect at the centre,O, of the circle.

2) The two diagonals of the inscribed square bisect each other andmeet at right angles.

Page 113: CEMI a la maison 9e et 10e ann ee - Lundi 23 mars 2020 ... · 19.Un nombre de Fibonacci. 20.Un multiple de 11. 22.Le plus petit nombre de la grille. 23.Le troisi eme c^ot e d’un

Approach 1: Recognize that the shaded region in this problem consists of four identical shadedregions, each having an area that can be calculated by subtracting the area of a triangle from thearea of a sector of a the circle (as in Problem 2).

The final calculation is as follows: Area = 4(π(5)2

4− 52

2

)= 25π − 50.

Approach 2: Recognize that the area of the shaded region is the area of the circle minus the area ofthe inscribed square.

The area of the circle πr2 = π(5)2 = 25π.

Let s be the side length of the square as shown in the figures below.

Note that 4BOC is a right isosceles triangle. Therefore, its base and height are both equal to theradius which is r = 5. We can calculate the value of s2 as follows:

Using the Pythagorean Theorem on 4BOC, we get

s2 = r2 + r2

s2 = (5)2 + (5)2

s2 = 25 + 25

s2 = 50

Alternatively, using the Pythagorean Theorem on 4BAD, withdiameter d = 10, we get

d2 = s2 + s2

102 = 2s2

100 = 2s2

50 = s2

Each of these calculations tells us that the area of the square is s2 = 50.

Therefore, the area of the shaded region is 25π − 50.

Problem 4 Discussion

One way to find the area of the shaded region is to observe that it is made up of two identical regionsas shown below. You can find the area of each of the regions using a similar method to that in thesolution to Problem 2, although the area of the triangle will not be as easy to calculate in this case.We leave the details to you, but give the key values in the calculations here.

Area of triangle AOB is1

2(10)(

√75)

Area of sector AOB is60

360(π(10)2)

Page 114: CEMI a la maison 9e et 10e ann ee - Lundi 23 mars 2020 ... · 19.Un nombre de Fibonacci. 20.Un multiple de 11. 22.Le plus petit nombre de la grille. 23.Le troisi eme c^ot e d’un

CEMC.UWATERLOO.CA | le CENTRE d’ÉDUCATION en MATHÉMATIQUES et en INFORMATIQUE

Le CEMI a la maison

9e et 10e annee - le jeudi 21 mai 2020

Concours - jour 3

La ressource d’aujourd’hui presente deux questions des concours mathematiques 2020 du CEMI.

Concours de mathematiques canadien par equipe 2020, probleme d’equipe no 5

Quel est le plus petit entier strictement positif a huit chiffres dont exactement quatre des chiffressont 4 ?

Concours de mathematiques canadien par equipe 2020, probleme d’equipe no 14

Jean a peche 21 poissons dont chacun avait une masse d’au moins 0,2 kg. Il a remarque que la massemoyenne des trois premiers poissons qu’il avait peches etait egale a la masse moyenne des 21 poissons.Les trois premiers poissons avaient une masse totale de 1,5 kg. Quelle est la masse du poisson le pluslourd que Jean aurait pu pecher ?

Plus d’infos :

Consulte la page internet du CEMI a la maison lundi, le 25 mai, pour les solutions aux problemes deConcours Jour 3.

Page 115: CEMI a la maison 9e et 10e ann ee - Lundi 23 mars 2020 ... · 19.Un nombre de Fibonacci. 20.Un multiple de 11. 22.Le plus petit nombre de la grille. 23.Le troisi eme c^ot e d’un

CEMC.UWATERLOO.CA | le CENTRE d’ÉDUCATION en MATHÉMATIQUES et en INFORMATIQUE

Le CEMI a la maison

9e et 10e annee - le jeudi 21 mai 2020

Concours, jour 3 - solutions

Voici les solutions aux deux problemes de concours.

Concours de mathematiques canadien par equipe 2020, probleme d’equipe no 5

Quel est le plus petit entier strictement positif a huit chiffres dont exactement quatre des chiffressont 4 ?

Solution :

Le plus petit nombre a huit chiffres est 10 000 000.Les 10 000 plus petits nombres a huit chiffres sont ceux de la forme 10 00a bcd ou a, b, c, et d sontles chiffres. Parmi les 10 000 plus petits nombres a huit chiffres, 10 004 444 est le seul qui ait quatrechiffres egaux a 4.Cela signifie que le plus petit nombre entier positif a huit chiffres ayant quatre chiffres qui sont des4 est 10 004 444.

Concours de mathematiques canadien par equipe 2020, probleme d’equipe no 14

Jean a peche 21 poissons dont chacun avait une masse d’au moins 0,2 kg. Il a remarque que la massemoyenne des trois premiers poissons qu’il avait peches etait egale a la masse moyenne des 21 poissons.Les trois premiers poissons avaient une masse totale de 1,5 kg. Quelle est la masse du poisson le pluslourd que Jean aurait pu pecher ?

Solution :

Comme la masse totale des trois premiers poissons est de 1, 5 kg, la masse moyenne des trois premierspoissons est de 0, 5 kg.Soit M la masse totale de tous les poissons. Puisque la masse moyenne des trois premiers poissons est

la meme que la masse moyenne de tous les poissons, cela signifie queM

21= 0, 5 kg ou M = 10, 5 kg.

Comme les trois premiers poissons ont une masse totale de 1, 5 kg, cela signifie que les 18 dernierspoissons que Jeff a peches ont une masse totale de 10, 5 kg − 1, 5 kg = 9 kg.Si 17 de ces 18 poissons ont une masse aussi faible que possible, le 18eme de ces poissons aura unemasse aussi importante que possible.La plus petite masse possible est de 0, 2 kg, donc la masse totale de 17 poissons, chacun ayant unemasse aussi petite que possible, est de 17 × 0, 2 kg = 3, 4 kg.La plus grande masse possible de tout poisson que Jeff aurait pu pecher est de 9 kg−3, 4 kg = 5, 6 kg.

Page 116: CEMI a la maison 9e et 10e ann ee - Lundi 23 mars 2020 ... · 19.Un nombre de Fibonacci. 20.Un multiple de 11. 22.Le plus petit nombre de la grille. 23.Le troisi eme c^ot e d’un

CEMC.UWATERLOO.CA | le CENTRE d’ÉDUCATION en MATHÉMATIQUES et en INFORMATIQUE

Le CEMI a la maison

9e et 10e annee - le vendredi 22 mai 2020Des puzzles binaires

Un puzzle binaire est un type de puzzle logique qui se fait sur une grille n× n ou n est pair. Ta tacheest de completer la grille en remplissant toutes les cellules vides selon les regles suivantes :

1. Chaque cellule de la grille doit contenir soit le chiffre 0 ou le chiffre 1.2. La grille ne peut pas avoir trois cellules consecutives ou plus, horizontalement ou vertica-

lement, qui contiennent toutes le meme chiffre.3. La moitie des chiffres dans chaque rangee de la grille doivent etre des 0 tandis que l’autre

moitie des chiffres doivent etre des 1. Il en va de meme pour chaque colonne.4. La grille ne peut pas contenir deux rangees identiques. Il en va de meme pour les colonnes

de la grille.

Exemple : Completons la rangee suivante dans une grille de puzzle binaire en suivant les troispremieres regles. Pour s’assurer de satisfaire a la regle 4, il faudrait considerer toutes les rangeeset les colonnes de la grille simultanement.

1 1 0 1On remarque que la troisieme cellule doit contenir un 0,sinon la rangee aurait trois cellules consecutives conte-nant des 1 (ce qui va a l’encontre de la deuxieme regle).

1 1 0 0 1

On remarque maintenant que la quatrieme cellule doitcontenir un 1, sinon la rangee aurait trois cellulesconsecutives contenant des 0 (ce qui va a l’encontre dela deuxieme regle).

1 1 0 1 0 1La rangee contient desormais quatre 1 et deux 0. Selonla troisieme regle, les deux cellules restantes doiventcontenir des 0.

1 1 0 1 0 0 1 0Donc, en suivant les regles enoncees ci-dessus, on nepeut completer cette rangee particuliere que d’une seulemaniere.

Remarque qu’on peut employer des raisonnements differents pour completer cette rangee. Par exemple,on pourrait commencer en observant que la rangee originale partiellement completee contient dejatrois 1 et qu’il faudrait alors utiliser exactement un 1 et trois 0 pour remplir les cellules restantes de larangee. On pourrait ensuite justifier qu’on n’a pas d’autre choix que de placer le 1 dans la quatriemecellule. Vois-tu pourquoi ? Qu’est-ce qui se produirait si on placait le 1 dans une cellule differente ?

Essaie les quatre puzzles binaires a la page suivante.

Ces puzzles augmentent en difficulte mais peuvent tous etre completes en utilisant un raisonnementsolide. Amuse-toi bien !

Plus d’infos : Consulte la page du CEMI a la maison vendredi, le 29 mai, pour les solutions a cespuzzles binaires. Le titre des puzzles binaires est une reference au systeme binaire. Ce systeme denumeration emploie uniquement les chiffres 0 et 1. Certaines branches des mathematiques ainsi quede nombreux appareils electroniques, y compris les ordinateurs, emploient le systeme binaire.

Page 117: CEMI a la maison 9e et 10e ann ee - Lundi 23 mars 2020 ... · 19.Un nombre de Fibonacci. 20.Un multiple de 11. 22.Le plus petit nombre de la grille. 23.Le troisi eme c^ot e d’un

CEMC.UWATERLOO.CA | le CENTRE d’ÉDUCATION en MATHÉMATIQUES et en INFORMATIQUE

PUZZLE 1

0 1

1 0 1 0

1 1 0

1 1 1

1 1

0 0 1

1 0 1

0 1

PUZZLE 2

1 1 1

0

0 0 1 1 1

0 0

0 1

0

1 1 1

PUZZLE 3

1 1 0

1

0 0

0

1 1 1

0 1 1

1 0

PUZZLE 4

0 0 1

1 1

0 0

0 0

0 1 1

1 1 1

Page 118: CEMI a la maison 9e et 10e ann ee - Lundi 23 mars 2020 ... · 19.Un nombre de Fibonacci. 20.Un multiple de 11. 22.Le plus petit nombre de la grille. 23.Le troisi eme c^ot e d’un

CEMC at Home

Grade 9/10 - Friday, May 22, 2020

Binary Puzzles - Solution

PUZZLE 1

0 0 1 0 1 1 0 1

1 1 0 0 1 0 1 0

1 0 1 1 0 0 1 0

0 0 1 1 0 1 0 1

1 1 0 0 1 1 0 0

1 0 0 1 0 0 1 1

0 1 1 0 0 1 0 1

0 1 0 1 1 0 1 0

PUZZLE 2

0 0 1 1 0 0 1 1

1 1 0 0 1 1 0 0

1 0 1 1 0 1 0 0

0 1 0 0 1 0 1 1

0 1 1 0 1 0 0 1

1 0 0 1 0 1 1 0

0 1 1 0 0 1 0 1

1 0 0 1 1 0 1 0

PUZZLE 3

1 0 0 1 1 0 1 0

0 1 1 0 0 1 0 1

0 1 0 1 0 1 1 0

1 0 1 0 1 0 0 1

0 1 0 1 0 1 0 1

1 0 1 0 0 1 1 0

1 0 1 0 1 0 1 0

0 1 0 1 1 0 0 1

PUZZLE 4

0 1 1 0 0 1 1 0

1 0 1 0 1 0 0 1

1 0 0 1 0 1 0 1

0 1 0 1 1 0 1 0

1 0 1 0 1 1 0 0

1 0 0 1 0 0 1 1

0 1 1 0 1 0 1 0

0 1 0 1 0 1 0 1

1

Page 119: CEMI a la maison 9e et 10e ann ee - Lundi 23 mars 2020 ... · 19.Un nombre de Fibonacci. 20.Un multiple de 11. 22.Le plus petit nombre de la grille. 23.Le troisi eme c^ot e d’un

CEMC.UWATERLOO.CA | le CENTRE d’ÉDUCATION en MATHÉMATIQUES et en INFORMATIQUE

Le CEMI a la maison

9e et 10e annee - le lundi 25 mai 2020Concours - jour 4

La ressource d’aujourd’hui presente une question des concours mathematiques 2020 du CEMI.

Concours Fryer 2020, no 2

Dans la figure ci-contre, les sommets du rectangleJKLM sont situes sur les cotes du triangle PQR demaniere que PJ = PK = 5 m, JQ = KR = 50 m,KL = 40 m et QR = 66 m.

(a) Quelle est la longueur de LR ?

(b) Quelle est la longueur de ML ?

(c) Determiner la hauteur du triangle PJK (soit lalongueur du segment de droite qui joint P au coteJK).

(d) Determiner la fraction de l’aire du triangle PQRqui est recouverte par le rectangle JKLM .

P

Q R

J K

LM

66 m

5 m5 m

40 m

50 m50 m

Plus d’infos :

Consulte la page internet du CEMI a la maison lundi, le 1er juin, pour la solution au probleme deConcours Jour 4.

Page 120: CEMI a la maison 9e et 10e ann ee - Lundi 23 mars 2020 ... · 19.Un nombre de Fibonacci. 20.Un multiple de 11. 22.Le plus petit nombre de la grille. 23.Le troisi eme c^ot e d’un

CEMC.UWATERLOO.CA | le CENTRE d’ÉDUCATION en MATHÉMATIQUES et en INFORMATIQUE

Le CEMI a la maison

9e et 10e annee - le lundi 25 mai 2020Concours, jour 4 - solutions

Voici les solutions au probleme du concours, accompagne d’une video.

Concours Fryer 2020, no 2

Dans la figure ci-contre, les sommets du rectangleJKLM sont situes sur les cotes du triangle PQR demaniere que PJ = PK = 5 m, JQ = KR = 50 m,KL = 40 m et QR = 66 m.

(a) Quelle est la longueur de LR ?

(b) Quelle est la longueur de ML ?

(c) Determiner la hauteur du triangle PJK (soit lalongueur du segment de droite qui joint P au coteJK).

(d) Determiner la fraction de l’aire du triangle PQRqui est recouverte par le rectangle JKLM .

P

Q R

J K

LM

66 m

5 m5 m

40 m

50 m50 m

Solution :

(a) Dans le triangle KLR, on a ∠KLR = 90◦. D’apres le theoreme de Pythagore,LR2 = 502 − 402 = 900, d’ou LR =

√900 = 30 m (puisque LR > 0).

(b) On demontre d’abord que les triangles JMQ et KLR sont isometriques.Puisque JKLM est un rectangle, alors JM = KL = 40 m.De plus, les hypotenuses JQ et KR sont de meme longueur. Donc, les triangles JMQ et KLRsont isometriques puisque leurs cotes sont isometriques deux a deux.Donc, MQ = LR = 30 m, d’ou ML = 66− 30− 30 = 6 m.

(c) Puisque PJ = PK = 5 m, alors le triangle PJK est isoceleet sa hauteur, PS, est la droite menee du milieu de sa baseJK jusqu’a P .Puisque JKLM est un rectangle, alors JK = ML = 6 m,d’ou SK = JK

2= 3 m.

Dans le triangle PSK, on a PS2 = 52 − 32 = 16 d’apres letheoreme de Pythagore, d’ou PS = 4 m (puisque PS > 0).Donc, la hauteur du triangle PJK, soit la droite menee dumilieu de sa base JK jusqu’a P , est de 4 m.

P

Q R

J K

LM

66 m

5 m5 m

40 m

50 m50 m

S

Voir la page suivante pour une solution a la partie (d) et un lien vers la video.

Page 121: CEMI a la maison 9e et 10e ann ee - Lundi 23 mars 2020 ... · 19.Un nombre de Fibonacci. 20.Un multiple de 11. 22.Le plus petit nombre de la grille. 23.Le troisi eme c^ot e d’un

CEMC.UWATERLOO.CA | le CENTRE d’ÉDUCATION en MATHÉMATIQUES et en INFORMATIQUE

(d) On determine d’abord l’aire du triangle PQR.Dans la figure ci-contre, soit la hauteur du triangle PQR ladroite menee de T (le milieu de sa base QR) jusqu’a P .Puisque PT et QR sont perpendiculaires, alors PT et KLsont paralleles (puisque KL est egalement perpendiculairea QR).Par symetrie, PT passe par le point S. La hauteur PT estdonc egale a PS + ST = PS + KL ou 4 + 40 = 44 m.L’aire du triangle PQR est egale a12×QR× PT = 1

2× 66× 44 = 1452 m2.

L’aire du rectangle JKLM est egale aML×KL = 6× 40 = 240 m2.La fraction de l’aire du triangle PQR qui est recouverte parle rectangle JKLM est donc egale a 240

1452= 20

121.

P

Q R

J K

LM

66 m

5 m5 m

40 m

50 m50 m

S

T

Video

Clique le lien suivant pour une discussion a propos de la solution de ce probleme de concours :https://youtu.be/gMpResbow9E

Page 122: CEMI a la maison 9e et 10e ann ee - Lundi 23 mars 2020 ... · 19.Un nombre de Fibonacci. 20.Un multiple de 11. 22.Le plus petit nombre de la grille. 23.Le troisi eme c^ot e d’un

CEMC.UWATERLOO.CA | le CENTRE d’ÉDUCATION en MATHÉMATIQUES et en INFORMATIQUE

Le CEMI a la maison

9e et 10e annee - le mardi 26 mai 2020Des carres parfaits

Un carre parfait (ou un nombre carre) est un entier qui est le carre d’un entier. Autrement dit, unentier s est un carre parfait lorsque s = n2, n etant un entier quelconque. On peut illustrer un carreparfait de plusieurs facons. Ces facons impliquent souvent l’aspect geometrique d’un carre.

Considere le carre parfait 16 = 42

Puisque 16 = 42, on peut illustrer le carre par-fait 16 en dessinant 16 points que l’on disposede maniere a creer une grille carree 4 × 4 :

Que remarques-tu si on regroupe les points de lagrille comme dans la figure ci-dessous ? Peux-tuvoir comment cela est une autre facon d’illustrerle nombre 16 ?

On peut verifier que 16 = 42 est la somme des 4 premiers entiers positifs impairs : 16 = 1 + 3 + 5 + 7.On comprend des illustrations ci-dessus pourquoi cela est vrai. Si on illustrait le carre parfait 25 = 52

de maniere a creer une grille carree 5 × 5 et qu’on regroupait les 25 points de la meme maniere quedans la figure ci-dessus, qu’observerait-on ?

Fait amusant : Le carre parfait s = n2 (n etant un entier strictement positif) est egal a lasomme des n premiers entiers positifs impairs consecutifs. Prends le temps de reflechir a laraison pour laquelle cela est vrai.

Problemes : Sers-toi du fait ci-dessus pour trouver un moyen efficace de repondre a chacune desquestions suivantes.

1. Quelle est la somme des 99 premiers entiers positifs impairs consecutifs ?

2. Si 1225 est la somme des m premiers entiers positifs impairs consecutifs, quelle est la valeur dem ?

3. Quelle est la valeur de la somme 1 + 3 + 5 + · · · + 141 + 143 + 145 ?

4. Quelle est la valeur de la somme 17 + 19 + 21 + · · · + 207 + 209 + 211 ?

5. Quelle est la valeur de la somme 2 + 4 + 6 + · · · + 296 + 298 + 300 ?

Plus d’infos : Consulte la page du CEMI a la maison mardi, le 2 juin, pour les solutions a cesproblemes. La somme des n premiers nombres impairs consecutifs, 1 + 3 + 5 + . . . + (2n − 1), estun exemple d’une suite arithmetique de raison 2 et de premier terme 1. Consulte cette lecon dans ledidacticiel du CEMI pour en savoir plus a propos des suites arithmetiques.

Page 123: CEMI a la maison 9e et 10e ann ee - Lundi 23 mars 2020 ... · 19.Un nombre de Fibonacci. 20.Un multiple de 11. 22.Le plus petit nombre de la grille. 23.Le troisi eme c^ot e d’un

CEMC at Home

Grade 9/10 - Tuesday, May 26, 2020

Perfect Squares - Solution

1. What is the sum of the first 99 consecutive positive odd integers?

Solution: The sum of the first 99 consecutive positive odd integers is equal to 992 = 9801.

2. If 1225 is the sum of the first m consecutive positive odd integers, what is the value of m?

Solution: Since the sum of the first m consecutive positive odd integers is equal to m2 we musthave m2 = 1225. Since m is positive, m =

√1225 = 35.

3. What is the value of the sum 1 + 3 + 5 + · · ·+ 141 + 143 + 145?

Solution: This is the sum of the first n consecutive positive odd integers for some n. What isthe value of n (the number of terms in the sum)? We can write odd numbers in the form 2k−1where k is an integer, and so we rewrite this sum as

1 + 3 + · · ·+ 143 + 145 =(2(1)− 1

)+(2(2)− 1

)+ · · ·+

(2(72)− 1

)+(2(73)− 1

)Rewriting the sum in this way allows us to count that there are 73 terms in the sum, and son = 73. Since this sum is the sum of the first 73 consecutive positive odd integers, the summust be equal to 732 = 5329. (Note that n can be calculated as follows: n = 145+1

2= 73.)

4. What is the value of the sum 17 + 19 + 21 + · · ·+ 207 + 209 + 211?

Solution: First, we note that the given sum can be calculated as the following difference ofsums:

17+19+21+· · ·+207+209+211 = (1+3+5+. . .+207+209+211)−(1+3+5+. . .+11+13+15)

You can verify that the sum 1+3+5+ . . .+207+209+211 has 211+12

= 106 terms. Since this isthe sum of the first 106 consecutive positive odd integers, the value of the sum is 1062 = 11 236.

You can verify that the sum 1 + 3 + 5 + . . .+ 11 + 13 + 15 has 15+12

= 8 terms. Since this is thesum of the first 8 consecutive positive odd integers, the value of the sum is 82 = 64.

Therefore, 17 + 19 + 21 + · · ·+ 207 + 209 + 211 = 11 236− 64 = 11 172.

5. What is the value of the sum 2 + 4 + 6 + · · ·+ 296 + 298 + 300?

Solution: The terms in this sum are consecutive positive even integers. There are 3002

= 150terms in the sum. To create a sum of consecutive positive odd integers, we can rewrite eachterm as an odd number plus 1, and then collect all the extra 1s as follows:

2 + 4 + 6 + . . . + 296 + 298 + 300

= (1 + 1) + (3 + 1) + (5 + 1) + . . . + (295 + 1) + (297 + 1) + (299 + 1)

= 1 + 3 + 5 + . . . + 295 + 297 + 299 + (1 + 1 + 1 + . . . + 1)

= 1502 + 150

= 22 650

Page 124: CEMI a la maison 9e et 10e ann ee - Lundi 23 mars 2020 ... · 19.Un nombre de Fibonacci. 20.Un multiple de 11. 22.Le plus petit nombre de la grille. 23.Le troisi eme c^ot e d’un

CEMC.UWATERLOO.CA | le CENTRE d’ÉDUCATION en MATHÉMATIQUES et en INFORMATIQUE

Le CEMI a la maison

9e et 10e annee - le mercredi 27 mai 2020Des souris melees

Six souris numerotees se deplacent a travers le reseau de chemins ci-dessous afin d’atteindre le fromage.Pour commencer, les souris s’alignent au hasard sur le cote gauche du reseau. Ensuite, chaque sourisse deplace le long d’un chemin en suivant les fleches. Lorsqu’une souris atteint un cercle jaune, elleattend l’arrivee d’une autre souris. Lorsqu’une autre souris arrive au cercle, les deux souris comparentleurs nombres. La souris avec le plus petit nombre part du cercle en suivant la fleche pleine tandisque la souris avec le plus grand nombre part du cercle en suivant la fleche pointillee.

Questions

1. Identifie le fromage qu’atteindra chaque souris apres qu’elles se soient toutes deplacees a traversle reseau de chemins ci-dessus.

2. Aligne a nouveau les souris au debut du reseau dans un nouvel ordre. Identifie le fromagequ’atteindra chaque souris apres qu’elles se soient toutes deplacees a travers le reseau de cheminsci-dessus. Repete cela a quelques reprises avec un ordre de depart different a chaque fois. Queremarques-tu?

3. Que se passerait-il si, en quittant les cercles, les souris avec les plus petits nombres suivaientles fleches pointillees et les souris avec les plus grands nombres suivaient les fleches pleines?Explique.

Reflechis aux questions 1, 2 et 3 avant de passer a la page suivante.

Page 125: CEMI a la maison 9e et 10e ann ee - Lundi 23 mars 2020 ... · 19.Un nombre de Fibonacci. 20.Un multiple de 11. 22.Le plus petit nombre de la grille. 23.Le troisi eme c^ot e d’un

CEMC.UWATERLOO.CA | le CENTRE d’ÉDUCATION en MATHÉMATIQUES et en INFORMATIQUE

Le reseau presente sur la premiere page est un exemple d’un reseau de tri qui trie six nombres.Il y a six entrees du cote gauche du reseau et six sorties du cote droit du reseau. Chaquecercle jaune represente une comparaison entre deux entrees (c.-a-d. lequel des nombres est leplus grand?) et produit deux sorties comme le montrent les fleches. Au fur et a mesure que lesentrees se deplacent a travers le reseau, elles sont reorganisees (ou triees) en fonction de leursgrandeurs.

4. Suppose que tu recois le reseau de tri suivant pour trier cinq nombres. Ce reseau suit lesmemes regles que le reseau de la premiere page. Les entrees sur le cote gauche du reseau sontles nombres 4, 7, 12, 18 et 29, dans un certain ordre. En utilisant l’information ci-dessous quanta la maniere dont ces nombres se deplacent a travers le reseau, est-il possible de determinerl’ordre de depart des cinq nombres d’entrees?

5. Dessine un reseau de tri qui trie exactement quatre nombres.

Peux-tu dessiner deux reseaux de tri differents qui trient exactement quatre nombres?

Activite: Essaie de dessiner ton reseau de tri sur une allee avec de la craie de trottoir oumodelise-le d’une certaine maniere sur le plancher de ton salon (que pourrais-tu utiliser pourles cercles et les fleches?). Prend un jeu de 52 cartes et demande aux membres de ta famille dechoisir une carte parmi les cartes d’une seule enseigne et demande-leur de s’aligner au hasardau debut du reseau. Jouez quelques tours afin que tu puisses evaluer l’efficacite de ton reseaude tri.

Plus d’infos:

Consulte la page du CEMI a la maison mercredi, le 3 juin, pour la solution a ce probleme.

Page 126: CEMI a la maison 9e et 10e ann ee - Lundi 23 mars 2020 ... · 19.Un nombre de Fibonacci. 20.Un multiple de 11. 22.Le plus petit nombre de la grille. 23.Le troisi eme c^ot e d’un

CEMC at Home

Grade 9/10 - Wednesday, May 27, 2020

Mixed Up Mice - SolutionIn computer science, a sorting network sorts a fixed number of items into a specific order which ispredetermined. In this activity, you explored examples of sorting networks that sorted integers.

1. The solution below shows the path that each mouse travelled. Each yellow circle shows numbersthat are being compared, and the mouse with the smaller number follows the solid line out ofthe circle, while the mouse with the larger number follows the dotted line.

2. Regardless of how the numbers are arranged at the start, each time the mice reach the cheese,their numbers are sorted from smallest to largest, reading from top to bottom. This is not acoincidence as the above network is a sorting network for six numbers.

3. If instead the mouse with the smaller number follows the dotted line and the mouse with thelarger number follows the solid line out of the circle, then the mice would end up being sortedfrom largest to smallest (reading top to bottom).

Note: The numbers we have chosen for the mice are all distinct, and the network rules currentlydo not specify what to do if two mice have the same number. A computer cannot typicallyhandle this lack of clarity. Instead, it needs to be given clear instructions to cover all possiblescenarios. For example, for this sorting network, we could say that if two equal numbers arecompared, then the mouse that comes from above follows the solid line out of the circle, whilethe mouse that comes from below follows the dotted arrow of the circle. If our only goalis to produce a sorted list of numbers, then it does not matter how we choose to handle thecomparison of two equal numbers. However, if mice with equal numbers have other informationassociated with them, then how we make this choice could become more important.

1

Page 127: CEMI a la maison 9e et 10e ann ee - Lundi 23 mars 2020 ... · 19.Un nombre de Fibonacci. 20.Un multiple de 11. 22.Le plus petit nombre de la grille. 23.Le troisi eme c^ot e d’un

4. It is not possible to completely determine the original order of the five inputs. How much canwe determine about the order? There are a number of ways to approach this question. Onepossible approach is described below.

Since we are given the starting position for 18, we know that Circle 1 has to contain 18 andanother number. Next, we see that Circle 2 contains 12 and 18. From here, we can trace backto the start of the network and place the 12 in the third input from the top. Looking at Circle2 again, we know that 12 is the smaller number, so it would proceed to Circle 3. From Circle 4we can determine that the 7 traces back to Circle 5 as the 12 traces back to Circle 3. Similarly,looking at Circle 6 we conclude that the 4 traces back to Circle 3. Following the arrows evenfurther back to the start of the network, we determine that the 4 was the bottom input. Last,looking at Circle 7 we determine that 29 comes from Circle 5 as 18 traces back to Circle 2.We cannot determine the original order of inputs 7 and 29.

5. Below are two different sorting networks for four numbers. The first sorting network takesadvantage of *parallel processing while the second does not.

Sorting Network - Parallel ProcessingSome comparisons occur simultaneously

Sorting Network - Sequential ProcessingOnly one comparison occurs at a time

*When a sorting network is used, it can be possible to perform some comparisons simulta-neously, also known as parallel processing. This can speed up a sorting process overall. Forexample, in the sorting network from problem 1., the comparison between 51 and 13 occurs atthe same time as the comparison between 28 and 42 as well as the comparison between 96 and77. A common use of parallel processing with sorting networks is in the design of hardware.

2

Page 128: CEMI a la maison 9e et 10e ann ee - Lundi 23 mars 2020 ... · 19.Un nombre de Fibonacci. 20.Un multiple de 11. 22.Le plus petit nombre de la grille. 23.Le troisi eme c^ot e d’un

CEMC at Home

Grade 9/10 - Thursday, May 28, 2020

Repetition By Product - Solution

Problem:

A positive integer is to be placed in each box below.

Integers may be repeated, but the product of any four adjacent integers is always 120.

Determine all possible values for x.

2 4 x 3Solution:

In both Solution 1 and Solution 2, let a1 be the integer placed in the first box, a2 the integer placedin the second box, a4 the integer placed in the fourth box, and so on, as shown below.

a1 a2 a4 a5 a7 a8 a10 a11 a13 a142 4 x 3Solution 1

Consider boxes 3 to 6. Since the product of any four adjacent integers is 120, we have2× a4× a5× 4 = 120. Therefore, a4× a5 = 120

2×4= 15. Since a4 and a5 are positive integers, there are

four possibilities: a4 = 1 and a5 = 15, or a4 = 15 and a5 = 1, or a4 = 3 and a5 = 5, or a4 = 5 anda5 = 3.

In each of the four cases, we will have a7 = 2. We can see why by considering boxes 4 to 7. We havea4 × a5 × 4× a7 = 120, or 15× 4× a7 = 120, since a4 × a5 = 15. Therefore, a7 = 120

15×4= 2.

Case 1: a4 = 1 and a5 = 15

Consider boxes 5 to 8. We have a5× 4× a7× a8 = 120, or 15× 4× 2× a8 = 120, or a8 = 12015×4×2

= 1.

Next, consider boxes 6 to 9. We have 4× a7× a8× x = 120, or 4× 2× 1× x = 120, or x = 1204×2

= 15.Let’s check that x = 15 satisfies the only other condition in the problem that we have not yet used,that is a12 = 3.Consider boxes 9 to 12. If x = 15 and a12 = 3, then a10 × a11 = 120

15×3= 8

3. But a10 and a11 must

both be integers, so is not possible for a10 × a11 = 83. Therefore, it must not be possible for a4 = 1

and a5 = 15, and so we find that there is no solution for x in this case.

Case 2: a4 = 15 and a5 = 1

Consider boxes 5 to 8. We have a5 × 4× a7 × a8 = 120, or 1× 4× 2× a8 = 120, or a8 = 1204×2

= 15.

Next, consider boxes 6 to 9. We have 4× a7 × a8 × x = 120, or x = 1204×2×15

= 1.Let’s check that x = 1 satisfies the only other condition in the problem that we have not yet used,that is a12 = 3.Consider boxes 7 to 10. Since a7 = 2, a8 = 15 and x = 1, then a10 = 120

2×15×1= 4. Similarly,

a11 = 12015×1×4

= 2. Then we have x× a10 × a11 × a12 = 1× 4× 2× 3 = 24 6= 120. Therefore, it mustnot be possible for a4 = 15 and a5 = 1. There is no solution for x in this case.

1

Page 129: CEMI a la maison 9e et 10e ann ee - Lundi 23 mars 2020 ... · 19.Un nombre de Fibonacci. 20.Un multiple de 11. 22.Le plus petit nombre de la grille. 23.Le troisi eme c^ot e d’un

Case 3: a4 = 3 and a5 = 5

Consider boxes 5 to 8. We have a5 × 4× a7 × a8 = 120, or 5× 4× 2× a8 = 120, or a8 = 1205×4×2

= 3.

Next, consider boxes 6 to 9. We have 4× a7 × a8 × x = 120, or x = 1204×2×3

= 5.Let’s check that x = 5 satisfies the only other condition in the problem that we have not yet used,that is a12 = 3.Consider boxes 7 to 10. Since a7 = 2, a8 = 3 and x = 5, then a10 = 120

2×3×5= 4. Similarly,

a11 = 1203×5×4

= 2. Then we have x× a10 × a11 × a12 = 5× 4× 2× 3 = 120. Therefore, the conditionthat a12 = 3 is satisfied in the case where a4 = 3 and a5 = 5. If we continue to fill out the entries inthe boxes, we obtain the entries shown in the diagram below.

5 4 2 3 5 4 2 3 5 4 2 3 45

We see that x = 5 is a possible solution. However, is it the only solution? We have one final case tocheck.

Case 4: a4 = 5 and a5 = 3

Consider boxes 5 to 8. We have a5 × 4× a7 × a8 = 120, or 3× 4× 2× a8 = 120, or a8 = 1203×4×2

= 5.

Next, consider boxes 6 to 9. We have 4× a7 × a8 × x = 120, or x = 1204×2×5

= 3.Let’s check that x = 3 satisfies the only other condition in the problem that we have not yet used,that is a12 = 3.Consider boxes 9 to 12. If x = 3 and a12 = 3, then a10 × a11 = 120

3×3= 40

3. But a10 and a11 must both

be integers, so it is not possible for a10 × a11 = 403

. Therefore, it must not be possible for a4 = 5 anda5 = 3, and so we find that there is no solution for x in this case.

Therefore, the only possible value for x is x = 5.

Solution 2

You may have noticed a pattern for the ai’s in Solution 1. We will explore this pattern.

a1 a2 a4 a5 a7 a8 a10 a11 a13 a142 4 x 3

Since the product of any four integers is 120, a1a2a3a4 = a2a3a4a5 = 120. Since both sides aredivisible by a2a3a4, and each is a positive integer, then a1 = a5.Similarly, a2a3a4a5 = a3a4a5a6 = 120, and so a2 = a6.In general, anan+1an+2an+3 = an+1an+2an+3an+4, and so an = an+4.We can use this along with the given information to fill out the boxes as follows:

x 4 2 3 x 4 2 3 x 4 2 3 4x

Therefore, 4× 2× 3× x = 120 and so x = 1204×2×3

= 5.

2

Page 130: CEMI a la maison 9e et 10e ann ee - Lundi 23 mars 2020 ... · 19.Un nombre de Fibonacci. 20.Un multiple de 11. 22.Le plus petit nombre de la grille. 23.Le troisi eme c^ot e d’un

CEMC.UWATERLOO.CA | le CENTRE d’ÉDUCATION en MATHÉMATIQUES et en INFORMATIQUE

Le CEMI a la maison

9e et 10e annee - le jeudi 28 mai 2020

Repetition par produit

Un nombre entier positif doit etre incrit dans chacune des cases ci-dessous.

Les nombres peuvent etre repetes, mais le produit de quatre nombres contigus doit toujours etre egala 120.

Determine toutes les valeurs possibles de x.

2 4 x 3

Plus d’infos :

Consulte la page du CEMI a la maison vendredi, le 29 mai, pour la solution a Repetition par produit.

Cette ressource du CEMI a la maison est un probleme passe du Probleme de la semaine. Le Problemede la semaine est une ressource hebdomadaire gratuite que le CEMI met a la disposition des ensei-gnant(e)s, des parents et des eleves pendant l’annee scolaire. Les publications du Probleme de lasemaine sont terminees pour cette annee scolaire en cours et reprendront le 17 septembre 2020. Pourt’abonner et consulter les problemes passes et leurs solutions, visite :https://www.cemc.uwaterloo.ca/resources/potw-f.php

1

Page 131: CEMI a la maison 9e et 10e ann ee - Lundi 23 mars 2020 ... · 19.Un nombre de Fibonacci. 20.Un multiple de 11. 22.Le plus petit nombre de la grille. 23.Le troisi eme c^ot e d’un

CEMC.UWATERLOO.CA | le CENTRE d’ÉDUCATION en MATHÉMATIQUES et en INFORMATIQUE

Le CEMI a la maison

9e et 10e annee - le vendredi 29 mai 2020

Eclaboussons-nous !Des amis ont cree une nouvelle facon amusante de rester frais lors des chaudes journees d’ete : eclaterdes ballons d’eau !

Voici comment ils jouent :

— Les amis se tiennent tous en cercle. Un des amis se voit attribuer le numero 1(la position 1) puis, en allant dans le sens horaire autour du cercle, les autres amis se voientattribuer les numeros 2, 3, 4, etc. jusqu’a ce que chaque ami ait recu un numero.

— En commencant par la personne en position 1 et en se deplacant dans le sens horaire autourdu cercle, une personne sur deux parmi les personnes seches (soit la deuxieme personne) eclateun ballon d’eau au-dessus de sa tete. (On te fournit plus de details dans l’exemple ci-dessous.)

— La derniere personne a rester seche remporte la partie !

Exemple : Voici comment le jeu se deroule si le cercle contient six amis.

La premiere foisautour du cercle, les

amis 2, 4 et 6recoivent des ballons

d’eau.

Puisque l’ami 2 estdeja disqualife, l’ami3 recoit le prochain

ballon d’eau.

Puisque les amis 4 et6 sont deja

disqualifies, l’ami 1recoit le prochain

ballon d’eau. Doncl’ami 5 remporte la

partie !

Questions :

Pour chaque nombre d’amis ci-dessous, determine la position de l’ami qui remportera la partie.

1. Le jeu se joue avec 7 amis.

2. Le jeu se joue avec 16 amis.

3. Le jeu se joue avec 41 amis.

Defi : Peux-tu determiner la position de l’ami qui remportera la partie si la partie se joue avec namis, n etant un entier strictement positif superieur ou egal a 2 ?

Plus d’infos :

Consulte la page du CEMI a la maison vendredi, le 5 juin, pour les solutions a ces questions.

1

Page 132: CEMI a la maison 9e et 10e ann ee - Lundi 23 mars 2020 ... · 19.Un nombre de Fibonacci. 20.Un multiple de 11. 22.Le plus petit nombre de la grille. 23.Le troisi eme c^ot e d’un

CEMC at Home

Grade 9/10 - Wednesday, May 13, 2020

Circle Splash - Solution

1. When the game is played with 7 friends, the friend in position 7 will win (remain dry). Weillustrate this using the following images:

Start

The first time aroundthe circle, friends 2,4, and 6 get water

balloons.

The second timearound the circle,friends 1 and 5 get

water balloons.

The third timearound the circle,

friend 3 gets a waterballoon and so friend

7 wins!

2. When the game is played with 16 friends, the friend in position 1 will win (remain dry). Insteadof drawing pictures to illustrate the solution, we start with the sequence of numbers from 1 to16 and cross off the numbers when this position gets a water balloon.

Each time we travel through the numbers, we cross out every second number that is not alreadycrossed out. Each time we reach 16, we loop around to 1.

Start: 1, 2, 3, 4, 5, 6, 7, 8, 9, 10, 11, 12, 13, 14, 15, 16First pass: 1, �2, 3, �4, 5, �6, 7, �8, 9, ��10, 11, ��12, 13, ��14, 15, ��16Second pass: 1, �2, �3, �4, 5, �6, �7, �8, 9, ��10, ��11, ��12, 13, ��14, ��15, ��16Third pass: 1, �2, �3, �4, �5, �6, �7, �8, 9, ��10, ��11, ��12, ��13, ��14, ��15, ��16Fourth pass: 1, �2, �3, �4, �5, �6, �7, �8, �9, ��10, ��11, ��12, ��13, ��14, ��15, ��16

The numbers were crossed out in the following order: 2, 4, 6, 8, 10, 12, 14, 16, 3, 7, 11, 15, 5, 13, 9.

This leaves 1 at the end.

Can you see a pattern forming here?

3. When the game is played with 41 friends, the friend in position 19 will remain dry. This can bedetermined by simulating the game as was done in the solution to either question 1 or question2, although it will take longer in this case. After discussing the challenge problem, we willreturn to this answer.

Challenge: Can you determine the position of the friend that wins the game if the game is playedwith n friends, where n is a positive integer that is at least 2?

Solution:

When the game is played with n friends, a simulation will not work. We need to step back andanalyze the game more generally.

Page 133: CEMI a la maison 9e et 10e ann ee - Lundi 23 mars 2020 ... · 19.Un nombre de Fibonacci. 20.Un multiple de 11. 22.Le plus petit nombre de la grille. 23.Le troisi eme c^ot e d’un

Here are some observations:

• The position of the friend who remains dry will never be even since all even numbers areeliminated during the first pass around the circle.

• If a circle has an even number of friends in it, then after one pass, we will be on position 1again and the number of friends left dry in the circle will be half of what it was at the start.

This second observation above leads to some more observations:

• If, after one pass, the circle still has an even number of friends that are dry, then after passtwo, we will be on position 1 again and the number of friends left dry in the circle will be halfof what it was.

• If after every pass (except for the final one), the circle continues to have an even number offriends that are dry, eventually we will land on position 1 with only two friends left dry in thecircle. On the final pass, the friend in position 1 remains dry and the other friend gets wet.

• In order for the circle to have an even number of friends left dry after every pass (except forthe final one), the number of friends at the start, n, needs to be a power of 2.

• We can conclude then that if n is a power of 2, then the friend in position 1 will always remaindry. (Notice that our answer to question 2 agrees with this.)

What if n is not a power of 2? At some point during the game, eliminating people one at a time,the number of friends left dry in the circle will become a power of 2. For example, if we started with12 friends, then after four water balloons, the circle will be left with 8 = 23 people that are still dry.When this happens, whichever position is “next” in the circle acts like position 1 (in the case wheren is a power of 2) and will be the position that remains dry for the rest of the game.

Given n, how many people need to be eliminated so that the number of people remaining dry in thecircle is a power of 2? Rewrite n as a power of 2 plus m as follows:

n = 2k + m

where 2k is the largest power of 2 less than or equal to n and m is a non-negative integer. Thus,after eliminating m people, there will be 2k people remaining dry in the circle.

The position that remains dry for the rest of the game is the position in the circle that comesimmediately after the mth person that is eliminated. What position is this? Since on the first triparound the circle every second person is eliminated, and it must be the case that 2m < n, the mthperson to be eliminated is in position 2m. This means the “next” position is position 2m + 1.

Can you see why it must be the case that 2m < n and why this is important for our argument above?

We can conclude that if n is not a power of 2 and we rewrite n as n = 2k + m (as outlined earlier),then the person that remains dry is the person in position 2m + 1.

Note 1: If n is a power of 2 then we have m = 0 and hence 2m + 1 = 2(0) + 1 = 1. So this formulatells us the position that remains dry in the case where n is a power of 2 as well.

Note 2: This formula agrees with our earlier answers. In question 1, we have n = 7 which canbe rewritten as 7 = 4 + 3 = 22 + 3. This means m = 3 and so the position that remains dry is2m + 1 = 2(3) + 1 = 7. In question 2, we have n = 16 which can be rewritten as 16 = 24 + 0. Thismeans m = 0 and so 2m + 1 = 2(0) + 1 = 1. In question 3, we have n = 41 which can be rewrittenas n = 32 + 9 = 25 + 9. This means m = 9 and so 2m + 1 = 2(9) + 1 = 19.

Page 134: CEMI a la maison 9e et 10e ann ee - Lundi 23 mars 2020 ... · 19.Un nombre de Fibonacci. 20.Un multiple de 11. 22.Le plus petit nombre de la grille. 23.Le troisi eme c^ot e d’un

CEMC.UWATERLOO.CA | le CENTRE d’ÉDUCATION en MATHÉMATIQUES et en INFORMATIQUE

Le CEMI a la maison

9e et 10e annee - le lundi 1er juin 2020

Concours - jour 5

La ressource d’aujourd’hui presente une question des concours mathematiques 2020 du CEMI.

Concours Galois 2020, no 2

Dans la figure ci-dessous, on voit un prisme droit a base rectangulaire de longueur L, de largeur`, de hauteur h, et dont l’aire totale et le volume sont obtenus, respectivement, par les formulesA = 2L` + 2Lh + 2`h et V = L`h.

l

h

L

(a) Un prisme droit a base rectangulaire a une longueur de 2 cm, une largeur de5 cm et une hauteur de 9 cm. Quelle est son aire totale ?

(b) Un prisme droit a une hauteur de 10 cm et une base carree. Le prisme a un volume de 160cm3. Quelle est la longueur des cotes de la base carree ?

(c) Un prisme droit a une base carree dont l’aire est egale a 36 cm2. Le prisme a une aire totalede 240 cm2. Determiner le volume du prisme.

(d) Un prisme droit a base rectangulaire a une longueur de k cm, une largeur de 2k cm et unehauteur de 3k cm, k ayant une valeur superieure a 0. Le prisme a un volume de x cm3 et uneaire totale de x cm2. Determiner la valeur de k.

Plus d’infos :

Consulte la page internet du CEMI a la maison lundi, le 8 juin, pour la solution au probleme deConcours Jour 5.

Page 135: CEMI a la maison 9e et 10e ann ee - Lundi 23 mars 2020 ... · 19.Un nombre de Fibonacci. 20.Un multiple de 11. 22.Le plus petit nombre de la grille. 23.Le troisi eme c^ot e d’un

CEMC.UWATERLOO.CA | le CENTRE d’ÉDUCATION en MATHÉMATIQUES et en INFORMATIQUE

Le CEMI a la maison

9e et 10e annee - le lundi 1er juin 2020

Concours, jour 5 - solutions

Voici la solution au probleme du concours

Concours Galois 2020, no 2

Dans la figure ci-dessous, on voit un prisme droit a base rectangulairede longueur L, de largeur `, de hauteur h, et dont l’aire totale et levolume sont obtenus, respectivement, par les formulesA = 2L` + 2Lh + 2`h et V = L`h.

l

h

L

(a) Un prisme droit a base rectangulaire a une longueur de 2 cm, une largeur de5 cm et une hauteur de 9 cm. Quelle est son aire totale ?

(b) Un prisme droit a une hauteur de 10 cm et une base carree. Le prisme a un volume de 160cm3. Quelle est la longueur des cotes de la base carree ?

(c) Un prisme droit a une base carree dont l’aire est egale a 36 cm2. Le prisme a une aire totalede 240 cm2. Determiner le volume du prisme.

(d) Un prisme droit a base rectangulaire a une longueur de k cm, une largeur de 2k cm et unehauteur de 3k cm, k ayant une valeur superieure a 0. Le prisme a un volume de x cm3 et uneaire totale de x cm2. Determiner la valeur de k.

Solution :

(a) Etant donne que l’aire totale d’un prisme droit a base rectangulaire est obtenue par la formuleA = 2L` + 2Lh + 2`h, alors un prisme droit a base rectangulaire dont la longueur, la largeuret la hauteur sont, respectivement, 2 cm, 5 cm et 9 cm a une aire totale de2(2)(5) + 2(2)(9) + 2(5)(9) = 20 + 36 + 90 = 146 cm2.

(b) Le volume d’un prisme droit a base rectangulaire est obtenu par la formule V = L`h.Si le prisme droit a une base carree, alors L = `, d’ou V = L2h.On pose V = 160 cm3 et h = 10 cm dans l’equation. On a donc 160 = L2(10) ou L2 = 16,d’ou L = 4 cm (puisque L > 0).Donc, un prisme droit a base carree dont la hauteur et le volume sont, respectivement,10 cm et 160 cm3 a une base dont les cotes ont une longueur de 4 cm.

(c) Si le prisme droit a une base carree, alors L = `.Puisque la base a une aire de 36 cm2, alors 36 = L · ` = L2, d’ou L = ` =

√36 = 6 cm

(puisque L > 0).Puisque le prisme a une aire totale de 240 cm2, alors on a

240 = 2(6)(6) + 2(6)h + 2(6)h ou 240 = 72 + 24h, d’ou h =240− 72

24= 7 cm.

Donc, le volume du prisme est egal a L`h = (6)(6)(7) = 252 cm3.

Voir la page suivante pour la solution de la partie (d).

Page 136: CEMI a la maison 9e et 10e ann ee - Lundi 23 mars 2020 ... · 19.Un nombre de Fibonacci. 20.Un multiple de 11. 22.Le plus petit nombre de la grille. 23.Le troisi eme c^ot e d’un

CEMC.UWATERLOO.CA | le CENTRE d’ÉDUCATION en MATHÉMATIQUES et en INFORMATIQUE

(d) Selon les dimensions du prisme representees en fonction de k, on peut representer le volumedu prisme par la formule x = k(2k)(3k) ou x = 6k3. De meme, on peut representer l’aire totaledu prisme par la formule x = 2(k)(2k) + 2(k)(3k) + 2(2k)(3k) oux = 4k2 + 6k2 + 12k2 = 22k2.On reporte x = 6k3 dans x = 22k2 pour obtenir :

6k3 = 22k2

6k3 − 22k2 = 0

2k2(3k − 11) = 0

Puisque k > 0, alors 3k − 11 = 0, d’ou k =11

3.

Page 137: CEMI a la maison 9e et 10e ann ee - Lundi 23 mars 2020 ... · 19.Un nombre de Fibonacci. 20.Un multiple de 11. 22.Le plus petit nombre de la grille. 23.Le troisi eme c^ot e d’un

CEMC.UWATERLOO.CA | le CENTRE d’ÉDUCATION en MATHÉMATIQUES et en INFORMATIQUE

Le CEMI a la maison

9e et 10e annee - le mardi 2 juin 2020

Des mathematiciennes et des mathematiciens celebres

Tout au long de l’histoire de l’humanite, plusieurs mathematiciennes et mathematiciens ont ap-porte des contributions importantes a ce domaine. Ces personnages historiques importants ont sou-vent mene des vies fascinantes remplies d’histoires interessantes. Cinq de ces figures eminentes sontpresentees ci-dessous.

Al-KhwarizmıC’etait un mathematicien du 9e siecle habitant la ville de Bagdad ouil tenait le poste de chef de la bibliotheque surnommee Maison de laSagesse. Le mot � algebre � nous provient du titre de l’un de ses livres.

Blaise PascalC’etait un mathematicien francais du 17e siecle dont les travaux ont jeteles bases de la theorie moderne des probabilites. Il a egalement beaucoupcontribue aux domaines de la physique et de la religion.

William Tutte

Il est ne en Angleterre et a ete un dechiffreur de codes important pen-dant la Seconde Guerre mondiale. En 1962, il a commence a travailler al’Universite de Waterloo ou son travail a beaucoup contribue a la theoriedes graphes.

Grigori Perelman

C’est un mathematicien russe contemporain a qui on a decerne la medailleFields en 2006 (qu’il a par contre refusee). Son travail en geometrie estimportant pour la conjecture de Poincare, un resultat qui a mene a unemeilleure comprehension de la topologie.

Maryam MirzakhaniElle a ete la premiere femme a recevoir la prestigieuse medaille Fields.Elle est nee en Iran et a ensuite etudie et travaille aux Etats-Unis avantqu’elle ne s’eteigne du cancer du sein en 2017.

Choisis deux des cinq figures eminentes et pour chacune :

1. Cherche des informations sur lui ou elle en ligne afin de trouver un fait nouveau interessant ason sujet.

2. Essaye de trouver un lien entre quelque chose que tu as etudie dans un cours de mathematiquesrecent et le travail mathematique de cette figure historique.

3. Si tu avais la chance de remonter dans le temps et de rencontrer ce mathematicien ou cettemathematicienne, quelles questions lui poserais-tu ?

Plus d’infos : Le Concours de mathematiques Pascal du CEMI porte le nom de Blaise Pascal enson honneur.

Page 138: CEMI a la maison 9e et 10e ann ee - Lundi 23 mars 2020 ... · 19.Un nombre de Fibonacci. 20.Un multiple de 11. 22.Le plus petit nombre de la grille. 23.Le troisi eme c^ot e d’un

CEMC.UWATERLOO.CA | le CENTRE d’ÉDUCATION en MATHÉMATIQUES et en INFORMATIQUE

Le CEMI a la maison

9e et 10e annee - le mercredi 3 juin 2020

Interagir avec les mathematiques

La technologie peut nous aider a faire des decouvertes mathematiques et a en apprendre davantage surles objets mathematiques. On vous presente ci-dessous trois exemples en ligne de telles technologiesdans differents domaines des mathematiques.

Des valeurs lineaires inconnues : Sers-toi des proprietes des relations lineaires pour trouverla combinaison d’un cadenas.

Lien vers l’application : https://www.geogebra.org/m/f833pgnu

Representer une histoire graphiquement : Une voiture roule sur une route. Identifie legraphique qui decrit le trajet de la voiture.

Lien vers l’application : https://www.geogebra.org/m/xbcrh6we

Esquisser un cercle : Essaye de dessiner un cercle a main levee.

Lien vers l’application : https://www.geogebra.org/m/trawnuex

Plus d’infos : Les lecons du didacticiel du CEMI contiennent des centaines d’applications mathematiquesinteractives. Pour le didacticiel du CEMI des niveaux 9/10/11, une bibliotheque interactive a eteconstruite qui vous permet d’effectuer une recherche par mot cle et/ou d’afficher uniquement lesapplications d’un certain domaine, d’une certaine unite ou d’une certaine lecon.

Page 139: CEMI a la maison 9e et 10e ann ee - Lundi 23 mars 2020 ... · 19.Un nombre de Fibonacci. 20.Un multiple de 11. 22.Le plus petit nombre de la grille. 23.Le troisi eme c^ot e d’un

CEMC.UWATERLOO.CA | le CENTRE d’ÉDUCATION en MATHÉMATIQUES et en INFORMATIQUE

Le CEMI a la maison

9e et 10e annee - le jeudi 4 juin 2020

Maximiser l’aire

Les rectangles ABJH et JDEF , dont la longueur de chacun de leurs cotes est un nombre entier, setouchent au coin J de facon a ce que le segment de droite HJD soit perpendiculaire au segment dedroite BJF . Les deux rectangles sont inclus dans un plus grand rectangle ACEG, comme indiquedans le diagramme ci-dessous.

L’aire du rectangle ABJH est de 6 cm2 et l’aire du rectangle JDEF est de 15 cm2.

Determine la plus grande aire possible du rectangle ACEG. Il est a noter que le diagramme n’estpas a l’echelle.

A B C

D

EFG

HJ

Plus d’infos :

Consulte la page du CEMI a la maison vendredi, le 5 juin, pour la solution a Maximiser l’aire.

Cette ressource du CEMI a la maison est un probleme passe du Probleme de la semaine. Le Problemede la semaine est une ressource hebdomadaire gratuite que le CEMI met a la disposition des ensei-gnant(e)s, des parents et des eleves pendant l’annee scolaire. Les publications du Probleme de lasemaine sont terminees pour cette annee scolaire en cours et reprendront le 17 septembre 2020. Pourt’abonner et consulter les problemes passes et leurs solutions, visite :https://www.cemc.uwaterloo.ca/resources/potw-f.php

1

Page 140: CEMI a la maison 9e et 10e ann ee - Lundi 23 mars 2020 ... · 19.Un nombre de Fibonacci. 20.Un multiple de 11. 22.Le plus petit nombre de la grille. 23.Le troisi eme c^ot e d’un

CEMC at Home

Grade 9/10 - Thursday, June 4, 2020

Maximize the Area - Solution

Problem:

Two rectangles, ABJH and JDEF , with integer side lengths, share a common corner at J suchthat HJD and BJF are perpendicular line segments. The two rectangles are enclosed by a largerrectangle ACEG, as shown.

The area of rectangle ABJH is 6 cm2 and the area of rectangle JDEF is 15 cm2.

Determine the largest possible area of the rectangle ACEG. Note that the diagram is not intendedto be to scale.

A B C

D

EFG

HJ

Solution:

Let AB = x, AH = y, JD = a and JF = b.

Then,

AB = HJ = GF = x,

AH = BJ = CD = y,

BC = JD = FE = a, and

HG = JF = DE = b.

A B C

D

EFG

H J

x

a

b

y

Also,area(ACEG) = area(ABJH) + area(BCDJ) + area(JDEF ) + area(HJFG)

= 6 + ya + 15 + xb

= 21 + ya + xb

Since the area of rectangle ABJH is 6 cm2 and the side lengths of ABJH are integers, then the sidelengths must be 1 and 6 or 2 and 3. That is, x = 1 cm and y = 6 cm, x = 6 cm and y = 1 cm, x = 2cm and y = 3 cm, or x = 3 cm and y = 2 cm.

Since the area of rectangle JDEF is 15 cm2 and the side lengths of JDEF are integers, then theside lengths must be 1 and 15 or 3 and 5. That is, a = 1 cm and b = 15 cm, a = 15 cm and b = 1cm, a = 3 cm and b = 5 cm, or a = 5 cm and b = 3 cm.

1

Page 141: CEMI a la maison 9e et 10e ann ee - Lundi 23 mars 2020 ... · 19.Un nombre de Fibonacci. 20.Un multiple de 11. 22.Le plus petit nombre de la grille. 23.Le troisi eme c^ot e d’un

To maximize the area, we need to pick the values of x, y, a, b which make ya+xb as large as possible.We will now break into cases based on the possible side lengths of ABJH and JDEF and calculatethe area of ACEG in each case. We do not need to try all 16 possible pairings, because trying x = 1cm and y = 6 cm with the four possibilities of a and b will give the same 4 areas, in some order, astrying x = 6 cm and y = 1 cm with the four possibilities of a and b. Similarly, trying x = 2 cm andy = 3 cm with the four possibilities of a and b will give the same 4 areas, in some order, as tryingx = 3 cm and y = 2 cm with the four possibilities of a and b. (As an extension, we will leave it toyou to think about why this is the case.)

Case 1: x = 1 cm, y = 6 cm and a = 1 cm, b = 15 cm

area(ACEG) = 21 + ya + xb = 21 + 6(1) + 1(15) = 42 cm2

Case 2: x = 1 cm, y = 6 cm and a = 15 cm, b = 1 cm

area(ACEG) = 21 + ya + xb = 21 + 6(15) + 1(1) = 112 cm2

Case 3: x = 1 cm, y = 6 cm and a = 3 cm, b = 5 cm

area(ACEG) = 21 + ya + xb = 21 + 6(3) + 1(5) = 44 cm2

Case 4: x = 1 cm, y = 6 cm and a = 5 cm, b = 3 cm

area(ACEG) = 21 + ya + xb = 21 + 6(5) + 1(3) = 54 cm2

Case 5: x = 2 cm, y = 3 cm and a = 1, b = 15 cm

area(ACEG) = 21 + ya + xb = 21 + 3(1) + 2(15) = 54 cm2

Case 6: x = 2 cm, y = 3 cm and a = 15, b = 1 cm

area(ACEG) = 21 + ya + xb = 21 + 3(15) + 2(1) = 68 cm2

Case 7: x = 2 cm, y = 3 cm and a = 3, b = 5 cm

area(ACEG) = 21 + ya + xb = 21 + 3(3) + 2(5) = 40 cm2

Case 8: x = 2 cm, y = 3 cm and a = 5, b = 3 cm

area(ACEG) = 21 + ya + xb = 21 + 3(5) + 2(3) = 42 cm2

We see that the maximum area is 112 cm2, and occurs when x = 1 cm, y = 6 cm and a = 15 cm,b = 1 cm. It will also occur when x = 6 cm, y = 1 cm and a = 1 cm, b = 15 cm.

The following diagrams show the calculated values placed on the original diagram. The diagram wasdefinitely not drawn to scale! Both solutions produce rectangles with dimensions 7 cm by 16 cm, andarea 112 cm2.

A B C

D

EFG

HJ

1 cm

6 cm

1 cm

15 cm

A B C

D

EFG

H J

x

a

b

y

A B C

D

EFG

HJ

6 cm

1 cm

15 cm

1 cm

2

Page 142: CEMI a la maison 9e et 10e ann ee - Lundi 23 mars 2020 ... · 19.Un nombre de Fibonacci. 20.Un multiple de 11. 22.Le plus petit nombre de la grille. 23.Le troisi eme c^ot e d’un

CEMC.UWATERLOO.CA | le CENTRE d’ÉDUCATION en MATHÉMATIQUES et en INFORMATIQUE

Le CEMI a la maison

9e et 10e annee - le vendredi 5 juin 2020

Les mathematiques et l’informatique dans les nouvelles

La plupart des semaines, la page d’accueil du CEMI fournit un lien vers une histoire dans les mediasa propos des mathematiques et/ou de l’informatique. Ces histoires nous montrent a quel point lesmathematiques et l’informatique sont importantes dans le monde d’aujourd’hui. Ces histoires sontd’excellentes sources de discussions.

En utilisant cet article de CBC News, pense aux questions suivantes. (L’adresse URL est egalementfournie ci-dessous.)

1. Selon toi, qu’est-ce que quelqu’un veut dire quand il dit que les humains sont plus intelligentsque les autres membres du regne animal ?

2. Qu’est-ce que l’intelligence artificielle ? Peux-tu decrire deux manieres dont tu te sers de l’in-telligence artificielle ?

3. Selon toi, quels sont les avantages et les desavantages de l’intelligence artificielle ?

4. Predire l’avenir : comment l’intelligence artificielle evoluera-t-elle au cours des 20 prochainesannees ?

L’adresse URL de l’article :https ://www.cbc.ca/news/technology/artificial-intelligence-human-brain-to-merge-in-2030s-says-futurist-kurzweil-1.3100124

Plus d’infos :

Une archive complete des articles precedents peut etre trouvee dans nos archives Les Maths etl’informatique dans les nouvelles. Des ressources similaires d’autres niveaux pourraient t’interesser.

1

Page 143: CEMI a la maison 9e et 10e ann ee - Lundi 23 mars 2020 ... · 19.Un nombre de Fibonacci. 20.Un multiple de 11. 22.Le plus petit nombre de la grille. 23.Le troisi eme c^ot e d’un

CEMC.UWATERLOO.CA | le CENTRE d’ÉDUCATION en MATHÉMATIQUES et en INFORMATIQUE

Le CEMI a la maison

9e et 10e annee - le lundi 8 juin 2020

Concours - jour 6

La ressource d’aujourd’hui presente une question des concours mathematiques 2020 du CEMI.

Concours Fryer 2020, no 3

Une suite Dlin est une suite dont le premier terme est un entier strictement positif et dont chaqueterme, apres le premier, est obtenu en ajoutant 1 au terme precedent et en doublant ce resultat. Parexemple, les sept premiers termes d’une suite Dlin de premier terme 4 sont :

4, 10, 22, 46, 94, 190, 382

(a) Le 5e terme d’une suite Dlin est 142. Quels sont les 4e et 6e termes de cette suite ?

(b) Determiner tous les premiers termes possibles de suites Dlin dans lesquelles 1406 paraıt commeterme.

(c) Lesquels des entiers strictement positifs, de 10 a 19, pourraient etre les premiers termes d’unesuite Dlin dont tous les termes, apres le premier, ont le meme chiffre des unites ?

(d) Combien des entiers strictement positifs, de 1 a 2020, pourraient etre le troisieme terme dansune suite Dlin ?

Plus d’infos :

Consulte la page internet du CEMI a la maison lundi, le 15 juin, pour la solution au probleme deConcours Jour 6.

Page 144: CEMI a la maison 9e et 10e ann ee - Lundi 23 mars 2020 ... · 19.Un nombre de Fibonacci. 20.Un multiple de 11. 22.Le plus petit nombre de la grille. 23.Le troisi eme c^ot e d’un

CEMC.UWATERLOO.CA | le CENTRE d’ÉDUCATION en MATHÉMATIQUES et en INFORMATIQUE

Le CEMI a la maison

9e et 10e annee - le lundi 8 juin 2020

Concours, Jour 6 - solutions

Voici la solution au probleme de concours.

Concours Fryer 2020, no 3

Une suite Dlin est une suite dont le premier terme est un entier strictement positif et dont chaqueterme, apres le premier, est obtenu en ajoutant 1 au terme precedent et en doublant ce resultat. Parexemple, les sept premiers termes d’une suite Dlin de premier terme 4 sont :

4, 10, 22, 46, 94, 190, 382

(a) Le 5e terme d’une suite Dlin est 142. Quels sont les 4e et 6e termes de cette suite ?

(b) Determiner tous les premiers termes possibles de suites Dlin dans lesquelles 1406 paraıt commeterme.

(c) Lesquels des entiers strictement positifs, de 10 a 19, pourraient etre les premiers termes d’unesuite Dlin dont tous les termes, apres le premier, ont le meme chiffre des unites ?

(d) Combien des entiers strictement positifs, de 1 a 2020, pourraient etre le troisieme terme dansune suite Dlin ?

Solution :

(a) Si le 5e terme d’une suite Dlin est 142, alors le 6e terme est (142 + 1)× 2 = 143× 2 = 286.

Puisqu’on obtient chaque terme apres le premier en ajoutant 1 au terme precedent et endoublant ce resultat alors, ayant le 5e terme de la suite, on peut faire � marche arriere � endivisant d’abord le 5e terme par 2 puis en soustrayant 1 de ce resultat afin d’obtenir le 4e

terme de la suite.Pour le voir, considerons deux termes consecutifs dans une suite Dlin, soit a suivi de b, quiverifient donc b = (a + 1)× 2.Pour determiner les operations necessaires pour obtenir a etant donne b (c’est-a-dire pourreculer dans la suite), on isole a dans l’equation afin d’exprimer ce dernier en fonction de b :

b = (a + 1)× 2b2

= a + 1b2− 1 = a

Donc, si le 5e terme de la suite Dlin est 142, alors le 4e terme est 1422− 1 = 71− 1 = 70.

(On peut verifier que 142 est bien le terme apres 70 : (70 + 1)× 2 = 142.)

(b) Si le 1er terme de la suite est 1406, alors on a clairement une suite Dlin dans laquelle 1406parait comme terme.Si le 2e terme d’une suite Dlin est 1406, alors le 1er terme de la suite est14062− 1 = 703− 1 = 702.

Si le 3e terme d’une suite Dlin est 1406, alors le 2e terme est 702 (ce qu’on a calcule dans laligne precedente) et le 1er terme de la suite est 702

2− 1 = 351− 1 = 350.

Si le 4e terme d’une suite Dlin est 1406, alors le 3e terme est 702, le 2e terme est 350 et le 1er

terme de la suite est 3502− 1 = 175− 1 = 174.

Page 145: CEMI a la maison 9e et 10e ann ee - Lundi 23 mars 2020 ... · 19.Un nombre de Fibonacci. 20.Un multiple de 11. 22.Le plus petit nombre de la grille. 23.Le troisi eme c^ot e d’un

CEMC.UWATERLOO.CA | le CENTRE d’ÉDUCATION en MATHÉMATIQUES et en INFORMATIQUE

A ce point, on voit que 174, 350, 702 et 1406 sont des 1er termes possibles de suites Dlin danslesquelles 1406 parait comme terme.On peut continuer ce processus de � marche arriere � (en divisant par 2 puis en soustrayant 1du resultat) pour determiner tous les 1er termes possibles de suites Dlin dans lesquelles 1406parait comme terme.

1406→ 702→ 350→ 174→ 1742− 1 = 86→ 86

2− 1 = 42→ 42

2− 1 = 20→ 20

2− 1 = 9

Si l’on tente de poursuivre ce processus au-dela de 9, on obtient 92− 1 = 7

2. Or, ce dernier

n’est pas possible car le 1er terme d’une suite Dlin doit etre un entier strictement positif (d’ouchacun des termes de la suite est donc un entier strictement positif).Donc, 9, 20, 42, 86, 174, 350, 702 et 1406 sont les 1er termes possibles de suites Dlin danslesquelles 1406 parait comme terme.

(c) Chacun des entiers de 10 a 19 peut etre le premier terme d’une suite Dlin. Donc, pour chacunde ces dix premiers termes possibles, on doit determiner le chiffre des unites des termes quisuivent.Si le 1er terme de la suite est 10, alors le 2e terme, (10 + 1) × 2 = 22, a 2 comme chiffre desunites tandis que le 3e terme, (22 + 1)× 2 = 46, a 6 comme chiffre des unites.Si le 1er terme de la suite est 11, alors le 2e terme, (11 + 1) × 2 = 24, a 4 comme chiffre desunites tandis que le 3e terme, (24 + 1)× 2 = 50, a 0 comme chiffre des unites.On dresse la liste des chiffres des unites des 2e et 3e termes pour chacun des premiers termespossibles dans le tableau ci-dessous :

1er terme 10 11 12 13 14 15 16 17 18 19Chiffre des unites du 2e terme 2 4 6 8 0 2 4 6 8 0Chiffre des unites du 3e terme 6 0 4 8 2 6 0 4 8 2

D’apres le tableau ci-dessus, on voit que 8 est le seul chiffre des unites que les 2e et 3e termespeuvent avoir en commun.Donc, si le 1er terme de la suite est 18 (ayant donc un 8 comme chiffre des unites), alors les2e et 3e termes de la suite ont 8 comme chiffre des unites. Donc, tous les termes ont le memechiffre des unites.De meme, si le 1er terme de la suite est 13 (ayant donc un 3 comme chiffre des unites), alorsles 2e et 3e termes de la suite ont 8 comme chiffre des unites.Il s’ensuit donc que tous les autres termes apres le premier auront 8 comme chiffre des unites.Parmi les entiers strictement positifs de 10 a 19, 13 et 18 sont ceux qui pourraient etre lespremiers termes d’une suite Dlin dont tous les termes, apres le premier, ont le meme chiffredes unites.

(d) Si le 1er terme d’une suite Dlin est x, alors le 2e terme est (x + 1)× 2 = 2x + 2 tandis que le3e terme est (2x + 2 + 1)× 2 = (2x + 3)× 2 = 4x + 6.Par exemple, si x = 1 (remarquons qu’il s’agit du plus petit 1er terme possible d’une suiteDlin), alors le 3e terme est 4×1+6 = 10. De plus, si x = 2, alors le 3e terme est 4×2+6 = 14.Quelle est la plus grande valeur possible de x (le 1er terme de la suite) telle que 4x + 6 (le 3e

terme de la suite) soit inferieur ou egal a 2020 ?En posant 4x + 6 = 2020 et en resolvant, on obtient 4x = 2014, d’ou x = 503, 5.Puisque le 1er terme de la suite doit etre un entier strictement positif, on ne peut avoir 2020comme 3e terme.De meme, en posant 4x + 6 = 2019 et en resolvant, on obtient une valeur non entiere de x.Donc, on ne peut avoir 2019 comme 3e terme d’une suite Dlin.

Page 146: CEMI a la maison 9e et 10e ann ee - Lundi 23 mars 2020 ... · 19.Un nombre de Fibonacci. 20.Un multiple de 11. 22.Le plus petit nombre de la grille. 23.Le troisi eme c^ot e d’un

CEMC.UWATERLOO.CA | le CENTRE d’ÉDUCATION en MATHÉMATIQUES et en INFORMATIQUE

Lorsque 4x + 6 = 2018, on obtient 4x = 2012, d’ou x = 503.Donc, si le 1er terme d’une suite Dlin est 503, alors le 3e terme de la suite, soit 2018, est unentier strictement positif entre 1 et 2020.De plus, 503 est le plus grand 1er terme possible tel que le 3e terme soit un entier strictementpositif entre 1 et 2020.Dans les suites Dlin, on obtient un 3e terme (4x + 6) different pour chaque premier termedistinct (x).Donc, afin de determiner combien des entiers strictement positifs de 1 a 2020 pourraient etrele 3e terme dans une suite Dlin, on determine le nombre de 1er termes tels que le 3e terme aitcette propriete.Le plus petit 1er terme possible est 1 (d’ou on a 10 comme 3e terme) et le plus grand 1er termepossible est 503 (d’ou on a 2018 comme 3e terme).De plus, chaque valeur de x entre 1 et 503 produit un 3e terme ayant une valeur entre 10 et2018.Donc, il y a 503 entiers strictement positifs, de 1 a 2020, qui pourraient etre le 3e terme dansune suite Dlin.

Page 147: CEMI a la maison 9e et 10e ann ee - Lundi 23 mars 2020 ... · 19.Un nombre de Fibonacci. 20.Un multiple de 11. 22.Le plus petit nombre de la grille. 23.Le troisi eme c^ot e d’un

CEMC.UWATERLOO.CA | le CENTRE d’ÉDUCATION en MATHÉMATIQUES et en INFORMATIQUE

Le CEMI a la maison

9e et 10e annee - le mardi 9 juin 2020

Des triplets pythagoriciens

Le theoreme de Pythagore : Dans un triangle rectangle ou crepresente la longueur de l’hypotenuse et a et b representent leslongueurs des deux cathetes, l’equation suivante est verifiee :

a2 + b2 = c2

Il convient de noter que tout triangle dont les longueurs des cotesa, b et c verifient l’equation a2 + b2 = c2 doit etre un trianglerectangle.

Un triplet pythagoricien est un triplet d’entiers (a, b, c) verifiant a2 + b2 = c2.

Si un triangle est forme de maniere que les longueurs des cotes a, b et c soient des entiers, alors cetriangle est un triangle rectangle uniquement si (a, b, c) est un triplet pythagoricien.

Probleme 1 : Le triplet (3, 4, 5) est un triplet pythagoricien. Si a = 3, b = 4 et c = 5, alors

a2 + b2 = 32 + 42 = 9 + 16 = 25 et c2 = 52 = 25.

On a donc a2 + b2 = c2.

Demontre que (5, 12, 13) et (7, 24, 25) sont egalement des triplets pythagoriciens.

4

35

12

513

24

725

Est-ce que chaque entier strictement positif fait partie d’au moins un triplet pythagoricien ? Il s’avereque la reponse a cette question est non. Cependant, chaque entier strictement positif superieur ou egal a 3fait partie d’un triplet pythagoricien. Explorons cette idee.

Probleme 2 : Considere les triplets pythagoriciens du Probleme 1 : (3, 4, 5), (5, 12, 13) et (7, 24, 25).Note que les entiers dans les coordonnees les plus a gauche des triplets sont les entiers impairs 3, 5et 7. Remarques-tu une regularite dans les deux autres entiers de chaque triplet ? Les deux entiersrestants dans chaque triplet sont des entiers consecutifs : 4 et 5, 12 et 13, 24 et 25. Explorons cetteregularite.

(a) Construis un triplet pythagoricien comprenant l’entier impair 9 en suivant ces etapes :

(i) Determine la valeur de n telle que 92 = 2n + 1. (Reponse : n = 81−12

= 40.)

(ii) Verifie que (n + 1)2 − n2 = 92. (Reponse : 412 − 402 = 1681 − 1600 = 81 = 92.)

(iii) Ecris un triplet pythagoricien dont le plus petit entier est 9. (Reponse : Puisque 412−402 =92, on a 92 + 402 = 412. Donc (9, 40, 41) est un triplet pythagoricien.)

Page 148: CEMI a la maison 9e et 10e ann ee - Lundi 23 mars 2020 ... · 19.Un nombre de Fibonacci. 20.Un multiple de 11. 22.Le plus petit nombre de la grille. 23.Le troisi eme c^ot e d’un

(b) Construis un triplet pythagoricien comprenant l’entier impair 11 en suivant ces etapes :

(i) Determine la valeur de n telle que 112 = 2n + 1.

(ii) Verifie que (n + 1)2 − n2 = 112.

(iii) Ecris un triplet pythagoricien dont le plus petit entier est 11.

(c) Utilise les idees dans (a) et (b) pour construire des triplets pythagoriciens qui incluent lesquatre prochains entiers impairs : 13, 15, 17, 19.

Peux-tu voir comment faire cela pour tout entier impair ? On explore cela dans le problemedefi.

Probleme 3 :

(a) Considere le triplet pythagoricien (3, 4, 5). Demontre qu’on obtient un autre triplet pythago-ricien en multipliant chacun des entiers du triplet par 2.

(b) Utilise l’idee dans (a) pour construire un autre triplet pythagoricien comprenant l’entierimpair 9.

(c) Demontre que le triplet (3n, 4n, 5n) est un triplet pythagoricien pour tout entier strictementpositif n.Note que (5n, 12n, 13n) et (7n, 24n, 25n) sont egalement des triplets pythagoriciens.

(d) Utilise les idees des Problemes 2 et 3 pour demontrer que tout entier de 4 a 20 fait partied’au moins un triplet pythagoricien.

Probleme defi : Reflechis a la facon dont tu pourrais utiliser certaines de ces idees pour demontrerque chaque entier superieur ou egal a 3 fait partie d’un triplet pythagoricien. Une approche possibleest decrite ci-dessous mais il en existe d’autres :

(a) Nombres impairs :

(i) Demontrer qu’on a (n + 1)2 − n2 = 2n + 1 pour tout entier strictement positif n.

(ii) Utiliser l’identite de la partie (i) pour expliquer pourquoi chaque entier impair superieurou egal a 3 fait partie d’un triplet pythagoricien.

(b) Nombres pairs :

(i) Demontrer qu’on a (n + 2)2 − n2 = 4n + 4 pour tout entier strictement positif n.

(ii) Utiliser l’identite de la partie (i) pour expliquer pourquoi chaque entier pair superieur ouegal a 4 fait partie d’un triplet pythagoricien.

Plus d’infos :

Consulte la page du CEMI a la maison mardi, le 16 juin, pour les solutions a ces problemes.

Page 149: CEMI a la maison 9e et 10e ann ee - Lundi 23 mars 2020 ... · 19.Un nombre de Fibonacci. 20.Un multiple de 11. 22.Le plus petit nombre de la grille. 23.Le troisi eme c^ot e d’un

CEMC at Home

Grade 9/10 - Tuesday, June 9, 2020

Pythagorean Triples - Solution

Problem 1: The triple (3, 4, 5) is a Pythagorean triple. Show that (5, 12, 13) and (7, 24, 25) are alsoPythagorean triples.

Solution:

If a = 5, b = 12, and c = 13, then a2 + b2 = 52 + 122 = 25 + 144 = 169 and c2 = 132 = 169.Therefore, a2 + b2 = c2.

If a = 7, b = 24, and c = 25, then a2 + b2 = 72 + 242 = 49 + 576 = 625 and c2 = 252 = 625.Therefore, a2 + b2 = c2.

Problem 2:

(a) Build a Pythagorean triple that includes the odd integer 9 by following these steps:

(i) Determine n such that 92 = 2n + 1. (Answer: n = 81−12

= 40.)

(ii) Verify that (n + 1)2 − n2 = 92. (Answer: 412 − 402 = 1681 − 1600 = 81 = 92.)

(iii) Write down a Pythagorean triple for which the smallest integer is 9. (Answer: Since412 − 402 = 92, we have 92 + 402 = 412 and so (9, 40, 41) is a Pythagorean triple.)

(b) Build a Pythagorean triple that includes the odd integer 11 by following these steps:

(i) Determine n such that 112 = 2n + 1.

(ii) Verify that (n + 1)2 − n2 = 112.

(iii) Write down a Pythagorean triple for which the smallest integer is 11.

(c) Use the ideas from (a) and (b) to build Pythagorean triples that include the next four oddintegers: 13, 15, 17, 19.

Solution:

(b) (i) We have 112 = 121 = 2n + 1 exactly when n = 121−12

= 60.

(ii) When n = 60 we have (n + 1)2 − n2 = 612 − 602 = 3721 − 3600 = 121 = 112.

(iii) Since 612 − 602 = 112, we have 112 + 602 = 612 and so (11, 60, 61) is a Pythagorean tripleinvolving the integer 11.

(c) For the integer 13: We have 132 = 169 = 2n + 1 exactly when n = 169−12

= 84. We can verifythat 852 − 842 = 132 which means 132 + 842 = 852 and so (13, 84, 85) is a Pythagorean tripleinvolving the integer 13.

Using this same method, we can also obtain the following Pythagorean triples:

(15, 112, 113), (17, 144, 145), (19, 180, 181)

Page 150: CEMI a la maison 9e et 10e ann ee - Lundi 23 mars 2020 ... · 19.Un nombre de Fibonacci. 20.Un multiple de 11. 22.Le plus petit nombre de la grille. 23.Le troisi eme c^ot e d’un

Problem 3:

(a) Consider the Pythagorean triple (3, 4, 5). Show that if you multiply each integer in the tripleby 2, then you obtain another Pythagorean triple.

Solution:

If we multiply each integer in the triple (3, 4, 5) by 2, then we obtain the triple (6, 8, 10). Wecan check that this triple is a Pythagorean triple as follows: 62 + 82 = 36 + 64 = 100 = 102.

(b) Use the idea from (a) to build another Pythagorean triple that includes the odd integer 9.

Solution:

If we multiply each integer in the triple (3, 4, 5) by 3, then we obtain the triple (9, 12, 15). Wecan check that this triple is a Pythagorean triple as follows: 92 + 122 = 81 + 144 = 225 = 152.

Note that we have now found two different Pythagorean triples that involve the odd number 9:(9, 40, 41) and (9, 12, 15).

(c) Show that for every positive integer n, the triple (3n, 4n, 5n) is a Pythagorean triple.

It is also true that (5n, 12n, 13n) and (7n, 24n, 25n) are Pythagorean triples.

Solution:

First we note that for every positive n, the numbers 3n, 4n, and 5n are positive integers. Also,we have (3n)2 + (4n)2 = 9n2 + 16n2 = 25n2 = (5n)2. This means that the triple (3n, 4n, 5n) isa Pythagorean triple.

Note: In a similar way, we can show that (5n)2 + (12n)2 = 25n2 + 144n2 = 169n2 = (13n)2 and(7n)2 + (24n)2 = 49n2 + 576n2 = 625n2 = (25n)2.

(d) Use the ideas from Problem 2 and Problem 3 to show that every integer from 4 to 20 is part ofat least one Pythagorean triple.

Solution:

We provide at least one triple for each integer. Some of the triples given below have alreadybeen justified earlier. See if you can determine how the other triples were built using the ideasfrom Problem 2 and Problem 3. For example, the second triple given for 10 was obtainedby multiplying each integer in the Pythagorean triple (5, 12, 13) by 2 and using the idea fromProblem 3(c).

4: (3, 4, 5)

5: (3, 4, 5), (5, 12, 13)

6: (6, 8, 10)

7: (7, 24, 25)

8: (6, 8, 10)

9: (9, 40, 41), (9, 12, 15)

10: (6, 8, 10), (10, 24, 26)

11: (11, 60, 61)

12: (5, 12, 13), (9, 12, 15), (12, 16, 20)

13: (13, 84, 85)

14: (14, 48, 50)

15: (9, 12, 15), (15, 112, 113)

16: (12, 16, 20)

17: (17, 144, 145)

18: (18, 24, 30), (18, 80, 82)

19: (19, 180, 181)

20: (12, 16, 20), (20, 48, 52)

Page 151: CEMI a la maison 9e et 10e ann ee - Lundi 23 mars 2020 ... · 19.Un nombre de Fibonacci. 20.Un multiple de 11. 22.Le plus petit nombre de la grille. 23.Le troisi eme c^ot e d’un

Challenge Problem: Think about how you might use some of these ideas to show that every integerthat is at least 3 is part of a Pythagorean triple. One possible approach is outlined below, but thereare others:

(a) Odd numbers:

(i) Show that for every positive integer n, we have (n + 1)2 − n2 = 2n + 1.

(ii) Use the identity from part (i) to explain why every odd integer that is at least 3 is partof a Pythagorean triple.

Solution:

(i) Method 1: Using the image provided, we see that the area of the largest square is repre-sented by the quantity (n + 1)(n + 1), the area of the medium square is represented bythe quantity (n)(n), the area of each of the two rectangles is represented by the quantity(1)(n), and the area of the smallest square is represented by the quantity (1)(1). Sincethe medium square, small square and two rectangular regions are used to form the largersquare we must have

(n + 1)(n + 1) = (n)(n) + (1)(n) + (1)(n) + (1)(1)

This simplifies to (n+1)2 = n2+2n+1 which can be rearranged to give (n+1)2−n2 = 2n+1.

Method 2: Using the distributive property, we have (n+1)(n+1) = (n+1)(n)+(n+1)(1)which means

(n + 1)2 = (n + 1)(n + 1) = (n + 1)(n) + (n + 1)(1) = n2 + n + n + 1 = n2 + 2n + 1

It follows that (n + 1)2 − n2 = (n2 + 2n + 1) − n2 = 2n + 1.

Method 3: If you have seen the formula for the difference of squares before, then you maysee that

(n + 1)2 − n2 = ((n + 1) + n)((n + 1) − n) = (2n + 1)(1) = 2n + 1

(ii) We follow the method from Problem 2 for a general odd integer k that is greater than 1:Let n = k2−1

2. Since k2 must also be an odd integer that is greater than 1, k2 − 1 must

be an even integer that is greater than 0. This means n is a positive integer. Rearrangingthe equation gives k2 = 2n + 1. Using the formula from (i), we get that

(n + 1)2 − n2 = 2n + 1 = k2

for this value of n. Rearranging the equation above gives

k2 + n2 = (n + 1)2

which shows that (k, n, n+1) is a Pythagorean triple that includes the given odd integer k.

Page 152: CEMI a la maison 9e et 10e ann ee - Lundi 23 mars 2020 ... · 19.Un nombre de Fibonacci. 20.Un multiple de 11. 22.Le plus petit nombre de la grille. 23.Le troisi eme c^ot e d’un

(b) Even numbers:

(i) Show that for every positive integer n, we have (n + 2)2 − n2 = 4n + 4.

(ii) Use the identity from part (i) to explain why every even integer that is at least 4 is partof a Pythagorean triple.

Solution:

(i) Since (n+ 2)2 = (n+ 2)(n+ 2) = (n+ 2)(n) + (n+ 2)(2) = n2 + 2n+ 2n+ 4 = n2 + 4n+ 4we have (n + 2)2 − n2 = (n2 + 4n + 4) − n2 = 4n + 4.

(ii) To show this you can follow the method from part (a) of the challenge problem. We donot give a full solution here, but instead outline the steps using an example.

We can build a Pythagorean triple that includes the even integer 6 by following these steps:

• Determine n such that 62 = 4n + 4:Solving we get n = 36−4

4= 8.

• From part (i) above, we know that for this n we will have (n + 2)2 − n2 = 62.We can verify this directly: (8 + 2)2 − 82 = 102 − 82 = 100 − 64 = 36 = 62.

• This works shows that (6, 8, 10) is a Pythagorean triple.

Suppose that k is an even integer that is greater than 2. If you can find find a positive integern such that k2 = 4n + 4, then the steps above show that (k, n, n + 2) is a Pythagorean triple.Can you see why there will always be such a value of n?

If k > 2 and is even then k2 > 4 and is a multiple of 4, and so k2− 4 > 0 and is a multiple of 4.It follows that n = k2−4

4is a positive integer and satisfies k2 = 4k + 4 as needed!

Parts (a) and (b) of the challenge problem show that every integer that is at least 3 is part of aPythagorean triple. Can you explain why the integers 1 and 2 cannot be part of Pythagorean triples?

Page 153: CEMI a la maison 9e et 10e ann ee - Lundi 23 mars 2020 ... · 19.Un nombre de Fibonacci. 20.Un multiple de 11. 22.Le plus petit nombre de la grille. 23.Le troisi eme c^ot e d’un

CEMC.UWATERLOO.CA | le CENTRE d’ÉDUCATION en MATHÉMATIQUES et en INFORMATIQUE

Le CEMI a la maison

9e et 10e annee - le mercredi 10 juin 2020

Des bases interplanetaires

Tu fais partie d’une mission interplanetaire dont le but est de cataloguer le nombre d’elements presentssur deux des lunes de Jupiter. Six astronautes differents ont rapporte le nombre d’elements qu’ils ontdecouverts mais, pour te jouer un tour, ils ont rapporte leurs decouvertes en utilisant des systemes denumeration differents de ceux auxquels tu es habitue. Pire encore, ils refusent de devoiler les detailsdes systemes de numeration qu’ils ont utilises.

AstronauteNombre total

d’elements

decouverts

Base

utilisee

Nombre d’elements

decouverts sur la

Lune 1

Nombre d’elements

decouverts sur la

Lune 2

Afon 131Breanna 105Cheng 221Denisa 56

Eka 105Fergus 221

Chacun des astronautes a rapporte son nombre en utilisant un systeme de numeration en base b.

On utilise normalement un systeme de numeration en base 10. Dans un tel systeme, tu peux utiliserles chiffres 0, 1, 2, 3, 4, 5, 6, 7, 8 ou 9 comme les chiffres de tes nombres. Dans un systeme de numerationen base 10, le nombre 763 represente l’entier ayant la valeur suivante :

7 × 100 + 6 × 10 + 3 × 1 = 7 × 102 + 6 × 101 + 3 × 100

Chacun des astronautes a rapporte son nombre en utilisant un systeme de numeration en base b, betant un entier strictement positif qui verifie 4 ≤ b ≤ 7. Dans un systeme de numeration en base b,les astronautes peuvent utiliser les chiffres 0, 1, . . . , (b− 1) comme les chiffres de leurs nombres. Dansun tel systeme, le nombre ABC (A, B et C etant des chiffres) represente l’entier ayant la valeursuivante :

A× b2 + B × b + C × 1 = A× b2 + B × b1 + C × b0

Par exemple, dans un systeme de numeration en base b = 4, le nombre 203 represente l’entier ayantla valeur suivante :

2 × 42 + 0 × 4 + 3 × 1.

Or, des nombres tels que 124 et 552 n’ont aucun sens dans ce systeme car ils contiennent des chiffressuperieurs a 3.

Probleme 1 : Les astronautes t’ont donne deux indices quant a leurs rapports :

— Exactement deux des astronautes ont rapporte leurs nombres en utilisant un systeme denumeration en base b = 4.

— Chacun des six astronautes a decouvert le meme nombre d’elements au total.

Determine combien d’elements ont ete decouverts au total sur les deux lunes (utilise le systemede numeration habituel en base 10 pour donner ta reponse) et determine quelle base chacun desastronautes utilisait pour rendre compte de ses decouvertes.

Page 154: CEMI a la maison 9e et 10e ann ee - Lundi 23 mars 2020 ... · 19.Un nombre de Fibonacci. 20.Un multiple de 11. 22.Le plus petit nombre de la grille. 23.Le troisi eme c^ot e d’un

CEMC.UWATERLOO.CA | le CENTRE d’ÉDUCATION en MATHÉMATIQUES et en INFORMATIQUE

Probleme 2 : On te dit maintenant que chaque astronaute a decouvert 16 elements differents surla Lune 1 et que les deux lunes avaient 3 elements en commun (ces nombres sont en base 10). Utilisecette information pour remplir les cellules vides dans le tableau ci-dessus. Assure-toi d’ecrire lesnombres en utilisant la bonne base !

Plus d’infos :

Consulte la page du CEMI a la maison mercredi, le 17 juin, pour les solutions a ces problemes.

Page 155: CEMI a la maison 9e et 10e ann ee - Lundi 23 mars 2020 ... · 19.Un nombre de Fibonacci. 20.Un multiple de 11. 22.Le plus petit nombre de la grille. 23.Le troisi eme c^ot e d’un

CEMC at Home

Grade 9/10 - Wednesday, June 10, 2020

Interplanetary Bases - Solution

Set-up: Six different astronauts have reported the number of elements they discovered on two moons,but reported their findings using different number systems than you are used to.

AstronautTotal Number of

Elements Discovered

Base

used

Number of Elements

Discovered on Moon 1

Number of Elements

Discovered on Moon 2

Afon 131Breanna 105Cheng 221Denisa 56

Eka 105Fergus 221

Each astronaut used a number system with a positive integer b as the base, with 4 ≤ b ≤ 7. In thebase b system, they can use any of the digits 0, 1, . . . , (b− 1) to form the digits in their numbers. Inthis base, the numeral ABC (where A, B, and C are digits) represents the integer with value

A× b2 + B × b + C × 1 = A× b2 + B × b1 + C × b0

Problem 1: The astronauts gave you two clues about the overall report:

• Exactly two of the astronauts reported their numbers in base b = 4.

• All six astronauts discovered (and reported discovering) the same number of elements in total.

Determine how many elements were discovered on the two moons, in total, and determine which baseeach of the astronauts was using to report their findings.

Solution:

If two astronauts reported their findings using the same number system, then they will have reportedidentical numerals. The only numerals that appear more than once in the table above are 105 and221. This means one of these two numerals must represent the total number of elements discoveredwritten in base b = 4. Since a base 4 representation of a number can only use the digits 0, 1, 2, and3, the numeral 105 cannot be a base 4 representation of any number. This means the total numberof elements discovered is represented in base 4 as 221. In base 4, the numeral 221 represents theinteger 2 × 42 + 2 × 4 + 1 = 32 + 8 + 1 = 41.

We now know that Cheng and Fergus reported their findings in base 4, and that there were 41elements discovered in total. There are several ways to approach the problem from here.

Consider the numeral 131. We know this is not in base 4 because the total in base 4 is representedby 221. The largest digit in 131 is 3, so this numeral makes sense in all three other bases: 5, 6, and7. In base 5, it represents the integer 1 × 52 + 3 × 51 + 1 = 25 + 15 + 1 = 41. Notice that in base 6,the numeral 131 would represent the integer 1 × 62 + 3 × 6 + 1 = 36 + 18 + 1 = 55 and in base 7 itwould represent 72 + 3 × 7 + 1 = 71. We conclude that the numeral 131 is in base 5 and that Afonused base 5.

This means the numeral 105 represents the total in either base 6 or base 7. In base 6, it representsthe integer 1×62 +0×6+5 = 36+5 = 41. In base 7, it represents the integer 1×72 +0×7+5 = 54.We conclude that the numeral 105 is in base 6 and so Breanna and Eka used base 6.

Page 156: CEMI a la maison 9e et 10e ann ee - Lundi 23 mars 2020 ... · 19.Un nombre de Fibonacci. 20.Un multiple de 11. 22.Le plus petit nombre de la grille. 23.Le troisi eme c^ot e d’un

Finally, we suspect that 56 is in base 7, and indeed, 5 × 7 + 6 = 41, so we conclude that Denisareported in base 7.

We can also solve this problem using algebra once we have determined that the total number ofelements is 41. For example, we know that the numeral 56 is the base b representation of 41 for someb between 4 and 7 inclusive. This means 41 = 5b + 6, or 35 = 5b, which can be solved for b to getb = 7. Similarly, to determine in which base the numeral 131 represents the number 41, we can solvethe equation 41 = 1 × b2 + 3 × b + 1 for b. Rearranging, we get 40 = b2 + 3b. If you have experiencefactoring quadratics, you can solve this equation for b. If not, since you only have four possibilitiesfor b, you can check b = 4, 5, 6, 7 and find that b = 5 is the only solution among these four choices.Finally, the numeral reported by Breanna leads to the equation 41 = b2 + 5 or 36 = b2. Since b ispositive, this means b = 6.

Problem 2: You are now told that each astronaut discovered 16 distinct elements on Moon 1 anddiscovered 3 elements common to both moons, with these numbers given in base 10. Using thisinformation, fill in the blank cells in the table above.

Solution:

The number of elements that were discovered on only Moon 1 is 16 − 3 = 13, which means theremust have been 28 elements discovered on Moon 2. Filling in the table means finding the base 4, 5,6, and 7 representations of 16 and 28. These representations are given in the table below. To helpexplain how these numerals were obtained, we show the work for the first row below the table.

AstronautTotal Number of

Elements Discovered

Base

used

Number of Elements

Discovered on Moon 1

Number of Elements

Discovered on Moon 2

Afon 131 5 31 103Breanna 105 6 24 44Cheng 221 4 34 130Denisa 56 7 22 40

Eka 105 6 24 44Fergus 221 4 34 130

To represent the integer 16 in base 5, we first note that its base 5 representation must have at mosttwo digits. This is because the numeral ABC represents the integer with value A× 52 + B × 5 + Cwhich is at least 25 (assuming A is a positive digit). Thus, we seek digits A and B between 0 and4 inclusive so that A × 5 + B = 16. It is easy to check that digits A = 3 and B = 1 satisfy thisequation, and in fact, it is true that no other pair of integers between 0 and 4 inclusive satisfies theequation. Therefore, the numeral 31 is the base 5 representation of the integer 16.

To represent the integer 28 in base 5, notice that 53 and hence all larger powers of 5 are greater than28, so the base 5 representation of 28 has at most three digits. As well, the two-digit numeral withthe largest value in base 5 is 44 which represents the integer 4 × 5 + 4 = 24, so this means the base5 representation of 28 has exactly three digits. Therefore, we seek integers A, B, and C all between0 and 4 inclusive satisfying A × 52 + B × 51 + C × 50 = 28 or 25A + 5B + C = 28. We know thatA ≥ 1 since the representation has three digits, but if A ≥ 2, then 25A ≥ 50 > 28, so this means wemust have A = 1. The equation then simplifies to 5B+C = 3, and the only solution to this equationwhere B and C are integers between 0 and 4 inclusive is B = 0 and C = 3. Therefore, the base 5representation of the integer 28 is 103.

There are systematic ways of expressing numbers in various bases. You may wish to do an internetsearch to learn more about this.

Page 157: CEMI a la maison 9e et 10e ann ee - Lundi 23 mars 2020 ... · 19.Un nombre de Fibonacci. 20.Un multiple de 11. 22.Le plus petit nombre de la grille. 23.Le troisi eme c^ot e d’un

CEMC.UWATERLOO.CA | le CENTRE d’ÉDUCATION en MATHÉMATIQUES et en INFORMATIQUE

Le CEMI a la maison

9e et 10e annee - le jeudi 11 juin 2020

PRODUITivite

Le produit des chiffres d’un nombre entier positif est le produit de tous les chiffres qui composent cenombre.

Par exemple, le produit des chiffres du nombre 234 est 2×3×4 = 24. Il existe d’autres nombres dontle produit des chiffres est aussi 24. Par exemple, 2233, 113181 et 38, ont tous un produit des chiffresegal a 24. Le nombre 38 est le plus petit nombre entier positif dont le produit des chiffres est 24.

Il existe plusieurs nombres entiers positifs dont le produit des chiffres est 2000.

Determine le plus petit nombre entier positif dont le produit des chiffres de 2000.

Plus d’infos :

Consulte la page du CEMI a la maison vendredi, le 12 juin, pour la solution a PRODUITivite.

Cette ressource du CEMI a la maison est un probleme passe du Probleme de la semaine. Le Problemede la semaine est une ressource hebdomadaire gratuite que le CEMI met a la disposition des ensei-gnant(e)s, des parents et des eleves pendant l’annee scolaire. Les publications du Probleme de lasemaine sont terminees pour cette annee scolaire en cours et reprendront le 17 septembre 2020. Pourt’abonner et consulter les problemes passes et leurs solutions, visite :https://www.cemc.uwaterloo.ca/resources/potw-f.php

1

Page 158: CEMI a la maison 9e et 10e ann ee - Lundi 23 mars 2020 ... · 19.Un nombre de Fibonacci. 20.Un multiple de 11. 22.Le plus petit nombre de la grille. 23.Le troisi eme c^ot e d’un

CEMC at Home

Grade 9/10 - Thursday, June 11, 2020

PRODUCTivity - Solution

Problem:

The “digit product” of a positive integer is the product of the individual digits of the integer.

For example, the digit product of 234 is 2 × 3 × 4 = 24. Other numbers also have a digit productof 24. For example, 2223, 113 181 and 38 each have a digit product of 24. The number 38 is thesmallest positive integer with a digit product of 24.

There are many positive integers whose digit product is 2000.

Determine the smallest positive integer whose digit product is 2000.

Solution:

Let N be the smallest positive integer whose digit product is 2000.

In order to find N , we must find the minimum possible number of digits whose product is 2000. Thisis because if the integer a has more digits than the integer b, then a > b.Once we have determined the digits that form N , then the integer N is formed by writing thosedigits in increasing order.

Note that the digits of N cannot include 0, or else the digit product of N would be 0.Also, the digits of N cannot include 1, otherwise we could remove the 1 and obtain an integer withfewer digits (and thus, a smaller integer) with the same digit product. Therefore, the digits of N willbe between 2 and 9, inclusive.

Since the digit product of N is 2000, we will use the prime factorization of 2000 to help determinethe digits of N :

2000 = 24 × 53

In order for a digit to have a factor of 5, the digit must equal 5. Therefore, three of the digits of Nare 5.

The remaining digits of N must have a product of 24 = 16. We need to find a combination of thesmallest number of digits whose product is 16. We cannot have one digit whose product is 16, butwe can have two digits whose product is 16. In particular, 16 = 2 × 8 and 16 = 4 × 4.

Therefore, N has 5 digits. They are 5, 5, 5, 2, 8 or 5, 5, 5, 4, 4. In order for N to be as small aspossible, its digits must be in increasing order. The smallest positive integer formed by the digits 5,5, 5, 2, 8 is 25 558. The smallest positive integer formed by the digits 5, 5, 5, 4, 4 is 44 555.

Since 25 558 < 44 555, the smallest N is 25 558. That is, the smallest positive integer with a digitproduct of 2000 is 25 558.

1

Page 159: CEMI a la maison 9e et 10e ann ee - Lundi 23 mars 2020 ... · 19.Un nombre de Fibonacci. 20.Un multiple de 11. 22.Le plus petit nombre de la grille. 23.Le troisi eme c^ot e d’un

CEMC.UWATERLOO.CA | le CENTRE d’ÉDUCATION en MATHÉMATIQUES et en INFORMATIQUE

Le CEMI a la maison

9e et 10e annee - le vendredi 12 juin 2020

Des morceaux d’une etoile

Un polygone est une figure fermee a deux dimensions formee de segments de droites. Un polygonesimple est un polygone dont les segments de droites qui le forment ne se coupent pas. Pensons a lamaniere dont on pourrait tracer les bords d’un polygone simple : on se deplace uniquement en lignesdroites et la seule fois ou on reviendrait a un point pour la deuxieme fois serait lorsqu’on auraittermine de tracer tous les bords du polygone.

Probleme :

On trouve 37 polygones simples dans l’etoile a 5 branchesci-contre. Decris les 37 polygones et explique pourquoi il n’yen a pas plus.

Les cotes de tes polygones doivent tous etre des segmentsde droites de l’etoile. Pour t’aider a commencer, voici troisexemples de polygones simples que l’on peut trouver dansl’etoile, ainsi qu’un polygone qui n’est pas un polygone simple.

Trois polygones simples(permis)

Un polygonequi n’est pas simple

(interdit)

Conseils pour compter les polygones :

1. Quels sont les polygones simples que tu peux trouver qui ne sont pas isometriques aux polygonespresentes dans les exemples ci-dessus ?

2. Combien de polygones simples n’incluent pas l’interieur du pentagone ?

3. Le nombre de polygones simples qui incluent l’interieur du pentagone est une puissance de 2.

Discussion de suivi :

As-tu pu trouver un moyen systematique de compter les polygones simples dans l’etoile a 5 branches ?Cette meme strategie pourrait-elle etre utilisee pour compter les polygones simples dans une etoile a6 branches ou dans une etoile a 7 branches ?

En general, une etoile a n branches est composee d’un n-gone au centre et de n triangles attaches ases cotes et pointant vers l’exterieur. Pensez a la question suivante :

Question complementaire : Combien de polygones simples existe-t-il dans une etoile a n branches ?

Plus d’infos : Consulte la page du CEMI a la maison vendredi, le 19 juin, pour la solution a ceprobleme.

Page 160: CEMI a la maison 9e et 10e ann ee - Lundi 23 mars 2020 ... · 19.Un nombre de Fibonacci. 20.Un multiple de 11. 22.Le plus petit nombre de la grille. 23.Le troisi eme c^ot e d’un

CEMC at Home

Grade 9/10 - Friday, June 12, 2020

Pieces of a Star - Solution

Problem: There are 37 simple polygons to be found in the 5-pointed star shown. Describe the 37simple polygons and explain why there are no more.

Example 1 Example 2

Solution: First, we note that there are 5 simple polygons in the star that do not include the pentagonin the centre: the 5 triangles that form the 5 points of the star. One such triangle is shown inExample 1 above.

Every other simple polygon in the star will consist of the pentagon in the centre plus some of the 5triangles which form the points of the star. One of these polygons is formed by choosing none of thetriangles. In this case we get the pentagon in the centre. Another of these polygons is formed bychoosing all of the triangles. In this case we get the entire 5-pointed star (or its boundary). Anotherof these polygons is formed by choosing the two triangles at the top right. In this case we get thepentagon shown in Example 2.

We could try to systematically draw all of these polygons, but we can count them without doing so.Each such polygon either includes a particular triangle or it does not. This means when drawing oneof these polygons, we have two choices for each triangle: in or out. This leads to 25 = 32 differentpossibilities for which triangles we include, and hence there must be 32 simple polygons that includethe pentagon in the centre.

Putting this all together, there are 5 simple polygons that do not include the pentagon (the 5 outertriangles) and 32 simple polygons that include the pentagon. Therefore, there are 5 + 32 = 37 simplepolygons in the star, in total.

Follow-up Question: How many simple polygons are there in an n-pointed star?

Solution: We can think of an n-pointed star as an n-gon with n triangles around it. The n trianglesform the n points of the star.

Some of the simple polygons in the star are the n triangles that form the n points of the star. Everyother simple polygon will consist of the n-gon in the centre plus some of the n triangles that formthe points of the star. Since there are n triangles, and for each triangle we have 2 choices (includeor not include), there are 2n such polygons.

Therefore, in total, we have n + 2n simple polygons in the n-pointed star.

In particular, when n = 6 we have 6 + 26 = 70 and so there are 70 simple polygons in the 6-pointedstar, and when n = 7 we have 7+27 = 135 and so there are 135 simple polygons in the 7-pointed star.

Page 161: CEMI a la maison 9e et 10e ann ee - Lundi 23 mars 2020 ... · 19.Un nombre de Fibonacci. 20.Un multiple de 11. 22.Le plus petit nombre de la grille. 23.Le troisi eme c^ot e d’un

CEMC.UWATERLOO.CA | le CENTRE d’ÉDUCATION en MATHÉMATIQUES et en INFORMATIQUE

Le CEMI a la maison

9e et 10e annee - le lundi 15 juin 2020

L’histoire de l’informatique

Les ordinateurs se trouvent sur nos bureaux, dans nos poches et meme dans nos refrigerateurs ! C’estremarquable car les ordinateurs modernes existent depuis moins de 100 ans. Pendant cette periode,on a observe un flux constant de nouvelles decouvertes et de progres technologiques.

Utilise cet outils en ligne pour retablir la liste suivante des evenements de l’histoire de l’infor-matique, du plus ancien au plus recent.

A. Deep Blue est le premier programme informatique qui bat un champion du monde d’echecs.

B. Construction de l’ordinateur mecanique Harvard Mark I, il est utilise a des fins militairespendant la seconde guerre mondiale.

C. Sun Microsystems developpe le langage de programmation Java.

D. L’ASCII est developpe pour creer des codes binaires standards pour 128 caracteres differents.

E. Les ordinateurs sont utilises pour determiner qu’une strategie gagnante parfaite n’existe paspour le jeu de dames.

F. Le premier courrier electronique (courriel) est envoye. Il est envoye par Ray Tomlinson a RayTomlinson.

G. Konrad Zuse concoit l’ordinateur electromecanique Z3 qui est considere comme le premierordinateur programmable automatique.

H. L’Altair 8800 est le premier ordinateur personnel a etre vendu en grand nombre.

I. Construction du robot nomme Elektro, il repond aux commandes vocales.

J. Guido van Rossum cree et diffuse le langage de programmation Python.

K. Doug Engelbart invente la souris d’ordinateur.

L. Creation de Cindy, le premier personnage de film en effets speciaux numeriques (Computer-generated imagery, abrege CGI en anglais) a caractere humain.

Plus d’infos :

Notre page internet Computer Science and Learning to Program est le meilleur endroit pour trouverles ressources informatiques du CEMI.

1

Page 162: CEMI a la maison 9e et 10e ann ee - Lundi 23 mars 2020 ... · 19.Un nombre de Fibonacci. 20.Un multiple de 11. 22.Le plus petit nombre de la grille. 23.Le troisi eme c^ot e d’un

CEMC.UWATERLOO.CA | le CENTRE d’ÉDUCATION en MATHÉMATIQUES et en INFORMATIQUE

Le CEMI a la maison

9e et 10e annee - le mardi 16 juin 2020

Peux-tu trouver les termes ?

Peux-tu trouver tous les termes mathematiques et d’informatique enumeres ci-dessous ?Bonne chance !

EXPOSANT MILIEU SYNTAXEPOLYNOMIALE SOMMET ALGORITHMEPENTE TABLEAU CONDITIONNELPARABOLE BOUCLE FONCTION

FACTORISATION BOOLEEN TESTS

Plus d’infos :

Consulte la page du CEMI a la maison mercredi, le 17 juin, pour les solutions a Peux-tu trouver lestermes ?

1

Page 163: CEMI a la maison 9e et 10e ann ee - Lundi 23 mars 2020 ... · 19.Un nombre de Fibonacci. 20.Un multiple de 11. 22.Le plus petit nombre de la grille. 23.Le troisi eme c^ot e d’un

CEMC.UWATERLOO.CA | le CENTRE d’ÉDUCATION en MATHÉMATIQUES et en INFORMATIQUE

Le CEMI a la maison

9e et 10e annee - le mardi 16 juin 2020

Peux-tu trouver les termes ? - solutions

EXPOSANT MILIEU SYNTAXEPOLYNOMIALE SOMMET ALGORITHMEPENTE TABLEAU CONDITIONNELPARABOLE BOUCLE FONCTION

FACTORISATION BOOLEEN TESTS

1

Page 164: CEMI a la maison 9e et 10e ann ee - Lundi 23 mars 2020 ... · 19.Un nombre de Fibonacci. 20.Un multiple de 11. 22.Le plus petit nombre de la grille. 23.Le troisi eme c^ot e d’un

CEMC.UWATERLOO.CA | le CENTRE d’ÉDUCATION en MATHÉMATIQUES et en INFORMATIQUE

Le CEMI a la maison

9e et 10e annee - le mercredi 17 juin 2020

Le classement

Dans une ligue de balle-molle a quatre equipes, chaque equipe joue 4 fois contre chacune des autresequipes.

Les equipes gagnent 3 points pour une victoire, 1 point pour un match nul et aucun point pour unedefaite.

Les points accumules a la fin de la saison sont :

Lions 22Tigers 19Mounties 14Royals 12

Dans combien de matchs y a-t-il eu une victoire et combien de matchs se sont termines a egalite ?

Plus d’infos :

Consulte la page du CEMI a la maison jeudi, le 18 juin, pour la solution au Classement.

Cette ressource du CEMI a la maison est un probleme passe du Probleme de la semaine. Le Problemede la semaine est une ressource hebdomadaire gratuite que le CEMI met a la disposition des ensei-gnant(e)s, des parents et des eleves pendant l’annee scolaire. Les publications du Probleme de lasemaine sont terminees pour cette annee scolaire en cours et reprendront le 17 septembre 2020. Pourt’abonner et consulter les problemes passes et leurs solutions, visite :https://www.cemc.uwaterloo.ca/resources/potw-f.php

1

Page 165: CEMI a la maison 9e et 10e ann ee - Lundi 23 mars 2020 ... · 19.Un nombre de Fibonacci. 20.Un multiple de 11. 22.Le plus petit nombre de la grille. 23.Le troisi eme c^ot e d’un

CEMC at Home

Grade 9/10 - Wednesday, June 17, 2020

The Standings - Solution

Problem:

In a softball league with four teams, each team has played every other team 4 times.

Each team earned 3 points for a win, 1 point for a tie and no points for a loss.

The total accumulated points were:

Lions 22Tigers 19Mounties 14Royals 12

How many games ended in a win and how many games ended in a tie?

Solution:

We begin by calculating the total number of games played. Since each team played every other team4 times, each team played 3 × 4 = 12 games. Since there are four teams, a total of 4×12

2= 24 games

were played. We divide by 2 since each game is counted twice. For example, the Lions playing theTigers is the same as the Tigers playing the Lions.

In games where one team won and one team lost, one team earned 3 points and the other 0 points,so a total of 3 points were awarded. In games that resulted in a tie, both teams earned 1 point, so atotal of 2 points were awarded.

If there were 0 ties, then 24 games would result in 24 × 3 = 72 points being awarded. However,22 + 19 + 14 + 12 = 67 points were actually awarded in all of the games. Since a total of 3 pointswere awarded when there was a win and a total of 2 points were awarded when there was a tie, everypoint below 72 must represent a tie. Since 72 − 67 = 5, there must have been 5 ties. Since 24 gameswere played, 24 − 5 = 19 games resulted in a win.

Therefore, there were 19 games that ended in a win and 5 games ended in a tie.

We should check that there is a combination of wins, ties and losses that satisfies the conditions inthe problem. Indeed, one possibility is:

Team Name Wins Ties Losses Total PointsLions 7 1 4 22Tigers 6 1 5 19

Mounties 3 5 4 14Royals 3 3 6 12

TOTALS 19 10 19 67

Notice that in the chart there are a total of 10 ties. That means that 5 games ended in a tie and atotal of 10 points were awarded for ties.

1

Page 166: CEMI a la maison 9e et 10e ann ee - Lundi 23 mars 2020 ... · 19.Un nombre de Fibonacci. 20.Un multiple de 11. 22.Le plus petit nombre de la grille. 23.Le troisi eme c^ot e d’un

CEMC.UWATERLOO.CA | le CENTRE d’ÉDUCATION en MATHÉMATIQUES et en INFORMATIQUE

Le CEMI a la maison

9e et 10e annee - le jeudi 18 juin 2020

Jeux et puzzles

Le CEMI a cree de nombreuses ressources au cours des derniers mois et nous esperons que vousles avez trouvees interessantes. Nous savons egalement qu’il existe de nombreux jeux et puzzles enligne crees par d’autres organisations qui ont trait aux mathematiques et a la logique. Nous avonsmis en evidence trois exemples ci-dessous que tu peux explorer pour t’amuser davantage avec lesmathematiques !

� Fraction Game � de NCTM (https://www.nctm.org)

Dans ce jeu qui a trait aux fractions, tu dois utiliser la logique et le sens du nombre poureffectuer des mouvements.

� The Remainders Game � de NRICH (https://nrich.maths.org)

Utilise tes connaissances des restes pour determiner un nombre mysterieux.

� Slitherlink Puzzles � par Krazydad (https://krazydad.com)

Dans un puzzle Slither Link, l’objectif est de relier horizontalement ou verticalement les pointsadjacents afin de creer un chemin sinueux qui ne forme qu’une seule boucle. De plus, ce cheminne peut se couper lui-meme ni bifurquer.

Il existe d’autres jeux et puzzles en ligne qui ont trait aux mathematiques et a la logique. Noust’encourageons a partager tes jeux et puzzles preferes dans des forums avec lesquels tu es a l’aise.

Page 167: CEMI a la maison 9e et 10e ann ee - Lundi 23 mars 2020 ... · 19.Un nombre de Fibonacci. 20.Un multiple de 11. 22.Le plus petit nombre de la grille. 23.Le troisi eme c^ot e d’un

CEMC.UWATERLOO.CA | le CENTRE d’ÉDUCATION en MATHÉMATIQUES et en INFORMATIQUE

Le CEMI a la maison

9e et 10e annee - le vendredi 19 juin 2020

Journee de relais - 1re partieDans le Concours de mathematiques canadien par equipe du CEMI, les eleves participent a uneversion mathematique d’une course de relais. Tout comme une course de relais, les coequipier(-iere)sse relayent au moyen d’un temoin pour terminer la course. Dans le cadre d’un relais mathematique, letemoin que les coequipier(-iere)s se relayent est un nombre ! Lis attentivement les problemes suivants.

Probleme 1 : Soit A le nombre de multiples de 5 situes dans l’intervalle de 1 a 2020 et B lenombre de multiples de 20 situes dans l’intervalle de 1 a 2020. Quelle est la valeur de 10A÷B ?

Probleme 2 : Remplace le N ci-dessous avec le nombre que tu as obtenu.Dans la figure ci-dessous, quatre segments de droites se coupent en points A,B,C,D et E. Lamesure de l’angle CED est egale a x◦. Quelle est la valeur de x ?

Probleme 3 : Remplace le N ci-dessous avec le nombre que tu as obtenu.Armen a paye 190 $ pour acheter des billets de cinema pour un groupe de N personnes composed’adultes et d’enfants. Les billets de cinema coutent 5 $ pour les enfants et 9 $ pour les adultes.Combien de billets pour enfants a-t-il achetes ?

Note que tu peux repondre au Probleme 1 sans aucune information supplementaire.

Afin de repondre au Probleme 2, tu dois d’abord determiner la valeur mystere de N . La valeur de Nutilisee dans le Probleme 2 sera la reponse obtenue dans le Probleme 1. (Par exemple, si tu obtiens5 comme reponse au Probleme 1, alors tu remplaceras le N dans le Probleme 2 par 5.)

De meme, il te faut la reponse au Probleme 2 pour repondre au Probleme 3. La valeur de N utiliseedans le Probleme 3 sera la reponse obtenue dans le Probleme 2.

Essaie maintenant le relais ! Tu peux utiliser cet outil pour verifier tes reponses.

Activite de suivi : Peux-tu creer ton propre relais mathematique ?A quoi dois-tu penser en creant les trois problemes du relais ?

Dans la partie 1 de cette ressource, tu completes un relais seul(e). Or, comme on le sait bien, lescourses de relais s’effectuent en equipe ! Dans un relais en equipe, trois personnes differentes repondentaux problemes. Le (la) joueur(-euse) 1 repond au probleme 1 et passe sa reponse au (a la) joueur(-euse) 2 ; qui l’utilise pour repondre au probleme 2 ; le (la) joueur(-euse) 2 passe sa reponse au (a la)joueur(-euse) 3 ; et ainsi de suite.Dans la partie 2 de cette ressource, tu trouveras des instructions te permettant de lancer un jeu derelais avec tes ami(e)s et ta famille ! Un relais a utiliser est fourni, mais tu peux aussi creer le tien !

Page 168: CEMI a la maison 9e et 10e ann ee - Lundi 23 mars 2020 ... · 19.Un nombre de Fibonacci. 20.Un multiple de 11. 22.Le plus petit nombre de la grille. 23.Le troisi eme c^ot e d’un

CEMC.UWATERLOO.CA | le CENTRE d’ÉDUCATION en MATHÉMATIQUES et en INFORMATIQUE

Le CEMI a la maison

De la 4e a la 12e annee - le vendredi 19 juin 2020

Journee de relais - 2eme partie

Relais pour la famille et les amis

Dans la premiere partie de cette ressource, tu as appris a faire un relais mathematique. Maintenant,pourquoi ne pas en essayer un avec ta famille et tes amis !

Tu peux constituer une equipe de relais et

— jouer juste pour le plaisir, sans faire la course avec une autre equipe, ou ;

— vous mesurer a une autre equipe de votre foyer (si vous avez au moins 6 personnes au total), ou ;

— vous mesurer a une equipe d’une autre famille ou d’un autre foyer en

— chronometrant votre equipe et en comparant les temps avec ceux des autres equipes pourdeclarer l’equipe victorieuse, ou ;

— en affrontant l’autre equipe en direct par videobavardage.

Voici les instructions pour jouer.

Instructions pour le relais :

1. Choisissez une equipe de trois personnes pour le relais. L’equipe participera a la competitionensemble.

2. Trouvez une personne pour vous aider a superviser le relais ; on l’appellera � l’arbitre �.

3. Chaque membre de l’equipe se verra attribuer un numero : 1, 2 ou 3. Le (la) joueur(-euse) 1 severra attribuer le probleme 1, le (la) joueur(-euse) 2 se verra attribuer le probleme 2 et le (la)joueur(-euse) 3 se verra attribuer le probleme 3.

4. Les trois coequipier(-ere)s ne doivent voir aucun des problemes du relais a l’avance et ne doiventpas se parler pendant le relais.

5. Juste avant le debut du relais, l’arbitre doit distribuer le probleme de relais correspondant achacun(e) des joueur(-euse)s, avec l’enonce du probleme face cachee (non visible).

6. L’arbitre signalera le debut du relais. A ce moment, les trois joueur(-euse)s peuvent commencera travailler sur leurs problemes.

Pensez a ce que les joueur(-euses) 2 et 3 peuvent faire avant de recevoir la valeur de N (lareponse a la question precedente qui leur a ete transmise par leur coequipier(-ere)).

7. Lorsque le (la) joueur(-euse) 1 pense avoir la bonne reponse au probleme 1, il ou elle inscritsa reponse sur la feuille de reponses et la transmet au (a la) joueur(-euse) 2. Lorsque le (la)joueur(-euse) 2 pense avoir la bonne reponse au probleme 2, il ou elle ajoute sa reponse sur lafeuille de reponses et la passe au (a la) joueur(-euse) 3. Lorsque le (la) joueur(-euse) 3 penseavoir la bonne reponse au probleme 3, il ou elle inscrit sa reponse sur la feuille de reponses etla transmet a l’arbitre.

Page 169: CEMI a la maison 9e et 10e ann ee - Lundi 23 mars 2020 ... · 19.Un nombre de Fibonacci. 20.Un multiple de 11. 22.Le plus petit nombre de la grille. 23.Le troisi eme c^ot e d’un

CEMC.UWATERLOO.CA | le CENTRE d’ÉDUCATION en MATHÉMATIQUES et en INFORMATIQUE

8. Si les trois reponses transmises a l’arbitre sont correctes, alors le relais est termine ! Si au moinsune reponse est incorrecte, l’arbitre renvoie la feuille au (a la) troisieme joueur(-euse).

9. A tout moment pendant le relais, les membres de l’equipe peuvent se passer la feuille de reponsesentre eux (elles), a condition de n’y ecrire que leurs reponses actualisees et de ne rien discuter.(Par exemple, si le (la) joueur(-euse) 2 est sur(e) que la reponse du (de la) joueur(-euse) 1 estincorrecte, alors il (elle) peut passer la feuille de reponses au (a la) joueur(-euse) 1, en silence.C’est un signal pour qu’il (elle) verifie son travail et essaie a nouveau).

Regardez a la page suivante afin d’y trouver un relais pour la famille et les amis !Des instructions pour l’arbitre sont comprises. Vous pouvez egalement proposer vos propresrelais. Vous pouvez trouver de nombreux autres relais des concours CTMC passes sur la pagedes concours passes du CEMI.

Vous trouverez ci-dessous des exemples de feuilles de reponses que vous pourrez utiliser pour vosrelais si vous le souhaitez.

Feuilles de reponses :

Reponse au probleme 1

Reponse au probleme 2

Reponse au probleme 3

Reponse au probleme 1

Reponse au probleme 2

Reponse au probleme 3

Reponse au probleme 1

Reponse au probleme 2

Reponse au probleme 3

Reponse au probleme 1

Reponse au probleme 2

Reponse au probleme 3

Page 170: CEMI a la maison 9e et 10e ann ee - Lundi 23 mars 2020 ... · 19.Un nombre de Fibonacci. 20.Un multiple de 11. 22.Le plus petit nombre de la grille. 23.Le troisi eme c^ot e d’un

CEMC.UWATERLOO.CA | le CENTRE d’ÉDUCATION en MATHÉMATIQUES et en INFORMATIQUE

Relais a troisInstructions pour l’arbitre :

1. Des questions adaptees aux differents niveaux de difficulte sont donnees selon les differentespositions de relais.

— Assignez l’un des trois premiers problemes (marques � Probleme 1 �) au (a la) joueur(euse) 1.

— Assignez l’un des trois problemes suivants (marques � Probleme 2 �) au (a la) joueur(euse) 2.

— Assignez l’un des trois derniers problemes (marques � Probleme 3 �) au (a la) joueur(euse) 3.

Choisissez un probleme de maniere a ce que chaque participant(e) soit a l’aise avec le niveaude sa question. Le niveau de difficulte de chaque question est represente a l’aide des symbolessuivants :

— Ces questions devraient etre accessibles a la plupart des eleves de 4e annee ou plus.

— Ces questions devraient etre accessibles a la plupart des eleves de 7e annee ou plus.

— Ces questions devraient etre accessibles a la plupart des eleves de 9e annee ou plus.

2. Utilise cet outil pour consulter a l’avance les reponses aux problemes de relais.

Problemes de relais (a decouper) :

Probleme 1

Le graphique montre le nombre de pains que trois amis ont fait cuire. Combien de pains Bo a-t-ellefait cuire ?

Probleme 1

Un triangle equilateral a des cotes de longueur x+ 4, y + 11 et 20. Quelle est la valeur de x+ y ?

Probleme 1

Deux cercles sont dessines sur la figure ci-dessous. Si le rayon du grand cercle est de 10 et que l’airede la region ombree (entre les deux cercles) est 75π, alors quel est le carre du rayon du plus petitcercle ?

Page 171: CEMI a la maison 9e et 10e ann ee - Lundi 23 mars 2020 ... · 19.Un nombre de Fibonacci. 20.Un multiple de 11. 22.Le plus petit nombre de la grille. 23.Le troisi eme c^ot e d’un

Probleme 2

Remplace N ci-dessous par le nombre que tu as recu.Kwame ecrit les chiffres entiers dans l’ordre de 1 a N (en incluant 1 et N). Combien de fois ecrit-ille chiffre � 2 � ?

Probleme 2

Remplace N ci-dessous par le nombre que tu as recu.La masse totale de trois chiens est de 43 kilogrammes. Le plus grand des chiens a une masse de Nkilogrammes et les deux autres chiens ont la meme masse. Quelle est la masse de chacun des pluspetits chiens ?

Probleme 2

Remplace N ci-dessous par le nombre que tu as recu.Les points (6, 16), (8, 22) et (x,N) se trouvent sur une ligne droite. Trouve la valeur de x.

Probleme 3

Remplace N ci-dessous par le nombre que tu as recu.Tu as des boıtes de meme taille et de meme forme. Si N oranges peuvent tenir dans une boıte,combien d’oranges peuvent tenir dans 2 boıtes, au total ?

Probleme 3

Remplace N ci-dessous par le nombre que tu as recu.Un matin, une petite ferme a vendu 10 paniers de tomates, 2 paniers de poivrons et N paniers decourgettes. Si les prix sont ceux indiques ci-dessous, combien d’argent la ferme a-t-elle gagne au totalgrace a ces ventes ?

Panier de tomates : 0,50 $Panier de poivrons : 2,00 $Panier de courgettes : 1,00 $

Probleme 3

Remplace N ci-dessous par le nombre que tu as recu.Elise a N boıtes, chacune contenant x pommes. Elle donne 12 pommes a sa sœur. Elle donne ensuite20% des pommes restantes a son frere. Apres cela, il lui reste 120 pommes. Quelle est la valeur de x ?